Vous êtes sur la page 1sur 56

The Residents Guide to the LMCC II

3rd Edition
The Licentiate of the Medical Council of Canada Exam, part II, also known as the MCCQE II, was the traditional means of
qualifying for a general license to practice medicine in Canada. Now that both the internship year and the general license are
no longer available, many residents view the exam as a stressful and expensive exercise in futility. While the process is
stressful and expensive, it need not be futile. Preparation for the exam can be an enlightening review. Scenarios tend to repeat
over the years, the pass rate is greater than 95% on the first attempt, and there is an option to rewrite, so dont panic.
The exam is an OSCE (Observed Scenario Clinical Exam) in which the candidate progresses through a series of stations. Your
starting point is determined alphabetically. At each station there is a physician examiner and either a real person posing as a
patient or a telephone over which you must speak to a patient or another physician requesting assistance.
The most recent sessions (since 1997) contain six short cases known as 5-minute couplets, in which the candidate is allotted 5
minutes to assess a patient and 5 minutes to write short answers to questions related to the case. There was also a series of six
longer cases in which the candidates were presented with a more involved clinical problem, such as a resuscitation or
psychosocial counseling session, lasting 10 minutes each. The physician examiner may ask one or two questions in the last
minute of a 10-minute station. There is one minute between stations during which you can look at a brief description of the
patient and consider your approach. Occasionally pilot questions will be included in the exam, which will not count towards
the final mark but are used to test new questions. You will not know which questions are pilot questions.
The content of the exam is general medicine. This means family practice & emergency medicine. The following topics appear
consistently:
Pediatrics diarrhea, development, neonatal jaundice, asthma
Obs/Gyn amenorrhea, vaginal blood, abdominal pain, PIH, OCP, elective abortion counseling
Suturing choice of suture, tetanus vaccine
Chest Pain read CXR, ECG
Resuscitation fluid resuscitation after blood loss, ABCDs
Overdose ASA, TCA
Needle stick AIDS, hepatitis, vaccinations
Psychiatry depression, mania, schizophrenia
Neurosurgery back and neck radiculopathies, carpal tunnel
(Note that every history should include name, age, occupation, past medical history, family history, medications, drugs/alcohol,
review of systems)

1997
1. First year university student, 9 weeks pregnant, considering abortion. Take a history and counsel. Findings: tearful,
guilty, sleep disturbance, has not engaged social supports.
History: combine a pregnancy history with a social history and a screen for depression.
Pregnancy: Patient ID (name, age, occupation). GTPAL (number of gestations, term pregnancies, premature births, abortions,
live children), history of problems, if any, with previous pregnancies. Current pregnancy, establish gestational age (GA) by last
menstrual period (LMP) if regular periods and sure dates (if unsure a dating ultrasound would be needed). The GA is the
number of weeks from the first day of the LMP. The EDC is first day of LMP + 7 days 3 months. Ask about use of alcohol,
smoking, drugs, domestic violence (50% begins in pregnancy), maternal illnesses during the pregnancy (particularly diabetes,
rubella, toxoplasmosis, herpes, CMV, thyroid dysfunction, HTN, hypercoagulation). Use of birth control, if any. Past medical
history, family history of pregnancy related problems, medications.
Social: Status of any relationships at present including relationship with the childs father. Social supports (family, friends,
boyfriend), do they know? Are they helping? Employment/financial/educational status of the patient, does the patient feel
prepared to raise a child?

Psychiatric: How does the patient feel about this decision? How is she coping? Cover mnemonic for major depression.
MSIGECAPS: mood (depressed), sleep (increased or decreasedif decreased, often early morning awakening), interest
(decreased), guilt/worthlessness, energy (decreased or fatigued), concentration/difficulty making decisions, appetite and/or
weight increase or decrease, psychomotor activity (increased or decreased), suicidal ideation positive diagnosis of major
depression requires five of these over a 2 week period, one of the five must be loss of interest or depressed mood. Symptoms
do not meet criteria for mixed episode, significant social/occupational impairment, exclude substance or GMC, not
bereavement.
Counseling: Make empathetic statements, e.g. This must be very hard for you.
Health while pregnant: recommend abstinence from harmful agents (alcohol, smoking, drugs) while pregnant and use of
medications only after consulting with a physician, treatment for pregnancy-related illnesses as above, and healthy eating
habits.
Social supports: Discuss the importance of engaging social supports, and consider a visit with both the patient and her partner
or other supporting person.
Abortion: Provide information on local abortion services. Make the patient aware that the gestational age limit after which
many practitioners will not perform an elective abortion in Canada is 20 weeks, but that this is a late limit and her decision
should be made sooner, before 16 weeks would be best. Inform the patient that further advice is available from private
gynecologists who perform abortions and counselors at elective abortion centers. Offer to refer the patient if she wishes.
Depression management: Normalize the patients depressed mood in view of her circumstances. If there is evidence of major
clinical depression, arrange close follow up to monitor for suicidal ideation, refer to psychiatry. Do not prescribe medications
at this time (because of the pregnancy).
2. 20 year old female wants an oral contraceptive. Take a history and counsel.
History: Name, age occupation/school level. Why does patient want an OCP? Has she been on it before or other forms of
contraception? If so, why was it stopped? How long has the patient been sexually active? How many partners? Current
contraception used. Is there a possibility that the patient could be pregnant? Obtain the date of last menstrual period.
Pregnancy history: GTPAL (number of gestations, term pregnancies, premature births, abortions, live children), history of
problems, if any, with previous pregnancies.
Gynecological history: Ask about sexually transmitted disease (STDs), PID, migraine, fibroids, diabetes, thromboembolic
disease, heart problems, cancer, liver disease. Date of last Pap smear, history of abnormal Pap smear and follow-up/treatment?
When did the patient start menstruating? Menstrual history: regularity and length of cycle and duration of periods, heaviness of
flow (number of pads required), cramping, associated discomfort/pain, bloating, mood swings (PMS). Medications, drugs,
alcohol, smoking, past medical history (especially breast cancer), family history, review of systems.
Counseling:
Contraindications to OCP: current pregnancy, undiagnosed vaginal bleeding, active cardiovascular/thromboembolic diseases
(includes coronary and carotid disease, symptomatic mitral valve prolapse, cerebrovascular disease, moderate-severe HTN,
active DVT), proliferative retinopathy, history of breast cancer or other estrogen dependent tumors (liver, breast, uterus),
impaired liver function (obstructive jaundice in pregnancy), congenital hyperlipidemia, age > 35 and smoking, Wilsons
disease. Relative contraindications to OCP: smoker > 35 years old, diabetes, migraines, fibroids.
Mechanism of action of OCP: standard preparations contains estrogen and progesterone or just progesterone, prevents
ovulation by interfering with feedback of hormone signaling, atrophic endometrium, change in cervical mucous (mucous
plugthought to be due to progesterone component).
Available preparations: 21 day vs. 28 day tablets (7 placebo days). Other preparations: Depo injections q3m (Depo-Provera
medroxyprogesterone, restoration of fertility may take up to 1-2 years, irregular menstrual bleeding), implants q5y (Norplant
levonorgestrel, six capsules inserted subdermally in arm, irregular menstrual bleeding). Longer term preparations offer lower
cost over the duration of action (but greater one-time cost) and greater convenience.
Benefits of all the hormonal contraceptives: ABCDEs: Anemia reduced, often clears Acne; Benign breast disease and cysts
decreased; Cancer (ovarian decreased), Cycles regulated, Increased Cervical mucous which reduces STDs; Dysmenorrhea

decreased, decreases Ectopic pregnancy rates and of course: virtually no chance of pregnancy when taken as directed (9899.5%).
Risks of hormonal contraceptives: slight weight gain is usual (5 lbs), increases risk of DVT especially in combination with
smoking, may stimulate estrogen-receptor positive breast cancers, but does not appear to cause them, may have to try two or
three different preparations to arrive at the one for the patient. Also note that hormonal contraceptives do not provide as much
protection against sexually transmitted diseases, compared to barrier methods.
Directions: Start OCP on the first day of the next menstrual period. Place package in an obvious location to help you to
remember. Take at the same approximate time each day. Use additional contraception for the first two months, as OCP
contraception is not reliable until then. If you miss a day, take two pills the next. If you miss two days, take two pills for the
next two days and use an alternative method until the next period. Give prescription for OCP of choiceany family members
(sisters/mother) on OCP? What works for them? Arrange follow up.
3. 16 year old boy with epilepsy documented by neurologist, comes to you because he does not want to see his parents
family doctor. Wants a drivers license. Take a history and counsel.
History of seizure disorder: Patient ID. Age of onset (primary generalized rarely begin < 3 or > 20 years old). Precipitants:
Sleep deprivation, drugs, EtOH, TV screen, strobe, emotional upset. Describe seizures (Jacksonian march? Salivation,
cyanosis, tongue biting, incontinence, automatisms, motor vs. visual/gustatory/olfactory), frequency, duration, what body parts
affected and in what order (motor frontal lobe, visual/olfactory/gustatory hallucinations = temporal lobe), promontory signs
(presence of aura: implies focal attack), post-ictal state (decrease in level of consciousness, headache, sensory phenomena,
tongue soreness, limb pains, Todds paralysis - hemiplegia), degree of control achieved with medications, at what dose and for
how long, corroboration from family if possible. Was a CT scan done when seizures were first diagnosed? Number and
description of recent seizures, are they different from previous seizures? Is the patient having any new symptoms such as
headache, morning vomiting, new neurological deficits. If the drug worked in the past why does the patient believe it isnt
working now? Side effects of antiepileptics: drowsiness, poor concentration, poor performance in school, ataxias, peripheral
neuropathy, acne, nystagmus, dysarthria, hypertrichosis (excessive hairiness), gingival hypertrophy (phenytoin). Medications,
drugs and alcohol, smoking, allergies, past medical history, family history, review of systems.
Compliance: Is the patient taking meds? Why not? Problems at school or home? Ask about relationship problems. Depression
screen as in #19 above. Social supports.
Physical exam: neurologic exam including mini mental, cranial nerves, bulk, tone, power, sensation, cerebellar exam, deep
tendon reflexes.
Treatment: Discuss importance of compliance with medication and avoiding dangerous activities such as driving until good
control is achieved. Ministry of Transportation regulations require 1 year seizure free before they will grant a drivers license
in Canada. Inform the MOT of the patients seizure disorder if you have not already done so and inform the patient that this is
required by law. If alcohol is an issue, inform the patient that chronic alcohol intake may decrease blood levels of antiepileptics
(via increased liver metabolism), and excess alcohol intake can precipitate seizures by lowering the seizure threshold thereby
precipitating a seizure. It is generally recommended that the patient not drink at all. Fatigue and concomitant illness can also
lower seizure threshold. The patient should consult a physician before taking other medications, as they may also lower the
seizure threshold. The same is also true of sedatives, cocaine, amphetamines and insulin. Fatigue and other illnesses can also
lower seizure threshold, in addition to various other medications. If patient is having stress management, anxiety issues, he
may require further counseling. Outline a treatment plan consisting of: EEG, CT head, metabolic screen, medications (if not
done already), and follow up appointments. Get the parents involved if possible.
Send blood for serum Dilantin (phenytoin) levels if patient is on this already. If Dilantin levels are therapeutic, but the patient is
having severe side effects or poor seizure control, a second drug may be added (usually carbamazepine or valproic acid).
Discuss what to do in the event of seizure, counsel parents if possible. Bystanders are not to insert objects into the patients
mouth. Turn patient on his side while seizing. Call ambulance or take to Emergency if seizure doesnt stop in 5 minutes.
Arrange regular follow up to monitor progress and serum Dilantin levels.

Indications and important side-effects of major antiepileptic drugs


Drug
Indication
Dose-related
Carbamazepine Partial or generalized Diplopia, dizziness, headache, nausea,
(Tegretol)
tonic-clonic seizures
drowsiness, neutropenia, hyponatremia
Phenytoin
(Dilantin)

Partial or generalized
tonic-clonic seizures,
status epilepticus

Valproate
(Epival,
Dapakene)
Ethosuximide
(Zarontin)

All generalized
seizures or partial
seizures
Absence seizures

Nystagmus, ataxia, nausea, vomiting,


gingival hyperplasia, depression,
drowsiness, paradoxical increase in
seizures, megaloblastic anemia
Tremor, weight gain, dyspepsia,
nausea, vomiting, alopecia, peripheral
edema
Nausea, anorexia, vomiting, agitation,
drowsiness, headache, lethargy

Idiosyncratic
Morbilliform rash, agranulocytosis,
aplastic anemia, hepatotoxic effects,
Stevens-Johnson, teratogenicity
Acne, coarse facies, hirsutism, blood
dyscrasias, Lupus-like syndrome,
rash, Stevens-Johnson, Dupuytrens,
hepatotoxic effects, teratogenicity
Acute pancreatitis, hepatotoxic
effects, thrombocytopenia,
encephalopathy, teratogenicity
Rash, erythema multiforme, StevensJohnson, Lupus-like syndrome,
agranulocytosis, aplastic anemia

4. 48 year old woman complains of diseased stomach. Has had negative investigations by several other doctors. Take
a history and perform a mental status examination. Q: Without looking at the patient again, describe her appearance.
What is your diagnosis?
History for depression: ID (name, age, employment status, marital status, living arrangements), chief complaint, HPI:
MSIGECAPS mood (depressed), anxiety, diurnal variation in mood and activity with nadir in the early morning, irritability,
change in sleep pattern (increased or decreased, if decreased usually morning awakening), interest (decreased), anhedonia
(inability to enjoy previously enjoyable activities), guilt, hopelessness, worthlessness, energy (fatigue), concentration, memory,
difficulty making decisions, appetite and weight gain or loss, psychomotor retardation/agitation, suicidal ideation. Also inquire
about past psychiatric/medical/surgical history, allergies, meds, family history, personal history, review of systems.
Suicidal ideation: does patient intend to harm self, reason for suicidal thoughts, current plan, lethality of plan, access to lethal
means, has patient given away prize possessions or written final notes to loved ones, previous attempts.
Diagnosis of major depression: Mnemonic for major depression MSIGECAPS: mood, sleep, interest, guilt, energy,
concentration, appetite, psychomotor, suicide, positive diagnosis of major depression requires five of these over a 2 week
period. One of the five must be either loss of interest or depressed mood. Cannot be a mixed episode and symptoms must
cause social/occupational impairment. Cannot be GMC or substance abuse or bereavement. (A diagnosis of depression cannot
be made in the face of bereavement within the past two months or drug or alcohol abuse.)
Dysthymia: A diagnosis of dysthymia requires depressed mood for most of the day, more days than not, for at least two years.
Presence while depressed of at least 2 of: poor appetite or overeating, insomnia or hypersomnia, low energy/fatigue, low self
esteem, poor concentration or difficulty making decisions, hopelessness. Never without depressed mood for more than 2
months at a time, no MDE, manic, mixed or hypomanic episodes in past, not GMC or substance abuse.
Manic episode: Expansive, elevated or irritable mood x 1 week with 3 of following: GSTPAID grandiosity (or inflated self
esteem), sleep (less need for), talkative, pleasurable activities (with painful consequences), activity increased (goal directed or
psychomotor), ideas (flight of), distractibility. Not mixed episode. Severe enough to cause psychotic features/impaired
social/occupational functioning. Not substance abuse or GMC.
Differential for depression: Check for bipolar mood disorder (manic-depressive), schizophrenia, psychotic depression and
obsessive-compulsive disorder. Ask about manic episodes, paranoia, hallucinations (esp. voices), obsessive thoughts, previous
psychiatric problems, family history of psychiatric disorders, substance abuse, relationship problems, problems at work
(basically a mental status exam).
Medical causes of depression: ask about hypothyroidism, adrenal dysfunction, hypercalcemia, mononucleosis. Consider
chronic fatigue syndrome. Drug use, smoking, allergies, past medical history including psychiatric history and history of
abuse. Family history, review of systems.
Mental status: appearance, behavior (dress, grooming, posture, gait, apparent age, physical health, body habitus, expressions,
attitude - cooperative?, psychomotor activity, attention, eye contact), speech (rate, rhythm/fluency, volume, tone, quantity,
spontaneity, articulation), mood (subjective emotional state in patients own words), affect (Quality euthymic, depressed,
elevated, anxious; Range full, restricted; Stability fixed, labile; Appropriateness; Intensity - flat, blunted), suicidal ideation

(low, intermediate, high poor correlation between clinical impression of suicide risk and probability of attempt), thought
process (coherent, flight of ideas, tangentiality, circumstantiality, thought blocking, neologisms, clanging, perseveration, word
salad, echolalia), thought content (delusions bizarre vs. non-bizarre, obsessions, preoccupations, phobias, recurrent themes),
perceptual disturbances (illusions, hallucinations, depersonalization, derealization), insight, cognition, judgment.
Multiaxial Summary:
Axis I clinical disorders DSM IV; differential diagnosis
Axis II personality disorders DSM IV; mental retardation
Axis III GMC (as they pertain to Axis I and other Axes)
Axis IV psychosocial and environmental problems
Axis V global assessment of functioning (GAF) 0-100
Mini-Mental Status: orientation to time/place (5 pts, year, season, month, day, day of week; 5 pts, country, province, city,
hospital, floor), memory (3 pts, honesty, tulip, black; 3 pts, delayed recall), attention/concentration (5 pts, serial 7s, WORLD
backwards), language tests: comprehension (3 pts, three point command), reading (1 pt, close your eyes), writing (1 pt,
complete sentence), repetition (1 pt, no ifs, ands or buts), naming (2 pts, watch, pen), spatial ability (1 pt, intersecting
pentagons)
Patients appearance: slovenly
Diagnosis: major depression
Treatment of major depression: pharmacotherapy, psychotherapy, family therapy. Start fluoxetine (prozac) 20 mg qAM, may
increase to 40 mg qAM after 1 week. Takes 2-4 weeks to work. Explain side effects of sleep disturbance, anorgasmia, nausea
(use SSRI summary below).
Admit: when patient is actively suicidal, i.e. plans are specific, patient has given away possessions, written final notes to loved
ones. For suicidal ideation without current intent, discharge only with written contract with the patient to inform you
immediately if they feel likely to make an attempt. Give number to call and arrange regular follow up before patient leaves the
office.
SSRI antidepressant summary
Fluoxetine (Prozac) generally energizing
Fluvoxamine (Luvox) more sedating
Sertroline (Zoloft) used in the elderly
Paroxetine (Paxil) used in mixed anxiety/depress

Start at: (mg OD)


20
50
50
10-20

Therapeutic: (mg OD)


20-80
150-300
50-150
20-60

5. 60 year old woman with acute confusion. Perform a focused physical exam excluding mental status.
Neurologic exam: what follows is a practical, regionally organized neurologic exam which can be completed in less than eight
minutes (with practice). It begins with the patient sitting, then standing, then lying down. Note that every physical exam
should include vitals, although in this case the examiner will ask you to move on.
Patient sitting (shoes and socks removed):
GCS/MMSE only if patient poorly responsive. The examiner will remind you to omit the mini mental status exam.
Glasgow Coma Scale
Eye Opening
(E)
Spontaneous
4
To speech
3
To Pain
2
never
1

Verbal Response
Oriented and converses
Confused conversation
Inappropriate words
Incomprehensible sounds
None

(V)
5
4
3
2
1

Best Motor Response


Obeys commands
Localizes pain
Withdrawal to pain
Abnormal flexion (decorticate)
Abnormal extension (decerebrate)
Nil

(M)
6
5
4
3
2
1

Mini-Mental Status: orientation to time/place (5 pts, year, season, month, day, day of week; 5 pts, country, province, city,
hospital, floor), memory (3 pts, honesty, tulip, black; 3 pts, delayed recall), attention/concentration (5 pts, serial 7s, WORLD

backwards), language tests: comprehension (3 pts, three point command), reading (1 pt, close your eyes), writing (1 pt,
complete sentence), repetition (1 pt, no ifs, ands or buts), naming (2 pts, watch, pen), spatial ability (1 pt, intersecting
pentagons)
Cranial nerves CN III, IV, VI: Extra-ocular movements (patient follows your finger or the handle of a reflex hammer in an
H-pattern, check for diplopia in the center and at the extremes of the visual fields. CN II: Visual fields by confrontation (one
eye at a time) patient holds own hand over one eye and counts fingers flashed in left and right fields simultaneously (upper and
lower) or identifies the wiggling finger as it enters each quadrant on confrontation if too confused to count. By acuity: Snellen
card. Fundi check for papilledema using ophthalmoscope. CN II, III, V1 & VII: Pupillary light reflex and accommodation,
corneal reflex. CN V: Facial sensation to light touch in the ophthalmic, maxillary and mandibular divisions of the trigeminal
nerve. Trigeminal motor: clench teeth, lateral jaw movement. CN VII, XII: Facial muscle power raise eyebrows, squint eyes
closed, show teeth, protrude tongue, observe palatal movement on saying Ah, puff cheeks, platysma. CN VII, IX, X: Gag
reflex, observe symmetric movement of palate, swallowing, taste. CN VIII: Gross hearing rub thumb and index lightly while
approaching the patients ear, note when they begin to hear the sound. CN XI: Sternocleidomastoid power and trapezius power.
Body Pronator drift arm straight out in front of patient with palms up, eyes closed. Look for curling of fingers from the
ulnar side, pronation and downward movement of the affected side. Hoffmans reflex with the patients relaxed hand in a
palm-down position, squeeze and flick the nail of the index or long fingers between your thumb and long finger. Thumb
flexion indicates a positive test and denotes upper motor neuron lesion (similar to Babinskis reflex). Cerebellar tests fingernose rapid alternating movements (dysdiadocokinesis), heal-shin. Body power deltoids, biceps, triceps, wrist extension and
flexion, finger abduction and adduction, psoas (hip flexion lift each knee off the bed against resistance), quadriceps,
hamstrings, ankle dorsiflexion (test plantar flexion while standing). Body sensation light touch, pin prick, and cold
temperature (use a meta K-basin or turning fork) are tested on the distal upper limbs (forearms and hands) and lower limbs
(foreleg and feet). Vibration sensation is tested using a C-120 Hz tuning fork on the joint capsules of the most distal joints at
which the vibration can be felt. Body reflexes biceps jerk, brachioradialis, knee, ankle, Babinski.
Patient standing
Gait observe for wide-base, Parkinsonian festination/shuttle, consistent lateralizing falls. Balance tight-rope walk, stand
with arms crossed and feet in line, Rhomberg (feet together, eyes closed). Plantar flexion power hold patients hands for
balance, ask patient to stand on one foot, then raise themselves up on the toes.
Patient lying supine
Tone passive rapid alternating forearm rotation, passive rapid elbow flexion/extension with one thumb on the biceps tendon
to feel for cog-wheeling. Rapid lifting of the relaxed leg from behind the knee heel remains on the bed in normal tone.
6. A young man is brought to the Emergency department with an epigastric stab wound sustained in a bar fight. There
is a nurse in the room. Manage.
Unstable means either an acutely changing condition, vital signs dangerously beyond normal ranges (e.g. low BP), or any lifethreatening condition which is inadequately controlled. This patient is unstable due to stab wound with unknown consequences
which is an acutely changing and inadequately controlled problem and therefore requires stabilization or resuscitation.
The ACLS and ATLS format is useful: i.e. primary survey, orders, secondary survey, orders. Note that the simulated patient
may have an impressive simulated injury. Do not allow this to distract you from the primary survey.
Primary survey: (mnemonic ABCD)
Airway: check for patent airway (look in mouth), is airway threatened by blood? High chance of aspiration due to poor level of
consciousness, neck or face swelling. If airway is compromised, immediately place an oral airway or intubate.
Breathing: is patient breathing, check O2 sat, may require immediate manual bag-valve mask followed by intubation with
positive pressure ventilation.
Circulation: BP, HR, rhythm on monitor, active high volume bleeding. Patient may require immediate chest compressions,
defibrillation or cardiac drugs if there is an unstable rhythm (Ventricular fibrillation, unstable ventricular tachycardia, PEA,
asystole, PST, or bradycardia).
Debilitation: refers to the Glasgow Coma Scale (GCS):

Glasgow Coma Scale


Eye Opening
(E)
Spontaneous
4
To speech
3
To Pain
2
never
1

Verbal Response
Oriented and converses
Confused conversation
Inappropriate words
Incomprehensible sounds
None

(V)
5
4
3
2
1

Best Motor Response


Obeys commands
Localizes pain
Withdrawal to pain
Abnormal flexion (decorticate)
Abnormal extension (decerebrate)
Nil

(M)
6
5
4
3
2
1

Note: Standard painful stimulus is rubbing the knuckle on the sternum. For withdrawal, apply pressure on the base of the nail
bed with a pen. Decorticate posture is arm flexion with leg extension on the same side of the body, may be unilateral or
bilateral. Indicates a lesion above the brainstem. Decerebrate posture is arm and ipsilateral leg extension, may be unilateral or
bilateral, indicates brainstem involvement. A GCS of 8 or less is considered an indication for intubation because of the risk of
poor protection of the airway from aspiration.
Primary orders: oxygen, monitoring (ECG, O2 sat, automatic BP cuff or arterial line), IV access: need two large-bore (16
gauge, 14 if possible femoral vein cortice with 2 lumens), run wide open with normal saline for acutely low BP, may need to
be more restrained if pulmonary edema is a problem. Coma cocktail if diagnosis not known already: thiamine 100 mg IV,
narcan 1 mg IV, flumazenil 0.1 mg IV (1 amp D50W is no longer included in this cocktail because of deleterious effects of high
serum glucose on the injured brain). Initial investigations: CBC, lytes, urea, creatinine, ABG, glucose, ionized Ca, CK-MB,
troponin, INR/PTT, ECG, portable CXR, cross-table lateral C-spine and hard collar if there is head injury or any significant
trauma.
Secondary survey: Head to toe physical exam. Vitals. Head and neck: inspect for lacerations and contusions, pupillary
response, dolls eyes (careful of neck, may not be able to turn head enough to do this), corneal reflexes, palpate facial bones for
stability, look in nose and ears for blood or CSF leaks, hemotympanum. Check oral cavity, gag reflex, palpate dorsal cervical
spines for pain and alignment, is the trachea midline? Chest: breath sounds, heart sounds, radial pulses bilaterally. Abdomen:
rigidity is an indicator for immediate general surgery, auscultate for bowel sounds, palpate liver and spleen. Log roll patient
onto back, inspect. Rectal. Palpate for pelvic stability and intactness of long bones.
Secondary orders: foley, NG tube if patient may go to surgery or require charcoal. Specific interventions based on findings.
Further X-rays, CT head if cause of decreased level of consciousness unclear or if there may have been a seizure.
Clearing C-spines: The principle of clearing C-spines is to rule out both bony fractures and ligamentous injury, either of which
can make the spine dangerously unstable. Most emergency physicians will clear the cervical spine in the case of an alert
patient who has no pain on palpation of the dorsal spinous processes and a normal cross-table lateral C-spine X-ray. If the
patient has neck pain, flexion/extension plane films are done. These involve gently flexing, then extending the neck and taking
views at each extreme. The patient must be alert enough to warn the examiner of paresthesias in the hands or increased neck
pain on movement during this procedure, which may indicate compromise of the neural elements. Flexion/extension views
may be done under fluoroscopy if the patient is not alert.
History: if available: How did the patient obtain the wound, was there mechanism of action for other injury, i.e. collapse/fall,
preceding and subsequent events, did patient lose consciousness, duration of unconsciousness, did the patient fall, injuries
during fall, medications and drugs, smoking, allergies, past medical history, family history, review of systems.
Management: For unstable blood pressure, blood loss is the most likely cause (CBC may be normal with a large acute blood
loss), type and cross for 4-6 units depending on estimated severity and hang blood as soon as possible. Is the patients
abdominal wound the first priority? Examine for head injury, other injuries. Explore wound with a gloved finger on secondary
survey, if wound is more than superficial, consult general surgery and prepare patient for immediate surgical exploration in the
OR. Obtain details of the history from family or witnesses if possible.
7. 30 year old woman with six weeks of epistaxis, petechiae and easy bruising. Perform a focused physical exam.
Findings: petechiae, bruises. Q: The patient has a normal CBC except for platelets 20 (normal 130-400). What is the
most likely diagnosis? What four areas on history would help to confirm this diagnosis? What four investigations
would you order?
Hematologic exam: patient disrobed to underwear, draped below the waist.

Patient sitting
Inspect the patient generally for petechiae, abnormal skin tone, hair falling out. Inspect the finger and toe nails for dystrophy,
flame hemorrhages, leukonychia, inspected the palm for erythema and Dupuytrens contracture. Look in the nose and mouth
for bleeding, petechiae, masses. Palpate the anterior and posterior triangles of the neck, the supra and infra clavicular areas,
and the axillae for lymph nodes. Palpate the thyroid while standing behind the patient, ask her to swallow. Chest from
behind the patient, inspect the skin. Percuss the lung fields for effusions and consolidations, auscultate the lung fields. Percuss
and auscultate the anterior lung fields. Listen over the aortic (right upper sternal border 2nd ICS systolic = stenosis, diastolic
= regurgitation, continuous = BT shunt, right or left), pulmonary (left upper sternal border 2nd ICS systolic = ASD, pulmonary
flow, continuous = ductus, LBT shunt), tricuspid (left lower sternal border 5th ICS systolic = regurgitation ... Stills murmur,
diastolic = stenosis) and mitral (apex 5th ICS and midclavicular line systolic = prolapse, regurgitation; diastolic = stenosis)
areas.
Patient lying supine
Compress the sternum and ribcage for pain (seen in multiple myeloma). Inspect the abdomen. Auscultate for bowel sounds.
Palpate for enlargement of the spleen and liver. Percuss the liver. Palpate the groin for lymph nodes. Note: avoid rectal exam
as this trauma may cause bleeding.
Most likely diagnosis: idiopathic thrombocytopenic purpura (ITP), also called immunologic thrombocytopenic purpura or
Wedhofs disease.
Four findings on history which would help to confirm the diagnosis: 1. Remitting-relapsing course, 2. Mild fevers, 3.
Splenic discomfort due to mild enlargement, 4. Bleeding after low doses of NSAID.
Four investigations: Blood smear, INR/PTT (for hemophilia), serum urea/creatinine (for hemolytic-uremic syndrome), serum
platelet-associated IgG (for ITP).
8. 62 year old man presents to the Emergency Department with 12 hours suprapubic discomfort and inability to
urinate. Catheterization yields 1200 cc urine. Take a history. Q: What is the most likely cause of this mans problem?
Give three other possible diagnoses. What four investigations would you order?
History: name, age, occupation. History of suprapubic pain and inability to urinate. History of pain on urination, frank blood
in the urine, color of urine, difficulty initiating or maintaining urinary stream, fever, renal pain, groin pain. Previous renal colic/
diagnosed prostate hypertrophy, prostate cancer, prostatism, nephrolithiasis, UTIs? Malignant symptoms: night sweats, weight
loss, fatigue. Medications, drugs/alcohol, smoking, past medical history, past surgical history, history of pelvic radiation,
TURP, family history, review of systems.
Most likely diagnosis: benign prostatic hyperplasia.
Other possible diagnoses: UTI, prostatitis, prostate cancer.
Four investigations: urea/creatinine, urinalysis, prostate specific antigen (PSA), renal ultrasound.
Treatment: watchful waiting (50% resolve spontaneously), medical (alpha-adrenergic antagonists Terazosin, doxazosin,
tamsulosin; 5-alpha-reductase inhibitors finasteride), surgery (TURP vs. open prostatectomy), minimally invasive (stents,
microwave therapy, laser ablation, cryotherapy, HIFU, TUNA).
9. 6 month old child who just had a seizure. Take a history from the mother in the Emergency Department. Findings:
short seizure with T 39.5 C. Never had seizures in the past. Developmentally normal. Q: What is the most likely
diagnosis? What would you tell the mother about any possible recurrence? What advice do you give if the child has
another seizure?
History: Name, age. Describe seizure duration, what body parts affected and in what order, premonitory signs, post-ictal state
(decrease in level of consciousness, headache, sensory phenomena). Previous seizure? Ask about preceding trauma or illness
or medications taken, the childs temperature at the time of the seizure. Meningitis signs, neurological. History of problems
during the pregnancy and birth. Developmental history. Childs medical history, surgical history, medications, allergies.

Selected Developmental Milestones


Speech
6 months
12 months
24 months
2-3 years
Gross motor
6 months
9 months
12 months
15 months
Fine motor
12 months
24 months
Social
6 months
9 months
2 years
5 years

initiates sounds, eye contact


2 words beyond mama and dada
2-3 word phrases
short sentences
roll over
stand
cruise
walk
pincer grasp
turns pages in a book
stranger anxiety
separation anxiety
says no
prints name

Most likely diagnosis: febrile seizure (febrile seizures usually 6 months to 6 years, associated with initial rapid rise in
temperature, no neurologic abnormalities/evidence of CNS infection/inflammation before or after, no history of non-febrile
seizures, most common generalized tonic-clonic, < 15 minutes duration, no recurrence in 24 hours, atypical may show focal
origin/> 15 minutes/> 1/24 hours/transient neurologic defect).
Prognosis: after a single febrile seizure 65% will never have another seizure. 30% will have further febrile seizures, 3% will
go on to have seizures without fever and 2% will develop lifelong epilepsy.
Management: find source of fever, Tylenol (antipyretics), LP to rule out meningitis if signs of meningitis, counseling and
reassurance to patient and parents if febrile seizures.
Treatment of recurrence: control fever with antipyretics (Tylenol), tepid bath, fluids for comfort only and use Ativan
(lorazepam) 1 mg SL/PO (or diazepam 5-10 mg PR) if a seizure occurs at home. Turn patient onto his/her side, do not force
objects or fingers into mouth. Bring to ER if seizure does not stop within fifteen minutes. Seizures do not cause mental
impairment unless they are prolonged (> 30 min), although seizures can be a symptom of brain damage. Patient should be
investigated with CT head and EEG. Prophylactic anticonvulsant therapy is a consideration with repeated seizures.
10. 68 year old man with difficulty swallowing. Take a history. Findings: throws up after eating. Can swallow liquids
only. Weight loss and fatigue. Smoker. Q: X-ray of barium swallow showing narrowing of contrast at T5-6: describe
the abnormality. What is the likely diagnosis? What investigation would confirm the diagnosis? What further
investigations would you order?
History: (Dysphagia = difficulty swallowing) onset, chronology, description of problem, aggravating and relieving factors. Is
the difficulty transferring food from mouth to esophagus suggesting oropharyngeal dysphagia with food getting stuck
immediately after swallowing often with nasal regurgitation? With esophageal dysphagia food seems to be stuck further down.
Is the problem worse with solids (suggests mechanical obstruction), or liquids (suggests neuromuscular dysfunction, often cant
swallow either solids or liquids). Is there a sensation of a lump in the throat? (globus hystericus = transitory sensation of a
lump in the throat related to anxiety). Progression from solid swallowing difficulty to difficulty swallowing both solids and
liquids suggests progressive obstruction such as from a worsening stricture or growing tumor. The combination of intermittent
obstruction and chest pain suggests esophageal spasm. Ask about peptic ulcer, reflux, hiatus hernia, weight loss, night sweats,
fatigue, hematemesis, black stools, pain. Medications, drugs/alcohol, allergies, smoking, past medical history, family history,
review of systems.
Oropharyngeal: Neurological cortical pseudobulbar palsy (UMN lesion) due to bilateral stroke; bulbar ischemia
(stroke); syringobulbia; tumor (LMN); peripheral polio; ALS. Muscular MD; cricopharyngeal incoordination (failure of
UES to relax with swallowing), sometimes seen with GERD; Zenkers diverticulum.
Esophageal: solid food only mechanical obstruction intermittent = lower esophageal ring/web; progressive heartburn
= peptic ulcer; age > 50 = carcinoma. Solid or liquid food neuromuscular disorder intermittent = diffuse esophageal
spasm; progressive reflux = scleroderma; respiratory symptoms = achalasia.

Differential diagnosis: Mechanical obstruction: tumor, stricture (secondary to GERD, trauma etc.). Neuromuscular
obstruction: achalasia (cardiac sphincter does not relax), cranial nerve palsy, MS, supranuclear palsy, stroke, motor neuron
disease, myasthenia gravis, muscular dystrophy.
Description of Barium swallow findings: string sign, graded narrowing of intra-esophageal diameter extending from T5 to T8
level.
Most likely diagnosis: esophageal cancer.
Investigation to confirm diagnosis: endoscopy with biopsy.
Further investigations: CT chest (for mediastinal and lymph node involvement), chest X-ray, liver function tests, abdominal
ultrasound (for mets).
11. 23 year old with BP 160/100 in both arms. Perform a focused physical exam. Q: Give four possible diagnoses.
What four investigations would you order? If these investigations were negative, give 5 steps in your initial
management plan.
Physical exam for hypertension: combines exams for atherosclerosis, coarctation, hyperthyroidism and Cushings. Patient
should be disrobed to underwear and draped below the waist.
Patient Sitting
Take vitals (need BP in all four limbs legs to be done when patient is lying down). Inspect for cyanosis, arcus senilis in the
eyes (sign of high cholesterol), bulging veins in the upper chest (SVC syndrome), supraclavicular fat pad, buffalo hump, moon
face, truncal obesity, striae, nicotine stains on fingers, clubbing, flame hemorrhages on nails, obesity, high work of breathing,
intercostal indrawing, symmetric chest movement, visible apex beat.
Fundoscopy for retinopathy of hypertension: (in order of increasing severity of damage) constriction and sclerosis of retinal
arterioles, hemorrhages, exudates, papilledema.
Thyroid exam: Inspect patient for proptosis, thyroid stare (upper lids do not overlap the irises). Have patient follow your
finger up and down to check for lid lag and globe lag. Is skin thin dry and flaky or diaphoretic. Palpate thyroid standing
behind patient, ask patient to swallow. Inspect nails for leukonychia and hands for tremor (can place a piece of paper on the
hand held horizontal to detect fine tremor) and clubbing/thickening of distal phalanges. Check biceps reflexes with thumb held
over the tendon, feel for slow return phase reflex of hypothyroidism.
Palpate the apex, note whether it is laterally displaced (lateral to the mid-clavicular line) and feel for thrill or heave, feel radial
pulses in the arms simultaneously, note any delay.
Percuss the lung fields anteriorly and posteriorly.
Auscultate the lung fields anteriorly and posteriorly, listen over the aortic (right upper sternal border 2 nd interspace systolic =
stenosis, diastolic = regurgitation, continuous = BT shunt, right or left), pulmonary (left upper sternal border 2 nd interspace
systolic = ASD, pulmonary flow, continuous = ductus, LBT shunt), tricuspid (left lower sternal border 5th interspace systolic
= regurgitation ... Stills murmur, diastolic = stenosis) and mitral (apex at 5th interspace and midclavicular line systolic =
prolapse, regurgitation; diastolic = stenosis) areas, as well as over the right clavicle, and both carotids. Listen for rub. To
bring out an aortic murmur (typically aortic regurgitation), and coarctation bruits, ask patient to lean forward, exhale and stop
breathing while you listen over the aortic and pulmonic areas.
Patient Lying Supine
Auscultate for bruits over the renals on the abdomen. Observe for pulsations due to abdominal aortic aneurysm, palpate
abdomen for hepatomegaly. Palpate femoral pulses, and auscultate for femoral bruits, palpate the popliteal pulses, inspect the
legs and feet for venous stasis or arterial insufficiency ulcers, palpate the dorsalis pedis and tibialis posterior pedal pulses. Feel
the ankles for pitting edema.
Tibial BP: BP cuff placed around calf, auscultate the tibialis posterior pulse posterior to medial malleolus on right and left.

10

JVP: Raise the head of the bed 30 degrees and inspect the neck. A jugular venous pulsation higher than 4-5 cm above the
sternal notch (approximately the level of the right atrium) is abnormal. If no JVP is visible then you may have to raise or lower
the bed for high or low JVPs respectively. Check the hepatojugular reflux (supine, mouth open, breathing normally
compress the liver, the jugular venous pulse should either not rise or remain elevated only transiently, a sustained elevation > 1
cm in the JVP is pathological). This is used to assess high jugular venous pressure and RV function.
Classification of BP
DBP: < 90 (normal), 90-104 (mild HTN), 105-114 (moderate HTN), > 115 (severe HTN)
SBP: < 140 (normal), 140-159 (borderline isolated systolic HTN), > 160 (isolated systolic HTN)
Four possible diagnoses: essential hypertension, renal HTN (renal artery insufficiency, renal parenchymal disease),
endocrine (thyroid hormone, OCP, primary hyperaldosteronism, hyperparathyroidism, pheochromocytoma, Cushings)
coarctation of the aorta, others (enzymatic defects, neurological disorders, drug-induced (prolonged corticosteroid use),
hypercalcemia, watch for labile white-coat HTN).
Investigations: Repeat BP 3x over next 6 months, urinalysis, CBC, Cr, lytes, fasting serum glucose, cholesterol panel, 12 lead
ECG, TSH, renal U/S.
Initial Management: 1. Smoking cessation. 2. Alcohol restriction to low risk drinking guidelines. 3. Salt restriction (max. 90130 mmol 3-7 g per day). 4. Saturated fat intake reduction. 5. Weight reduction if BMI > 25 (at least 4.5 kg). 6. Regular
aerobic exercise (50-60 min, 3-4x per week).
12. 21 year old female with bloody diarrhea. Take a history. Findings: Abdominal cramping. Six watery stools in the
past four hours containing maroon colored blood. Feels dizzy and weak. No previous history of diarrhea previously
well. Q: What two findings on history indicate the seriousness of the problem? Give three possible diagnoses. Give four
investigations appropriate to this situation.
History: name, age, occupation. Onset, duration, frequency, of diarrhea. Appearance of stools: how well formed, is blood on
(anal/rectal laceration) or admixed with stools, is blood bright red (lower tract bleed) or dark brown-black (upper tract bleed,
e.g. stomach). Pain with bowel movements, abdominal pain or cramps with location, radiation, precipitating factors and
alleviating factors, quality, severity, timing with respect to defecation, gas bloating. Heart burn, peptic ulcer, reflux, hiatus
hernia. Extra-intestinal manifestations of inflammatory bowel disease: ask about iritis, arthritis, mouth ulcers, anal ulcers, skin
lesions, kidney stones. Infectious diarrhea: inquire about fever, nausea, vomiting, weight loss, fatigue. Recent travel,
consumption of unusual foods or foods which may have been contaminated. Recent exposure to antibiotics. Family members
sick at home. Pelvic pain, vaginal discharge, vaginal bleeding. Past medical history, medications (especially NSAIDs,
laxatives, antibiotics), family history of Crohns, ulcerative colitis, familial polyposis, review of systems.
Two findings which indicate the seriousness of the problem: patient feels dizzy and weak.
Three possible diagnoses: gastroenteritis, bleeding peptic ulcer, inflammatory bowel disease.
Four investigations: CBC with differential, stool for ova & parasites with culture & sensitivities, Clostridium difficile toxin.
Endoscopy (but above first). Type and cross for 4 units PRBCs.

1996
13. Middle aged woman with systolic ejection murmur radiating into the carotids. Perform physical exam.
The physical exam for a patient with a heart murmur is a cardiopulmonary exam.
Patient in sitting position: take vitals
Inspect for surgical scars, trauma, bony abnormalities, cyanosis, arcus senilis in the eyes (sign of high cholesterol), bulging
veins in the upper chest (SVC syndrome), nicotine stains on fingers, clubbing, flame hemorrhage on nails, obesity, work of
breathing, intercostal indrawing, symmetric chest movement, visible apex beat.
Palpate the apex, note whether it is laterally displaced (lateral to the mid-clavicular line) and feel for thrill or heave, feel radial
pluses bilaterally.

11

Percuss the lung fields anteriorly and posteriorly.


Auscultate the lung fields anteriorly and posteriorly, always evaluate heart sounds before murmur. First listen for S1 and S2,
then look for S3 and S4 and any other unusual heart sounds. Listen over the aortic (right upper sternal border 2nd ICS systolic
= stenosis, diastolic = regurgitation, continuous = BT shunt, right or left), pulmonary (left upper sternal border 2nd ICS
systolic = ASD, pulmonary flow, continuous = ductus, LBT shunt), tricuspid (left lower sternal border 5th ICS systolic =
regurgitation ... Stills murmur, diastolic = stenosis) and mitral (apex 5th ICS and midclavicular line systolic = prolapse,
regurgitation; diastolic = stenosis) areas as well as over the right clavicle, and both carotids. Listen for rub. To bring out an
aortic murmur (typically aortic regurgitation), ask patient to lean forward, exhale and stop breathing while you listen over the
aortic area. To bring out a mitral murmur, ask patient to lie supine and roll partly onto the left side while you listen over the
apex. In general, murmurs are accentuated by increasing the dynamicity of the heart with mild exercise, such as asking the
patient to walk up a flight of stairs.
Murmurs are described in terms of where they are heard loudest, where the sound radiates, whether it occurs in systole or
diastole, the pitch (e.g. high, low), quality (e.g. harsh, blowing, musical), contour (e.g. crescendo, decrescendo or plateau) and
its loudness graded out of six (e.g. II/VI). The murmur of aortic stenosis is loudest over the aortic area, radiates to the clavicle
or carotids, occurs in systole, has medium or high pitch, is harsh and crescendo-decrescendo. A mitral regurgitation murmur by
contrast, is loudest over the apex, also occurs in systole, radiates to the axilla, is medium to high in pitch, blowing and plateau.
Innocent murmurs are <3/6 in intensity, peak early in systole, stop long before S2, are heard best at the base of the heart (aortic
and pulmonary areas), are not associated with clicks or heaves, and ECG and CXR are normal.
Patient lying supine
Auscultate for bruits over the renals on the abdomen. Observe for pulsations due to abdominal aortic aneurysm, palpate
abdomen, femoral pulses, and auscultate for femoral bruits, palpate the popliteal pulses, inspect the legs and feet for venous
stasis or arterial insufficiency ulcers, palpate the dorsalis pedis and tibialis posterior pedal pulses. Feel the ankles for pitting
edema.
JVP: Raise the head of the bed 30 degrees and inspect the neck. A jugular venous pulsation higher than 4-5 cm above the
sternal notch (approximately the level of the right atrium) is abnormal. If no JVP is visible then you may have to raise or lower
the bed for high or low JVPs respectively. Check the hepatojugular reflux (supine, mouth open, breathing normally
compress the liver, the jugular venous pulse should either not rise or remain elevated only transiently, a sustained elevation > 1
cm in the JVP is pathological). This is used to assess high jugular venous pressure and RV function.
14. 50 year old man with left-sided chest pain. Manage (means history, physical, investigations and treatment).
Findings: bruise on chest wall, normal CXR and ECG.
Differential Diagnosis for Chest Pain
Cardiac
Angina
Pulmonary
MI
Pneumonia w/ pleuritis
Pericarditis
Pneumothorax
Myocarditis
PE
Dissecting aorta
Pulmonary hypertension

Non-Cardiac
GI
Esophageal reflux
Ulcer

MSK/Neuro
Arthritis
Chondritis
Rib fractures
Herpes Zoster

Psychologic
Anxiety
Panic

History for chest pain: describe the pain, location, radiation, quality, time of onset, duration, intensity, circumstances under
which it occurs, aggravating and relieving factors, associated symptoms such as nausea, shortness of breath, dizziness,
diaphoresis, dependent edema. Leg pain. Respiratory symptoms: cough, sputum, fever, hemoptysis. GI symptoms: heartburn,
dysphagia. Previous episodes, chronology of these. History of trauma, asthma, bronchitis, COPD, pneumothorax, recent viral
illness and previous chicken pox (Herpes Zoster can cause chest pain), gastritis, peptic ulcer, reflux. Risk factors for heart and
lung disease: smoking, hypertension, hyperlipidemia. Past medical history, especially diabetes, heart disease including
pericarditis, lung disease, GI problems, surgical history, and family history. Medications, drug use, smoking, allergies, review
of systems.
Physical exam: Cardiopulmonary exam as in question #13.
Investigations: CXR, ECG.

12

Treatment: given a normal CXR and ECG with a chest wall bruise as evidence of trauma send patient home, recommend nonprescription pain medication (Tylenol and/or ibuprofen) and advise that the pain should subside gradually. Since the patient is
at risk because of his age group and male gender, explain the symptoms of myocardial infarct (MI) and advise to return
immediately if these occur.
15. Young man with recent onset back pain and limp. Take history and physical.
A differential for low back pain is:
1. Degenerative (90% of all back pain)
Mechanical (degenerative, facet joint pain, muscle strain/spasm)
Spinal stenosis (congenital, osteophyte, central disc)
Peripheral nerve compression (disc herniation or rupture)
2. Cauda Equina syndrome
3. Neoplastic: primary or metastatic
4. Trauma: fracture (compression, distraction, translation, rotation)
5. Spondyloarthropathies: e.g. ankylosing spondylitis
6. Discitis/osteomyelitis
7. Referred: aorta (abdominal aortic aneurysm), renal (pyelonephritis), ureter (nephrolithiasis), pancreas (pancreatitis)
8. Malingering
Because discogenic and stenotic radiculopathy which have not improved over at least 4 weeks may be treatable surgically, the
priority of a history and physical for back pain is to differentiate radiculopathy from other causes and to identify the nerve root.
The most common disk herniation is a posterolateral L4-5, which compresses the L5 root. The herniation will also compress
the L4 root if the herniation is far lateral and the S1 root if it is more medial (central). The second most common herniation is a
posterolateral L5-S1, which compresses the S1 root. In the thoracic and lumbar spine, the nerve roots exit below the pedicles
of the vertebra of the same number, while in the neck the nerve root exits above the pedicle of the vertebra of the same number.
L5 compression produces radiation from buttock to lateral calf, lateral calf pain, numbness of the medial dorsum of foot
(including web of great toe), and ankle dorsiflexion weakness, S1 compression produces radiation posteriorly down leg to heel,
posterior calf pain, lateral foot numbness and ankle plantar flexion weakness (with decreased ankle jerk).
History: Red flags (BACKPAIN) B: bowel or bladder dysfunction; A: anesthesia (saddle); C: constitutional
symptoms/malignancy; K: chronic disease; P: paresthesias; A: age > 50; I: IV drug user; N: neuromotor deficits.
Describe the pain, location, radiation (L5 radiculopathy causes radiation from buttock to lateral calf, S1 radiates posteriorly
down leg to heel), quality, duration, frequency, intensity, circumstances under which it occurs, aggravating and relieving
factors. Onset and chronology, previous episodes. Previous investigations, treatment.
Pain worse lying down and bilateral leg weakness suggests spinal stenosis or ankylosing spondylitis. Spinal stenosis is
characterized by worsening of symptoms with standing and walking, with relief on bending and setting (a typical history of
leaning on and bending over the shopping cart for relief of pain while shopping is suggestive of spinal stenosis). Ankylosing
spondylitis is characterized by morning stiffness relieved by activity. Pain worse in back than in buttock or leg suggests
mechanical back pain. Pain worse in buttock or leg than in back suggests radiculopathy. Predominating symptoms of stiffness
are suggestive of ankylosing spondylitis. Back pain is recurring and tends to be nocturnal. Morning stiffness improves over
the day. May be associated with weight loss, fever, fatigue, anemia. Focus on joint symptoms (typically large joints), uveitis
(occurs in one third of cases), and family history.
Has the patient had a fever, weight loss, night sweats (signs of cancer), urinary tract infection (sign of urinary retention), joint
pain, uveitis (inflammation of the uveal tract: iris, ciliary body, and choroids sign of ankylosing spondylitis)? Ask about
effect on activities of daily living, functional limitations. Associated numbness, weakness. Are the symptoms improving or
worsening? What are the patients conclusions about the pain and expectations of the physician? Medications, drugs and
alcohol, smoking, past medical history, family history, review of systems.
Cauda equina syndrome: Inquiry into bowel, bladder, and sexual function to reveal this rare syndrome is obligatory and a
source of frequent false alarms. Because these functions may not recover once lost, cauda equina syndrome due to a surgically
treatable lesion is a surgical priority if the time course is subacute and an emergency if the loss of function is acute. The
syndrome consists of saddle anesthesia (perineal numbness), lax anus, impotence, urinary retention and bowel incontinence.
Note that this combination of signs is due to preservation of sympathetic tone with loss of parasympathetic tone. Sympathetic

13

tone is preserved because it is carried extra-spinally, while parasympathetic signals are carried via the inferior spine and nerve
roots. Note that bowel contraction and penile erection are parasympathetically driven.
Physical exam:
Standing
Assess gait, posture, range of motion including rotation, lateral and forward flexion, extension (pain worse on forward flexion
and relief on extension suggest discogenic pain, pain worse on extension suggests facet joint pain). For ankylosing spondylitis:
Wright-Schober test positive when distance between the lumbrosacral junction and a point 10 cm above (identified by
palpation on the erect spine), distract by less than 5 cm on full forward flexion of the spine. Modified Schober (i.e. detection of
decreased forward flexion of lumbar spine). Lateral flexion is impaired when the hand moves downward by less than 3 cm on
the ipsilateral thigh. Look for scoliosis on standing (shoulder heights equal?) and forward flexion; check for rib hump. Inspect
back for spina bifida. Palpate for tender areas especially sacroiliac joints, compress pelvis to elicit pain of sacroiliitis (hallmark
of ankylosing spondylitis). Muscle tone, percuss costovertebral angles for renal pain. Have patient walk on toes, heels. Ask
patient to stand on one foot at a time and push up into tiptoe for ankle plantar flexor strength (S1).
Sitting
Knee jerks (L4) with quadriceps exposed, watch contraction. Ankle jerks (S1), rapidly dorsiflex each foot to test for clonus.
Babinski. Compare calf girths for wasting by measuring calf circumference 10 cm below tibial tuberosity. Test power of
quadriceps, hamstrings, psoas (raise knee up against resistance), ankle dorsiflexors. Ask patient to straighten both legs and
compare this position to the degree of forward flexion the patient was able to achieve on standing range of motion. Suspicion
of malingering is raised if the patient claims to be unable to bend from a standing position but is able to extend the knees from
a sitting position.
Supine
Feel for lymph nodes at neck, clavicle, axillae, groin. Test hip extensors (patient presses leg into bed while you try to raise it).
Sensation in both legs: light touch, pin prick compare medial dorsum of foot (L5) with lateral foot (S1) and lateral calf (L5)
with posterior calf (S1). Vibration and position sense in big toes. Straight leg raise: raise patients heel on bed as far as patient
will allow, note angle, note whether this reproduced the patients ipsilateral or contralateral radicular pain. Bowstring test: flex
hip to 90 degrees, extend knee to the point of pain and press on the hamstring tendon, which is medial, note reproduction of
pain. Peripheral vascular exam: inspect for venous stasis or arterial insufficiency ulcers, check femoral pulses and auscultate
for femoral bruits, feel popliteal, dorsalis pedis and tibialis posterior pulses.
16. 25 year old man wishes to refill a prescription of Fiorinal for tension headache. Manage.
History: description of headache pain, location (one-sided vs. bilateral/occiput-vertex Do you feel pain on one or both
sides? If one-sided, is it always the same side? If present on both sides, did the pain start on one side? Is it usually maximal on
one side?), quality (pulsatile vs. non-throbbing What kind of pain is it tightening, pressing, throbbing, pounding, pulsating,
burning, etc? Do different types of pain occur at different times in any one attack? If so, what types?), intensity, duration (at
least 72 hours in migraine, if not treated), onset including time of day (morning headache associated with raised intracranial
pressure), previous episodes, aggravating/relieving factors (e.g. coughing and straining worsen headache in raised ICP and
chocolate or cheese can trigger migraines), associated symptoms (aura, nausea, vomiting, photophobia light, phonophobia
sound, osmophobia odors, nuchal rigidity, weakness, numbness, visual disturbances), medical history, medication history
(when was this prescribed, do you have the empty bottle, has it been prescribed before?), current meds, allergies, family
history, substance abuse inquiry, smoking, allergies, mood, stress, anxiety inquiry. Review of systems.
Red flags for headaches: must rule out headaches resulting from meningitis, trauma (subarachnoid hemorrhage, epidural
hemorrhage), tumor, temporal arteritis. History: new onset, headache worse at night, headache wakes patient at night, fever,
neck stiffness, seizures, trauma, changes in LOC/behavior, vomiting, severe, very young/old patients. Physical exam: fundi
abnormal, Kernig/Brudzinski signs (meningitis), focal neurological findings.
Given benign history with no suspicion of raised ICP or focal deficits and a description of headache consistent with the
common tension headache, a full neurological examination is not indicated. Suggest to the examiner that you would perform a
brief neurological screening exam. You will be told to move on.

14

Treatment: Explain that Fiorinal is a combination preparation of barbiturate (butalbital), caffeine and ASA which is properly
used only for the relief of occasional tension headaches. It is habit-forming, can precipitate a withdrawal syndrome including
agitation, delirium and seizures and has additive sedative effects with other CNS depressants. The fact that this patient has
consumed an entire prescription in four days suggests overuse due to dependence. He may also have analgesic headache
syndrome in which inappropriately used analgesics actually cause headaches. Suggest a drug holiday with weaning from
caffeine and alcohol, proper sleep hygiene, diet, exercise and stress management. Chronic headache may also be a symptom of
depression or anxiety, arrange follow up to evaluate for these if the patient does not improve.
17. Elderly woman in hospital post-op day 5 of total hip replacement. Acute chest pain, tachycardia, tachypnea,
shortness of breath. Manage.
Worry about: Life-threatening causes of acute chest pain: MI, PE, pneumothorax and tension pneumothorax, aortic dissection.
Other causes angina, gastritis, reflux, peptic ulcer, pericarditis, herpes zoster, musculoskeletal.
Differential Diagnosis for Chest Pain
Cardiac
Angina
Pulmonary
MI
Pneumonia w/ pleuritis
Pericarditis
Pneumothorax
Myocarditis
PE
Dissecting aorta
Pulmonary hypertension

Non-Cardiac
GI
Esophageal reflux
Ulcer

MSK/Neuro
Arthritis
Chondritis
Rib fractures
Herpes Zoster

Psychologic
Anxiety
Panic

History: Rapid cardiopulmonary history including any history of high blood pressure, heart problems, smoking, COPD.
Physical exam: Is a cardiopulmonary exam as in question #13 above with additional attention to risk factors for post-op
complications (i.e. inactivity/decreased mobility resulting in DVT/PE).
Homans sign: pain in the calf on dorsiflexion of the foot indicates thrombophlebitis. Check that trachea is midline. Inspect
surgical wound. Is the patient on DVT prophylaxis or anti-coagulation?
Treatment: Raise head of bed. Give oxygen 6 L/min by mask. Monitor oxygen saturation. Order stat CBC, lytes, glucose,
INR/PTT, serial CK-MB and Troponin, ABG, CXR, ECG. Give chewable ASA 160-325 mg immediately. Secure IV access,
bolus IV lasix 40 mg, push if fluid overload is suspected, and ventolin if wheezes are heard, give sublingual nitro spray or 0.3
mg sublingual nitro if blood pressure is adequate and 1 mg morphine IV. Repeat nitro q5min x 3. May require additional
morphine and nitro. Repeat CK-MB and Troponin q8h x 3.
ECG: if ECG shows significant ST elevation (more than one millimeter in two anatomically consecutive leads), or a new left
bundle branch block, then the patient is having an MI. Order stat Cardiology consult for possible lytic therapy or cardiac
catheterization. If less severe signs of ischemia are present (flipped T waves, ST depression), follow with repeat ECGs until
resolved.
S1Q3T3: This classic pattern (wide S-wave in lead I, Q-waves in lead III, T wave inversion in lead III) with right axis deviation
and RBBB are signs of right heart strain seen in massive PE.
A-a gradient: An elevated A-a (Alveolar pO2-arterial pO2) gradient is a sign of pulmonary embolus but also occurs in any
condition in which there is a ventilation-perfusion mismatch (e.g. pneumonia, pulmonary edema, COPD). It is determined
from the ABG:
A-a = 713 (FiO2) 1.25 (PaCO2) PaO2
[normal: 12 in child 20 in 70 year old]
Note that the inspired oxygen fraction (FiO2) is not known unless the patient is on room air, a ventimask or mechanically
ventilated. This is because the patient breathes in a proportion of room air which mixes with the oxygen delivered by face
mask or nasal prongs thereby diluting it by an unknown amount. Roughly, however, 2L/min gives 26% FiO 2, 3L=30%,
4L=35%, and 6L=40%. 40% is considered the maximum inspired oxygen obtainable without a high flow mask such as a
ventimask.
ABG normal values: pH 7.35-7.45, pO2 80-100 mmHg, bicarbonate 24, pCO2 40

15

Indications for intubation: An ABG showing poor pO2 (in the 60s, or if less then 80 on high inspired O2 concentrations),
elevated pCO2 (greater than 80), acidemia, or GCS < 8 (not able to protect airway) may indicate need for intubation if these are
not quickly correctable. Consult ICU.
CXR signs of PE: wedge-shaped infiltrate (Hamptons hump) or oligemic area, unilateral effusion, raised hemidiaphragm. A
normal CXR is also consistent, and usual, with PE.
Specific investigations for PE: CT chest (can only see PE which is large enough to be clinically significant), V/Q scan
(conclusive when it shows high or low probability), and serial (q2d) leg Dopplers for presence of DVT above the knee.
Treatment: if suspicion of PE is high, anticoagulate (before waiting for tests) with heparin 7500 U IV bolus, then infuse at
1200 U/h. Measure PTT q6h, adjust dose for PTT 70-90s. If a diagnosis of PE is made, coumadin (warfarin) should be started,
continue anti-coagulation for 3 months.
18. Young mother with 6 week old baby has recently immigrated from Ghana. Poor English skills. Concerned about
whether she should have her baby immunized. Counsel.
Counseling: General principles of counseling are to be aware of communication barriers such as language difficulties, to
understand the patients objectives, fears, preconceptions, to deal with these in an empathetic, non-judgmental way and to
normalize them to transmit information in a way that is consistent with the patients expectations and understandable to them,
and to invite further questions and feedback.
History: Ask if the patient would prefer someone, perhaps a family member to translate. Ask about the patients concerns,
what does she want to know and why? Explain that vaccines protect children from diphtheria, tetanus, pertussis, polio (DPTP),
mumps, measles, rubella (MMR), influenza (Hib) and hepatitis B (Hep B). All of which were once common and caused
serious, sometimes fatal illness in Canada, and all of which are now hardly ever seen because of vaccines. Explain that
because the vaccines stimulate the immune system, some children have a temporary sore arm (with induration and tenderness)
at the injection site, malaise, mild fever or rash. Allergic reactions to vaccines also occur including urticaria, rhinitis,
anaphylaxis. It is very rare to have a more serious reaction (seizures, encephalopathy have been reported). Standard modern
vaccines are not known to cause disease or to have long-term deleterious effects. Compare these risks with the risk of not
getting vaccinated. Explain the recommended immunization schedule (below), give the patient some information pamphlets,
invite further questions and ask her to return in two weeks for the childs first immunization.
Contraindications to vaccination: previous serious reaction to vaccine. Special contraindication to MMR, which is a live
attenuated vaccine suspended in egg white protein and preserved with neomycin: allergy to egg or neomycin, pregnancy and
immunocompromised state (except healthy HIV positive children). Special contraindications to the pertussis component of
DPTP (which is thought to be the component responsible for seizures and encephalopathic vaccination reactions when they
occur): progressive neurologic disorder and epilepsy. Hib not to be given after age 5. TdP should not be given in first trimester
of pregnancy.
Recommended Immunization Schedule:
DPTP = diphtheria, tetanus, acellular pertussis and inactivated polio, Hib =
Hemophilus influenza type b, MMR = measles, mumps, rubella, Hep B = hepatitis B,
TdP = tetanus, diphtheria toxoid and pertussis, Td = tetanus and diphtheria toxoid,
NB: MMR is administered SC while the others are IM.

2 months
4 months
6 months
1 year
18 months
4-6 years
12-13 years
14-16 years
q10yrs

DPTP, Hib (given as Pentacel)


DPTP, Hib
DPTP, Hib
MMR
DPTP, Hib
MMR, DPTP
Hep B (3 doses initial, 1 month, 6 months)
TdP (certificate of immunizations for high school)
Td

19. 40 year old woman, appears sad, requests sleeping pills. Manage.
History: Name, age, occupation. Social situation. Stressful life events. Depression commonly presents with sleep disturbance
therefore screen and treat for depression as in question #4 above. This should be in addition to a sleep history. Medications,
allergies, drugs/smoking/alcohol, past medical/surgical history, family history, review of systems.

16

Sleep history: usual requirements, chronology of sleep problems, stressor, sleep hygiene (when, where, regularity, shifts at
work, quiet, late, exercise, meals, alcohol, caffeine, prescription and non-prescription remedies, drugs and medications), sleep
latency (time to fall asleep), nocturnal awakening, early morning wakening, daytime somnolence, somnolence while driving,
working or during conversation.
Proper sleep hygiene: regular bed and wake times, avoid daytime naps, regular exercise but not late in the evening, do not use
the bed for reading, TV, paperwork, etc., avoid caffeine, alcohol, smoking.
20. 60 year old woman with multiple pains investigated by several other doctors, all lab tests normal. Manage.
History of multiple pains: should address the differential for multiple pains.
Differential for multiple pains:
Depression with somatization: major depression presents with a somatic complaint; commonly headache, stomach pains,
sleep disturbance, eating disturbance, or bowel habit changes. This is a frequent presentation of depression in the elderly.
Treat as in depression (see #19 above).
Somatization disorder: multiple non-intentional complaints in multiple organ systems beginning before age 30 that occur over
several years, with treatment sought and significant impairment in functioning. Diagnostic criteria: at least 8 physical
symptoms that have no organic pathology: 4 pain symptoms at 4 different sites, 2 GI symptoms other than pain, 1 reproductive
or sexual symptom other than pain, 1 pseudo-neurological symptom (e.g. temporary blindness). Complications: anxiety,
depression, unnecessary medications or surgery. Often a misdiagnosis for an insidious illness so rule out all organic illnesses
(e.g. MS). Treatment: counseling, psychotherapy, close follow-up, reassurance.
Conversion disorder: psychic perturbation presents as one or two neurological complaints affecting voluntary motor or
sensory function. Psychological factors thought to be etiologically related to the symptom as the initiation of symptoms is
preceded by conflicts or other stressors. La belle indifference patients inappropriately cavalier attitude towards a serious
symptom. Treatment: anxiolytics (e.g. lorazepam 1 mg PO q6h), relaxation therapy, counseling, close follow-up.
Pain disorder: e.g. chronic post-traumatic or post-surgical pain. Pain not fully accounted for by current tissue injury,
exacerbated by psychic factors and associated with functional impairment. Treatment: amitriptyline 25-75 mg PO qHS.
Hypochondriasis: exaggeration or misinterpretation of normal sensory phenomena to the point of functional disability.
Evidence does not support a physical disorder. Associated with obsessive fear of serious illness and doctor shopping despite
reassurance. Belief is not delusional as person acknowledges unrealistic interpretation. Treatment: counseling, reassurance,
close follow-up.
Fibromyalgia: also called fibrositis and fibromyositis. 80-90% of cases occur in middle-aged females, may afflict 5% of adult
women, typically cardiovascularly unfit, depressed, previously normal life (onset often after car accident). Associated with
absent or decreased non-REM stage 4 sleep, patients wake from sleep feeling unrefreshed. Constant, aching, axial pain with
bilateral tender points (not trigger points, at which referred pain is triggered due to myofaschial pain from overuse, e.g. tennis
elbow). The disorder follows a waxing and waning course ultimately without progression or resolution, and may become
disabling. Characteristic reproducible tender points are located bilaterally at lateral border of sternum, sternocleidomastoid,
posterior neck, trapezius, rhomboids, over sacroiliac joints, lateral thigh, posterior and medial knee. Patient should have eleven
of the above eighteen tender points for a diagnosis. Treatment: amitriptyline 25-75 mg PO qHS.
Chronic fatigue syndrome: similar to fibromyalgia but fatigue is the dominant feature and pain and tender points may be less
prominent or absent. Treatment: amitriptyline 25-75 mg PO qHS.
Factitious disorder or malingering: Factitious disorder involves misrepresentation of history and symptoms for the purpose
of assuming the sick role with its inherent secondary gains (attention and sympathy, justification for inadequacies).
Munchausens syndrome is the type of factitious disorder in which physical findings are faked by contamination of lab tests or
ingestion of inappropriate medication or substances. Typically the patient is a medical paraprofessional motivated by hostility
toward the medical establishment, e.g. nurse takes coumadin to fake hemophilia. Malingering is distinguished from factitious
disorder by a motivation for secondary gain other than the sick role, such as insurance benefits. Treatment: counseling.
History: Pain description, location, duration, chronology, aggravating and relieving factors, are pains linked to one another?

17

Somatoform disorders screen: How has your health been for most of your life? How have your pains affected your job,
social life, relationships, and your life generally? Are you often unwell, how often do you visit the doctor? Do you worry that
you have a serious illness? If a doctor tells you that there is nothing wrong, how does that make you feel? Do you believe him
or her?
Associated symptoms: review of systems, medications, allergies, smoking, alcohol, drug use, family history, depression
history as in #4 above.
Diagnosis and treatment: For non-specific pains with depressive symptoms the patient most likely has depression with
somatization. Treat for depression as in #4.
21. A young man presents to the Emergency Department having twisted his ankle. Manage.
History for ankle sprain: history of a plausible mechanism of injury involving significant inversion or eversion of the foot
with pain and swelling. Time of injury, onset of pain and swelling (may be delayed), noises heard at time of injury. Previous
ankle or other injuries. Ability to walk post injury (often preserved if ligaments are not ruptured). Past medical history,
medications, allergies, family history.
Physical exam: inspect for gross deformity, erythema, swelling, bruising. Check distal circulation, sensation, active and
passive range of motion, palpate for tenderness at joints. Examine the joints above and below the affected joint. Identify sites
of maximal tenderness. Point tenderness over the area anterior (anterior talofibular ligament), inferior (calcaneofibular
ligament), or posterior (posterior talofibular ligament) to the lateral malleolus are signs of lateral ligamentous injury.
Tenderness over the area medial and inferior to the medial malleolus indicates deltoid (medial) ligament injury.
Talar drawer sign: Stabilize the tibia and pull forward on the heel, talar drawer sign is anterior movement of the talus. Greater
than 3 mm anterior movement may be significant. 1 cm is significant and indicates anterior talofibular ligament rupture.
Talar tilt: Stabilize the tibia, grasp the talus and tilt in inversion and eversion. Movement beyond the normal range compared
with the opposite side is a positive talar tilt and indicates lateral calcaneofibular ligament rupture if the tilt occurs in inversion
or medial (deltoid) ligament rupture if the tilt occurs in eversion.
Squeeze test: Pain in the ankle on squeezing the calf is a sign of ankle fracture.
Ottawa Ankle Rules: for taking ankle series x-rays (includes lateral and AP ankle with mortis view). X-ray if there is pain
over the malleolar zone AND tenderness on palpation of the posterior medial or lateral malleolus OR if the patient is unable to
weight bear immediately and in ER. For foot series (AP and lateral foot): X-ray if there is pain in midfoot zone AND bony
tenderness over the navicular or base of 5th metatarsal OR unable to weight bear immediately and in ER. Calcaneal views if
there is pain on palpation of heel.
Treatment for ankle sprain: (remember RICE) Rest, use crutches, avoid weight bearing but early mobilization. Ice for 20
minutes QID for 2-3 days. Consider NSAIDs: Ibuprofen 400-600 mg PO q6h if no contraindications. Compression with
tensor bandage or tape but not to the point of pain. Elevate. Rehabilitation: start active range of motion exercises 2 days post
injury, may weight bear after pain and swelling have subsided. Full ligament healing may take 6 weeks in severe injury or
more if re-injury occurs. Complete ligament rupture with joint instability (positive talar drawer sign or talar tilt) should be
evaluated by Orthopedics.
22. 16 year old known epileptic on Dilantin is having 3 seizures per month and requests better medication. Manage.
Findings: not taking meds, experiencing stress.
See question #3 above.
23. 50 year old woman with headache and normal vitals. Take a history. Q: Describe appropriate investigations and
treatment for temporal arteritis.
History: description of headache pain, location (one-sided vs. bilateral/occiput-vertex Do you feel pain on one or both
sides? If one-sided, is it always the same side? If present on both sides, did the pain start on one side? Is it usually maximal on
one side?), quality (pulsatile vs. non-throbbing What kind of pain is it tightening, pressing, throbbing, pounding, pulsating,
burning, etc? Do different types of pain occur at different times in any one attack? If so, what types?), intensity, duration (at
least 72 hours in migraine, if not treated), onset including time of day (morning headache associated with raised intracranial

18

pressure), previous episodes, aggravating/relieving factors (e.g. coughing and straining worsen headache in raised ICP and
chocolate or cheese can trigger migraines), associated symptoms (aura, nausea, vomiting, photophobia light, phonophobia
sound, osmophobia odors, nuchal rigidity, weakness, numbness, visual disturbances). A history of unilateral lancinating pain
with swelling and tenderness in the temporal area should prompt inquiry after symptoms of polymyalgica rheumatica (PR
pain and stiffness in muscles of neck, shoulders, upper arms, hips, lower back and thighs no weakness or atrophy
increased ESR, anemia, normal CK responds to steroids immediately), which is related to temporal (giant cell) arteritis and
may be a more systemic variant of the same underlying disease. Symptoms of both PR and temporal arteritis include low grade
fever, malaise, anorexia, weight loss, bilateral proximal muscle weakness, aching and pain, as well as joint inflammation. Jaw
claudication, stroke and blindness may occur due to vasculitic occlusion of arterial supply. Ask about visual changes. Medical
history, medication history (when was this prescribed, do you have the empty bottle, has it been prescribed before?), current
medications, allergies, family history, substance abuse inquiry, smoking, allergies, mood, stress, anxiety inquiry. Review of
systems.
Investigations for temporal arteritis: CBC (mild anemia with increased WBC), ESR (greater than 50 mm/h, [normal 30]), Creactive proteins, liver enzymes, temporal artery biopsy, may add temporal artery angiogram to guide biopsy.
Treatment: (in the absence of visual symptoms) without waiting for biopsy, start high dose oral prednisone 60 mg PO OD until
symptoms subside and ESR normal, then 40 mg PO OD for 4-6 weeks, then taper to 5-10 mg PO OD for 2 years (relapses
occur in 50% if treatment is terminated before 2 years). Treatment does not alter biopsy results if the sample is taken within 2
weeks. Monitor ESR regularly. If visual symptoms are present, or develop during treatment, the patient is admitted and given
prednisolone 1000 mg IV q12h for 5 days.
24. HIV positive man. 1 week of shortness of breath, cough, fatigue. Perform a physical exam. Q: Give a differential
diagnosis for a CXR showing a fine reticular pattern in the left lower lobe. Manage.
Physical exam: A physical exam for query pneumonia consists of the cardiopulmonary exam as in question # 13 with
additional attention to the particular signs and symptoms of HIV infection.
Examination for lobar consolidation: In general, pulmonary effusion decreases transmission of breath and vocal sounds to
the chest wall, while consolidation (seen in pneumonia) increases it. Four maneuvers bring out the effect of increased
transmission: tactile fremitus is increased transmission of palpable fremitus to the chest wall while the patient repeats
ninety-nine, bronchophony is enhanced transmission of spoken words such as ninety-nine, egophony is a change from an
ee to an ay sound over the affected area while the patient sustains an ee sound, and whispered pectoriloquy is a marked
increase in audibility through the chest wall over the affected area while the patient whispers the words ninety-nine or onetwo-three.
Signs of consolidation: increased tactile fremitus, percussion dullness, crackles, bronchial breath sounds, increased voice
transmission (bronchophony, egophony, whispered pectoriloquy).
Signs of HIV Infection: (and possible impending AIDS) check entire skin surface for Kaposis sarcoma, examine pharynx for
thrush or oral hairy leukoplakia (Epstein-Barr virus-related epithelial proliferation causing raised white plaques on the sides of
the tongue), palpate neck, clavicle, axillae, and groin for lymph nodes enlarged by non-Hodgkins lymphoma. Examine
abdomen for hepatic or splenic enlargement.
Differential diagnosis of unilateral lobar reticular pattern on CXR: pneumocystis carinii pneumonia (PCP), Kaposis,
lymphoma, cytomegalovirus (CMV), tuberculosis, Cryptococcus neoformans, Hemophilus, Streptococcus, mycoplasma,
chlamydia. The classic CXR of PCP, an AIDS-defining illness, is bilateral hilar infiltrates, but X-ray findings are variable and
may be alveolar or interstitial.
Investigations: O2 sats/ABG, CBC with differential and CD4 count, LDH (elevated in 95% of PCP pneumonias and not in
other pneumonias), blood cultures, sputum for cytology/gram stain/culture/TB stain if sputum available (cough usually nonproductive and induced sputum may fail), bronchoscopy with cytology, gram stain and culture of bronchial washings and
brushings (may see bronchial Kaposis).
Treatment: Septra DS 2 tabs q8h x 14 days outpatient with 1 tab OD or BID 3/week continued as prophylaxis. More specific
therapy with results of diagnostic tests. In severe illness, admit to hospital, give IV Septra at same dose and Prednisone 40 mg
BID x 5 days, then OD x 5 days, then 20 mg OD for prophylaxis. Patient should be referred to an AIDS specialist for antiviral
and possible experimental therapies. Counseling and referral to support organizations. Follow up appointment.

19

25. A 2 day old infant has serum bilirubin 220 mol/L (ref. Max 200 mol/L). Take a history from the mother. Q: What
are the possible causes for this abnormality? Give investigations and treatment.
50% of term infants develop visible jaundice (> 85-120 mol/L or 5-6 mg/dL)
Mothers obstetrics history: GTPAL (number of gestations, term pregnancies, premature births, abortions, live children),
history of previous pregnancies including neonatal jaundice, maternal medical history esp. liver disease, illness during
pregnancy esp. diabetes (large birth weight, pre-eclampsia), rubella (teratogenic), toxoplasmosis (from cats, infects fetal brain),
herpes (infects fetus, frequently fatal), CMV (damages fetal liver), teratogenic medications taken during pregnancy, drug and
alcohol use, maternal blood type, complications of present pregnancy including gestational hypertension or diabetes,
hyper/hypothyroid, hypercoagulation. Family history of neonatal jaundice, liver problems.
Newborn history: gestational age at birth, caesarean, induction, rupture of membranes artificial or prolonged, fetal distress,
forceps or vacuum delivery, meconium, APGARs, was resuscitation required? Initial blood work, breast feeding? How often
and how well, color of 1st stool, color of urine, vomiting, neonate muscle tone, behaviors, fever, irritability, lethargy.
Causes of neonatal jaundice: unconjugated = physiologic neonatal jaundice OR pathologic: hemolytic ABORh
incompatibility, neonatal sepsis, splenomegaly, hereditary spherocytosis, G6PD etc.; non-hemolytic breast milk jaundice,
breakdown of cephalohematoma, polycythemia, sepsis, Gilberts, Crigler-Najjar, hypothyroidism. Conjugated: GI obstruction
in fetus (increases enterohepatic circulation), bile duct obstruction, drug-induced and multiple other less common causes.
Investigations: Use blood obtained by venipuncture, not heal prick, as sludging of heel prick blood in skin capillaries distorts
cell counts and concentrations. Measure direct (conjugated) and indirect (unconjugated) bilirubin, neonatal and maternal blood
types, Coombs test (see below), blood smear, CBC with reticulocyte count. Septic work-up, urinalysis, blood cultures, CXR,
AXR, CSF.
Coombs test: a two-part test. 1. Direct antiglobulin test: detects presence of anti-red cell autoimmune antibodies attached to
red cells. Patients red cells agglutinate when mixed with a solution containing anti-human immunoglobulin antibodies, i.e.
multiheaded anti-immunoglobulin antibodies bind to the constant chains of antibodies on the red cell surface, sticking red cells
together. 2. Indirect antiglobulin test: detects presence of anti-red cell antibodies in patients serum. Patients serum incubated
with red cells of the same blood group, test is positive if agglutination occurs.
Treatment: The aim of treatment is to correct anemia and decrease hyperbilirubinemia. Severe hyperbilirubinemia may lead to
kernicterus (deposition of bilirubin in the brainstem and basal ganglia leading to mental retardation, cerebral palsy, hearing
loss and paralysis of upward gaze). Indications for intervention: In general, serum total bilirubin level greater than 300 um/L is
an indication for phototherapy, while levels greater than 400 um/L call for exchange transfusion or plasmaphoresis.
Therapeutic modalities: In phototherapy photoisomers of bilirubin are produced which are more water-soluble and can be
excreted without conjugation, while exchange transfusion replaces the babys blood with donor blood. Plasmaphoresis is the
replacement of blood plasma with donor plasma while retaining the patients own red and white cells.
Specific treatment of jaundice depends on the cause:
Physiologic jaundice: occurs on day 2-3 in 50% of term infants (NEVER within 1st day) and resolves by day 7. It is more
common in preterm infants where it occurs later, up to day 6, and lasts longer. Due to transient limitation in bilirubin
conjugation (i.e. the hyperbilirubinemia is unconjugated or indirect). Usually requires no treatment.
ABORh incompatibility jaundice: Rh incompatibility (Rh negative mother with Rh positive fetus) tends to be more severe
than other incompatibilities (ABO incompatibility: e.g. O mother with A or B fetus) and may cause hydrops fetalis
(generalized edema, including pulmonary edema, with high output heart failure). Treatment for all types is exchange
transfusion or plasma electrophoresis.
Sepsis-related jaundice: treat underlying cause and use phototherapy.
Breast milk jaundice: occurs day 4-7 and peaks at 2nd to 3rd week, rare (1:200), long chain fatty acids in breast milk
competitively inhibit glucuronyl transferase activity. Treatment is to substitute formula for breast milk for 2-4 days, then
resume.
26. A 50 year old man is denied life insurance because of abnormal liver function tests. AST > ALT very elevated, ALP
slightly elevated, Bili normal. Take a history. Q: Give a differential diagnosis. What investigations would you order?

20

History: Name, age, occupation. History of jaundice, hepatitis, foreign travel, blood transfusions, recreational IV drug use.
Dark urine, pale stool, abdominal pain, fever/chills, decreased appetite, weight loss, night sweats, nausea and vomiting,
pruritis, easy bruising, gynecomastia, hemorrhoids (from portal hypertension), alcohol use & CAGE questionnaire (see below).
Sexual history: number of past and present partners, genders of same, sexually transmitted disease. Medications, drug use,
smoking, allergies, past medical history, family history, review of systems.
CAGE questionnaire: Control have you tried to cut down on your alcohol? Anger have you ever felt angry when someone
suggested you decrease your alcohol intake? Guilty have you ever felt guilty about your drinking? Eye opener do you
sometimes have a drink to get started in the morning?
Liver Enzymes: AST & ALT are sensitive but non-specific markers of hepatocellular damage hepatitis (inflammation)
vascular injury (ischemia). AST > ALT = alcoholic liver disease; ALT > AST = viral hepatitis. ALP & GGT are markers
of cholestatic disease intrinsic (toxic, infectious, inflammatory), systemic (sepsis, pregnancy), infiltrative (tumor, fat,
lymphoma), mass lesions (stone, tumor, abscess). AST/ALT > 2 (AST usually < 300) alcoholic liver. Serum transaminases
> 1000 due to 1) viral hepatitis, 2) drugs, 3) passage of common bile duct stone, 4) hepatic ischemia.
Differential: alcoholic liver disease, viral hepatitis, liver malignancy (metastatic or primary).
Investigations: Viral serology (Hep A, B, C antibody and B antigen presence of B antigen for > 6 months indicates chronic
carrier state), GGT, AST, ALT, AlkPhos, LDH, bilirubin, INR/PTT, albumin, glucose (cirrhosis), serum ceruloplasmin, serum
copper (Wilsons disease), serum ferritin, total iron binding capacity (TIBC, for hemochromatosis), ANA, anti-smooth muscle
antibody (autoimmune hepatitis, also called chronic active hepatitis), abdominal ultrasound, liver biopsy.
27. 19 year old female with vaginal discharge. Take a history. Q: Give three possible diagnoses, what investigations
would be helpful?
History: Name, age, occupation, description of discharge, onset, chronology, previous episodes, volume, color, consistency,
odor, timing (related to menses?). Associated symptoms: pain including abdominal, burning, fever, itch, dyspareunia, dysuria,
urgency, frequency, aggravating and relieving factors. Sexual history: number of past and present partners, gender, type of
contraception (condoms), possibility of pregnancy, past history of sexually transmitted disease. Obstetrics/gynecology history:
(GTPAL) pregnancies, abortions/miscarriages, births, pap smears (normal?), menstrual pattern. Medications (especially
antibiotics), oral contraceptives, other drug use, allergies. Past medical history including diabetes. Family history, review of
systems.
Causes of discharge: Physiological: normal midcycle discharge, increased estrogen states. Infectious: candidiasis, bacterial
vaginosis (Gardnerella vaginalis), trichomonas infection, chlamydia, gonorrhea (NOTE: gonorrhea and chlamydia can cause
cervicitis, PID and urethritis, but do not cause vaginitis but Toronto Notes includes them in the differential for vaginal
discharge), bartholinitis or Bartholin abscess, PID. Neoplastic: vaginal intraepithelial neoplasia (VAIN), vaginal squamous cell
carcinoma, invasive cervical carcinoma, fallopian tube carcinoma. Other: allergic/irritative vaginitis, foreign body, atrophic
vaginitis, enterovaginal fistulae.
Investigations: speculum exam, swab and culture, saline slide microscopy and KOH whiff test (add KOH to vaginal secretions
on a slide). These give results as follows:
Candidiasis: inflamed appearance, lumpy white discharge, spores and pseudohyphae seen under microscope. Treatment:
miconazole vaginal suppository.
Bacterial vaginosis: non-inflamed, thin gray secretions, clue cells under microscope (epithelial cells with obscured borders
due to adherence of bacteria), fishy odor on KOH test. Treatment: metronidazole 500mg PO BID x 7 days (in pregnancy use
Amoxicillin 500mg TID x 7 days).
Trichomonas: inflammations, frothy yellow-gray-green discharge, motile trichomonads seen under microscope. Treatment:
metronidazole 2 g x 1 or 500mg PO BID x 7 days (in pregnancy use Clotrimazole vaginal suppositories).
28. 60 year old man with microscopic hematuria on routine urinalysis. Take a history. Q: Give a differential diagnosis,
what investigations would be helpful?

21

Etiology of Hematuria by Age Group


Age
Etiology (in order of decreasing frequency)
0-20
Glomerulonephritis, UTI, congenital anomalies
20-40
UTI, stones, bladder tumor
40-60
Male: bladder tumor, stones, UTI
Female: UTI, stones, bladder tumor
> 60
Male: BPH, bladder tumor, UTI
Female: bladder tumor, UTI
History: patient ID, suprapubic pain, pain on urination, frequency, urgency, frank blood in the urine (globular clots from
bladder or string shaped clots from ureters), color of urine, difficulty initiating or maintaining urinary stream, renal pain, back
pain, groin pain. Provoking factors (e.g. exercise, trauma). History of recent UTI, STDs, TB exposure, pelvic irradiation,
bleeding diathesis, smoking. Fever, chills, nausea, fatigue. Previous renal colic/diagnosed nephrolithiasis? History of
hypercalcemia, hypertension. Malignant symptoms: night sweats, weight loss, fatigue. Medications, drugs (NSAIDs,
anticoagulants)/alcohol, smoking, anticoagulants and salicilates, past medical history, past surgical history, family history
(polycystic kidney disease?), review of systems.
Differential diagnosis: BPH, transitional cell carcinoma of bladder, UTI, nephrolithiasis, hydronephrosis, prostatitis, prostate
cancer, renal cell carcinoma, glomerulonephritis, trauma (heart valve, muscle injury/breakdown), essential hematuria (tends to
occur in children). NOTE: microscopic hematuria is normal in 10% of population.
Investigations: prostate specific antigen (PSA), CBC (to rule out anemia, leukocytosis), urine for microscopy (casts, crystals,
culture and sensitivity, cytology), abdominopelvic ultrasound, cystoscopy, intravenous contrast urography, intravenous
pyelogram (IVP).
29. 25 year old male with a history of dyspepsia and binge drinking has abdominal pain. Perform a focused physical
exam. Q: What radiological investigations would you order and why?
Physical exam for abdominal pain: vitals, posture (unmoving in fetal position suggests peritonitis while writhing suggests
renal colic), jaundice, nutritional status, buccal mucosa, teeth, breath (hepatic fetor), parotid hypertrophy, glossitis, inspect
chest for telangectasia, gynecomastia, loss of axillary hair. Hands: palmar erythema, clubbing, Dupuytrens contracture,
wasting of hand intrinsics.
Abdominal exam (supine): inspect for caput medusa, Cullens sign (umbilical bruising intraperitoneal bleed), GrayTurners sign (bruising of flank retroperitoneal bleed), pulsations, auscultate for bowel sounds, renal bruits. Estimate size of
liver and spleen by percussion. Palpate for liver edge, Murphys sign (examiner presses on liver edge after patient has exhaled,
patient catches breath on inspiration), splenic enlargement (begin palpation at right lower quadrant to catch very large spleen
and percuss at Traubs space), hard stool in bowel. Note cough tenderness, rigidity, rebound, guarding, tenderness, pain at
McBurneys point (one third of the way along the line between the right anterior iliac crest and the umbilicus). Rovsings sign:
palpation of the LLQ produces RLQ pain. Psoas sign: pain on passive or active flexion at the hip, indicates peritoneal irritation
over the psoas or psoas abscess. Obturator sign: pain on internal or external rotation of the hip, indicates bowel herniation into
the obturator canal. Assess for ascites. Ask patient to roll onto side and pound costovertebral angles lightly with fist, CV angle
tenderness indicates kidney pain due to pyelonephritis or nephrolithiasis. Palpate groins for hernias.
Rectal: palpate prostate, rectal shelf, check for gross or occult blood.
Radiological Investigations: Abdominal 3 views: supine, upright, left lateral decubitus. Dilated bowel with multiple air/fluid
levels indicates ileus. Dilated proximal bowel with collapsed distal bowel indicates obstruction. Check for gastric distension.
Small bowel has circular plica: lines go all the way across. Large bowel has interrupted haustra: lines go halfway across.
Check for calcified kidney stone, fecolith and appendiceal air/fluid level. Can see gallstones and abdominal aortic aneurysm if
calcified. Abdominal ultrasound: gallstones, cholecystitis, pancreatitis, appendicitis, hydronephropathy, kidney stones,
abdominal aneurysm. CXR: check for free air under the diaphragm in an upright film.
30. 70 year old male with neck pain and left arm weakness. Perform focused physical exam. Findings: decreased
sensation over left index and middle finger, mild wrist extensor and triceps weakness. Q: Describe a cervical spine film
of the patients neck (shows narrowing of C6-7 disk space). Diagnosis and treatment?
Physical exam for neck pain: guided by differential for the causes of neck pain: musculoskeletal, discogenic, stenotic,
malignant or brainstem infarct. Take vitals, cranial nerves: pupillary reflexes, extra-ocular movements, visual fields, facial

22

muscles (frown, raise eyebrows, show teeth, protrude tongue), facial sensation, gag, Horners triad (ptosis, anhidrosis, miosis
on side of sympathetic palsy?), sternocleidomastoid and trapezius power. Cerebellar testing: finger-nose, heel-shin,
dysdiadocokinesis, gait, Rhomberg, Pronator drift.
Neck: inspect for lesions, asymmetry, muscle wasting, especially sternocleidomastoids, palpate for nodes, masses, palpate
dorsal vertebral spines, range of motion.
Shoulders, arms, hands: inspect for symmetry, wasting, fasciculations, skin lesions. Power: Test deltoids (C5), biceps (C6),
triceps and wrist extension (C7), hand intrinsics (C8). Note that each muscle group actually has mixed nerve root innervation,
i.e. deltoids and biceps (C5,6), triceps (C6,7,8), wrist extension (C6,7), hand intrinsics (C8,T1). Sympathetic outflow occurs at
C8, T1. Sensation: check pinprick, vibration, light touch over the shoulder (C5), thumb (C6), index and middle finger (C7),
ring and little finger (C8). Deep tendon reflexes at biceps, triceps, brachioradialis, Hoffmans sign (the Babinski of the upper
limb: flick relaxed index finger dorsally, thumb abducts for positive test). Tone/rigidity: check for increased tone by rapid
supination and rapid extension of elbow.
Lateral Cervical Disc Syndrome
C4-5
Root involved C5
Motor
Deltoid
Supraspinatus
Biceps
Reflex
Supinator
Sensory
Shoulder

C5-6
C6
Biceps

C6-7
C7
Triceps

C7-T1
C8
Digital flexors
Intrinsics

Biceps
Thumb

Triceps
Middle finger

Finger jerk
Ring finger
Little finger

Peripheral nerves: Check two point discrimination at each fingertip. To determine peripheral nerve damage: Median nerve
territory is the palmar surface of the thumb, and the palmar surface and dorsal tips of the index, middle and thenar side of the
ring fingers. Sensation to the ulnar side of the hand is the ulnar nerve, and the dorsal surface of the thenar side of the hand is
radial nerve innervated. The median nerve also innervates most muscles of the thenar eminence, and the 1st and 2nd lumbricals.
The thumb is weak in abduction at 90 degrees to the plane of the hand in median nerve dysfunction. Tinels sign: tapping the
palmar surface of the wrist elicits shooting paresthesia in median distribution. Phalens sign: maximally flexing both wrists by
pushing the dorsi of the hands together elicits median nerve distribution numbness or paresthesias after 30-60 seconds.
C-spine X-rays: Lateral: an adequate lateral shows the top of the T1 vertebra: look for alignment of the anterior and posterior
margins of vertebral bodies as well as spinous processes. Spinous processes may have abnormal separation in injury. The
maximal normal distance between the posterior aspect of the anterior arch of C1 and the dens is 3mm in adults and 5mm in
children. Look for regularity of disk space height, gas in the disk space (suggests degeneration), osteophytes, pre-vertebral
swelling greater than one third of the vertebral body width (7 mm from C1-4, 22mm from C5-7). Hangmans fracture: coronal
plane fracture through the base of both pedicles of C2, caused by hyperextension injury, separates the posterior elements of C2
from its body. AP: check alignment of processes and vertebral bodies, distance between spinous processes should be regular,
pedicles should be seen in cross section (erosion of a pedicle can cause the winking owl sign where the pedicles are the eyes
and the spinous process, the beak). Odontoid view: trace bone cortex around the outline of the dens, misalignment of this
outline indicates odontoid fracture, articular spaces of atlantoaxial joints on either side of the dens should be equal. Note:
odontoid fracture type I tip, type II base, type III through body of C2.
Diagnosis: A narrowed C6,7 disk space suggests disk degeneration at that level. C6,7 disk herniation would impinge on the C7
nerve root (cervical roots exit above the vertebra of the same number) which is consistent with clinical C6,7 nerve root
dysfunction on sensory and motor exam.
Treatment: most patients respond to conservative therapy: soft collar, NSAID, acetaminophen. If symptoms persist for two
weeks or neurologic symptoms progress, refer to Neurosurgery for myelogram, CT neck and possible CT myelogram, MRI, or
EMG, nerve conduction studies. May require decompressive laminectomy or anterior discectomy with bone graft fusion.
31. A 30 year old patient with type I diabetes presents to the emergency department with abdominal pain and vomiting.
Take a history. Q: Labs: Glucose 25, K 6.0, pH 7.22, Bicarb 14. What is your diagnosis and management?
History for abdominal pain and vomiting: quality of the pain, location, onset, chronology, radiation, associated symptoms,
aggravating and relieving factors. Number of episodes of vomiting, description of vomit, presence of blood and bile.
Associated prodromal illness, fever, malaise, sore throat, cough, urinary symptoms, diarrhea. Foods eaten, other people sick?

23

Previous similar episode? Polydipsia, polyuria, lethargy, anorexia, hyperventilation? Other precipitants of DKA, recent
surgery, recent trauma, pregnancy, MI, non-compliance or wrong insulin dose, infection.
Diabetes history: time since diagnosis, medications, blood sugar monitor at home? Diabetic control, polyuria, polydipsia, diet,
exercise, drugs, alcohol, smoking, complications of diabetes (retinopathy, neuropathy, nephropathy, infections). Who follows
patients DM? Has patient taken insulin since feeling unwell, last insulin dose? Past medical history, current medications,
allergies, family history, review of systems.
Diagnosis: diabetic ketoacidosis.
Management: Foley, IV, lytes, glucose, ABG, serum ketones. Septic work-up: CBC, CXR, blood cultures, urinalysis, ECG if
K is critically elevated. 1 L NS per hour x 2-3 hours or until tachycardia and BP normalize, then 500 cc/hr x 2 hours, then 250
cc/h x 5 hours. Insulin drip at 2 U/hr. Check glucose and lytes q2h. When glucose drops to 15, switch fluids to maintenance
2
/3-1/3 D5W/NS + 20 mEq KCl/L (4:2:1 rule). Begin diet and regular insulin regimen. If the DKA was the result of noncompliance close follow-up and education such as diet and diabetes management counseling with a dietitian are required.
32. A mother with her 6 week old who has been vomiting for three days. Take a history. Q: Investigations show a
palpable mass in the right epigastrium, metabolic hypochloremic alkalosis. What is the diagnosis? Give a differential
diagnosis for vomiting in an infant.
History of infant vomiting: age of onset, duration, severity, chronology, association with feeding or body position, description
of force, volume, color, composition (bilious, fecal, blood, regurgitant), getting worse or better, is child still hungry afterward,
or does he settle. Coughing or gagging with feeds (tracheoesophageal fistula). Associated diarrhea, constipation, fever, weight
loss, abdominal distention, urination. Are other children sick? Has child been in contact with an infected person.
Mothers obstetrics history and newborn history: as in question #25 above
Development history: age and weight normograms, feeding history: quantity, frequency, breast vs. bottle (which formula),
colic, feeding difficulties. Past medical history, medications, family history.
Diagnosis: pyloric stenosis.
Differential diagnosis for infant vomiting: Newborn: congenital malformation (pyloric stenosis, tracheoesophageal fistula,
duodenal atresia, malrotation of the intestine). Post newborn period: gastroenteritis, peritonitis, appendicitis, hepatitis, ulcers,
pancreatitis, overfeeding, reflux, food allergy, milk protein intolerance, systemic infection.

1995
33. 79 year old female collapses in the mall. Patient is drowsy, unresponsive to verbal stimuli. Manage. Findings: HR
40, BP 80/40, ECG complete heart block.
Rescusitation: ATLS/ACLS format as in question #6 above.
Management of complete heart block: (P waves seen on ECG not related to QRS complexes). Transcutaneous pacing
(atropine 1 mg IV may be tried but is rarely effective). Patient will require sedation (midazolam 2 mg IV) and analgesia
(morphine 2 mg IV) before starting external pacing. Will require placement of a transvenous pacer until a permanent pacer can
be placed. Consult Cardiology/CCU/ICU.
Causes of AV conduction abnormalities: calcification of the conducting system (Levs and Lenegres disease), inferior MI,
coronary spasm, digitalis overdose, tricyclic antidepressant overdose, -blockers, calcium channel blockers, viral rheumatic
fever, Lyme disease, sarcoid, amyloid, hemochromatosis, cardiac tumor, congenital.
34. 25 year old male with tricyclic antidepressant overdose. Manage.
Resuscitation: ATLS/ACLS as in question #6 above ABCD, orders, secondary survey, second orders.
History (from patient and family/friends): ask for the empty pill bottles to confirm the drug (this may be a pass criterion), how
many pills, when taken, concurrent ingestions of alcohol or other drugs. Where was the patient found? Was there a period of
unconsciousness, how long did this last? Other symptoms (visual blurring, seizure). Did the patient give any warning of the

24

attempt (note, phone calls, giving away possessions), was there a preceding depression or strange behavior, problems at work
or with a relationship. Previous attempts? Medications, drugs, alcoholism, smoking, allergies, past medical and psychiatric
history, family history, review of systems.
Investigations: CBC, lytes, urea, creatinine, glucose, INR/PTT, ABG, CK, serum osmolality, ALP, AST, ALT, total bili, GGT,
Toxicology Screen (ASA, acetaminophen, TCA level, barbiturates, benzodiazepines, EtOH), ECG. CXR (for aspiration
pneumonia).
Treatment: Gastric lavage (rare) if less than 1 hour since ingestion. Activated charcoal 10 g/g drug ingested or 1 g/kg body
weight NG. Hydrate with normal saline to promote diuresis for excretion of TCA and possible myoglobinuria (occurs due to
muscle breakdown following prolonged coma lying on a hard surface). Alkalinize with 1 amp bicarb IV (or 1-2 mEq/kg) and
hyperventilation if the patient is intubated. Follow ABGs or venous gases, aim for pH 7.45-7.55. Remaining treatment is
symptomatic: treat seizures with lorazepam 2 mg IV, treat cardiac dysrhythmias, hypotension, agitation and coma as they arise.
Consult ICU for 24 hours minimum monitoring. QRS > 0.1s indicates increased risk of seizures and dysrhythmias. Psychiatric
consult after patient is medically cleared.
TCA Toxicity:
Therapeutic levels are 2-4 mg/kg. Life-threatening symptoms usually occur at levels > 10 mg/kg.
Anticholinergic effects: hyperthermia, tachycardia, mydriasis (dilated pupils), decreased sweating and secretions, vasodilation,
constipation, urinary retention, ileus. CNS effects are generalized seizures, myoclonus, ataxia, hyperreflexia, confusion,
agitation, hallucinations, acute psychosis, decreased level of consciousness, respiratory depression (mnemonic: Hot as a hare,
blind as a bat, dry as a bone, red as a beet, mad as a hatter, the bowel and bladder lose their tone and the heart goes on alone).
Quinidine effects: conduction delay (prolongation of QRS, PR, QT, T wave flattening), heart block, bradycardia, asystole,
ventricular dysrhythmias and resultant hypotension.
35. 16 year old female in hospital for ASA overdose. Medically cleared. Take a history.
History: patient name, age, occupation. Circumstances surrounding the attempted suicide. Preceding conflicts at work or with
family or in a relationship. Recent loss of employment or loved one. Warning signs: suicide note, giving away prized
possessions. Describe the attempt, how many pills taken, what kinds, concurrent alcohol or drug use. Did the patient really
want to die or was the attempt a cry for help? Gauge the lethality of the attempt in terms of the means used and the chances of
discovery. Previous attempts, describe these. Is patient now actively suicidal or remorseful. If the patient actively suicidal,
what is the current plan? Medications, drug/alcohol use, allergies, past medical history, family history (esp. psychiatric), social
supports: review of systems.
Psychiatry: History for depression/mania, mental status exam, multiaxial diagnosis as in question #4 above.
36. Father with 3 year old child who is not speaking. Take a history. Findings: not speaking well, recurrent ear
infections, poor hearing. Make a diagnosis.
History: of not speaking should determine whether the problem is primary (never spoke) or secondary (stopped speaking).
Secondary causes of mutism are psychological upset (due to family discord, etc.) and rare inherited neurodegenerative
conditions. Primary mutism may be part of a global developmental delay or related to hearing problems which are either
congenital (inherited, intrauterine infections), ototoxic drugs (e.g. streptomycin) or trauma.
Pregnancy and birth history: GTPAL (number of gestations, term pregnancies, premature births, abortions, live children),
history of previous pregnancies including neonatal jaundice, maternal medical history, illness during pregnancy, rubella
(teratogenic), toxoplasmosis (from cats, infects fetal brain), herpes (infects fetus, frequently fatal), CMV (damages fetal liver),
teratogenic medications taken during pregnancy, drug and alcohol use, family history of deafness or late speaking. Newborn
history: gestational age at birth, caesarean, induction, rupture of membranes artificial or prolonged, fetal distress, forceps or
vacuum delivery, meconium, APGARs, was resuscitation required? Initial blood work, breast feeding? How often and how
well, color of 1st stool, color of urine, vomiting, neonate muscle tone, behaviors, fever, irritability, lethargy.
Developmental history (from parent): See question #9. Growth: expected height and weight for age? Speech, has child ever
spoken words or phrases, are these used appropriately, has the child made sounds, chronology and description of these. How
does the child communicate if not through speech? Gross Motor: when did the child start walking, running. Fine motor: when
did you notice pincer grasp, turning pages in a book. Social behavior.

25

Hearing: does the child wake up in response to sounds? Startle to loud sounds? Come when called? Understand spoken
instructions? History of ear infections, wax problems. Ask about swimming. Past medical history, medications, allergies,
family history, review of systems.
Diagnosis: given recurrent otitis media with poor hearing the most likely diagnosis is retarded speech development due to poor
hearing. Refer to ENT for hearing tests, possible tubes (tympanic drainage).
37. 40 year old female with fatigue. Take a history. Findings: cold intolerance, weight gain. Make a diagnosis.
Fatigue history: onset, chronology, past episodes, functional limitations, associated with exertion? Recent viral illness
(mononucleosis), cold intolerance, weight gain, dry skin, brittle hair, hoarseness (hypothyroidism), associated muscle aches
(fibromyalgia), chest pain (angina), shortness of breath (congestive heart failure). Sleep history: usual requirements,
chronology of sleep problems, stressor, sleep hygiene (when, where, regularity, shifts at work, quiet, late, exercise, meals,
alcohol, caffeine, prescription and non-prescription remedies, drugs and medications), sleep latency (time to fall asleep),
nocturnal awakening, early morning wakening, daytime somnolence, somnolence while driving, working or during
conversation. Depression screener: as in question #4 above. Must fully explore suicidal ideation: does patient intend to harm
self, reason for suicidal thoughts, current plan, lethality of plan, access to lethal means, has patient given away prize
possessions or written final notes to loved ones, previous attempts. Medications (especially TCAs, sedatives,
antihypertensives), allergies, drug/alcohol use, smoking, past medical history, family history, review of systems.
Differential Diagnosis: Chronic fatigue, CHF, ischemia, thyroid disease, sleep disturbance, depression.
Diagnosis: most likely hypothyroidism given cold intolerance and weight gain.
38. 35 year old male with back pain and stiffness. Take history and perform a focused physical exam. Findings: 10cm
separation between lumbar spines while erect increases by less than 5 cm when back is flexed forward (positive WrightSchober test), lateral flexion impaired. Q: Give the diagnosis and two associated conditions.
History and Physical: see question #15.
Diagnosis: based on typical history of back pain, lumbar spine X-rays showing fusion of the sacroiliac joints or sacroiliitis and
syndesmophytes (disk spaces undergoing fusion), elevated ESR and HLA-B27 tissue antigen positive this is probably
ankylosing spondylitis.
Associated conditions: inflammatory arthritis, uveitis, psoriasis, IBD, amyloidosis, radiculopathy, pericarditis, angina,
conduction abnormalities.
Treatment: No cure. Regular therapeutic exercise to prevent deformity/disability (esp. swimming and back extension
exercises). To control pain and stiffness: indomethacin (100 mg PO OD), naproxen (250 mg PO BID-TID), etc. Surgery
helpful in severe cases: e.g. total joint replacement.
39. 35 year old male with 1 year history of back and leg pain. Do a physical exam. Findings: right foot dorsiflexion
weakness with dorsal foot numbness. Make a diagnosis and identify the level of lesion.
History and Physical exam: see question #15.
Diagnosis: L4,5 disk herniation with L5 radiculopathy.
40. 47 year old male with impotence. Wants a pill for this. Fears losing his girlfriend. Take a history and counsel.
Background: The causes of erectile dysfunction are subdivided in to psychic and organic categories. Current literature favors
an approximately 1:1 ratio of organic to psychic etiology. The problem is rarely primary (never had ability to sustain erection).
The organic causes are: drugs (-blockers, thiazides, H2 blockers, antidepressants, antipsychotics, Digoxin, clofibrate,
sedatives, alcohol, heroin), hormonal (diabetes, gonadal dysfunction, prolactin-secreting pituitary tumor, associated with loss of
libido and testicular atrophy), neurogenic (stoke, MS, spinal cord injury), iatrogenic (drugs, surgery, radiation), vascular
(peripheral vascular disease, DM, HTN). Impotence is grossly defined as inability to have satisfactory intercourse due to
erectile dysfunction in at least 25% of encounters.
History: Current partners. Problems in these relationships. Why is the patient seeking medical attention for this now? Onset
of erectile dysfunction and chronology. Description of the problem: no erection at all, cannot sustain erection, ejaculate too

26

quickly to satisfy partner, cannot achieve orgasm or orgasm without ejaculation, retrograde ejaculation. Circumstances under
which impotence occurs: only with certain partners, only at certain times or locations, what percentage of the time? Is
impotence related to lack of sexual desire? Presence and firmness of morning or nocturnal erections. Does the patient sustain
erections in masturbation? Associated problems: anxiety attacks, anhedonia/depression, perineal or peripheral numbness, poor
peripheral circulation. Exercise, medications, contraceptive use, drug/alcohol use, smoking, cholesterol, allergies, past medical
history, family history, review of systems.
Counseling: Discuss causes of impotence in terms of organic vs. inorganic etiology and that it tends to cause great anxiety
(normalize patients feelings). Erectile dysfunction can often be improved with lifestyle changes: exercise, weight loss,
improved diet, decreased alcohol intake, smoking cessation, stress management, sleep hygiene, better diabetic control, and
joint counseling with partner to decrease anxiety. Improvement of patients relationship with partner: address sexual boredom.
Review medications: suggest changes. Explain that many organic causes of impotence are unfortunately not reversible.
Describe therapeutic options: counseling with partner on alternative means of sexual gratification, testosterone preparations or
bromocryptine (for prolactinoma) if patient is shown to have hormonal disturbance on blood work (measure testosterone and
gonadotropins), sildenafil (viagra), yohimbine and trazodone preparations for impotence (poorly effective and expensive),
penile self-injection with phentolamine, papavarine & PGE1 or PGE1 alone (30 gauge needle, last 30-60 min., quite popular),
vacuum-rubber ring device, penile prostheses. Arrange follow up with both partners.
41. Young woman with tunnel vision. Negative investigations by a neurologist and ophthalmologist. Take a history.
Findings: concerned that her husband is having an extramarital affair. Counsel.
History: Description of visual problem, functional limitations, onset duration, chronology, aggravating/relieving factors,
associated headache, eye pain, nausea, anxiety, palpitations, tremor. Previous episodes of eye problems or other unusual
phenomena (blindness, paralysis, numbness, abdominal pain). What doctors has patient seen, what did they say? Problems at
work, home, with relationships. Who can the patient go to for support in her life? Psychiatric problems in the past? Depression
screener and sleep history as in questions #3 and #19. Medications, drug and alcohol use, allergies, past medical history, family
history, review of systems.
Conversion disorder: psychic perturbation presents as one or two neurological complaints affecting voluntary motor or
sensory function. Psychological factors thought to be etiologically related to the symptom as the initiation of symptoms is
preceded by conflicts or other stressors. La belle indifference patients inappropriately cavalier attitude towards a serious
symptom. Treatment: anxiolytics (e.g. lorazepam 1 mg PO q6h), relaxation therapy, counseling, close follow-up.
Counseling: Normalize this problem and validate the patients feelings: the fact that several specialists have said there is
nothing wrong with the patients vision does not mean that there is not a subtle medical problem which may become apparent
later. For this reason it is important to stick with one doctor who knows the patient well and can coordinate further referrals if
necessary. Many people who are faced with the possibility of marital infidelity automatically activate a defense mechanism
which gives them time to adjust, and which is not under conscious control: Such a reaction also helps the patient to enlist
needed support from others. This is a normal reaction for these people. These symptoms can vary widely from paralysis, to
numbness, to pains, to inability to speak, and visual problems including blindness and tunnel vision. Outline a plan for
management: address possible sources of anxiety and stress: a frank discussion with the patients husband about fidelity is
required and may be done privately or if both partners are willing, in consultation with you or a marital therapist. The patient
should try to consolidate a support network: parents, friends, etc. Consider depression, sleep or anxiety medications as
appropriate. Arrange follow up with both partners.
42. 30 year old male from another city. Wants Fiorinal prescription for chronic headaches renewed. Manage.
See question #16.
43. 23 year old female with 24 hour abdominal pain. Perform focused physical exam. Findings: peritoneal signs, point
tenderness at McBurneys point. Q: Give a differential diagnosis, order investigations. What further history would
help confirm diagnosis?
Physical exam for abdominal pain: see question #29.
Rectal: rectal shelf, check for gross or occult blood. Also include a pelvic manual and speculum exam. (wont be asked to
perform this at the LMCC II). Check for pain with cervical motion (seen in PID), pain on palpation of ovaries, mass, cervical
discharge. Take swabs (see investigations).

27

Differential diagnosis: appendicitis, ovarian cyst, rupture or ovarian torsion, mittelschmerz, ectopic pregnancy (lifethreatening), hepatitis, cholecystitis, gastroenteritis, peptic ulcer, pelvic inflammatory disease (PID), urinary tract infection
(UTI), pyelonephritis, kidney stone, inflammatory bowel disease, intestinal obstruction due to volvulus or IBD.
Investigations: ABX 3 views, abdominal-pelvic ultrasound, CBC, lytes, urea, creatinine, INR/PTT, glucose, hCG. Urinalysis.
Stool for occult blood. Cervical swabs for culture and pap smear. If OR is imminent order type and cross for 2 units, CXR.
Helpful further history: a history of gradual onset vague periumbilical or LLQ pain migrating to a sharper, more localized
pain in the RLQ over several hours associated with anorexia, nausea, and controlled by still fetal posture suggests appendicitis.
44. 62 year old female with left lower quadrant pain. Perform physical exam. Findings: low grade fever, some
abdominal distention, LLQ tenderness without rigidity, poorly defined left lower quadrant mass. Q: Abdominal series
shows multiple air/fluid levels. Describe. Give differential diagnosis with most likely diagnosis. Order further
investigations.
Physical exam for abdominal pain: see question #29 and #43.
Differential diagnosis: diverticulitis, diverticular abscess, constipation with obstruction, GI malignancy with perforation,
gallstone ileus, obstruction due to volvulus (usually RLQ pain), Crohns, mesenteric ischemia or infarct, ovarian tumor, PID,
uterine perforation.
Most likely diagnosis: diverticulitis.
Investigations: abdominal-pelvic CT (ultrasound if CT unavailable), stool for occult blood, urinalysis, cervical swabs and pap,
CBC, lytes, urea, creatinine, INR/PTT, glucose, for possible preop: ECG, CXR, type and cross 4 units.
45. 24 year old female with left lower quadrant abdominal pain who has an IUD. Perform physical exam. Findings:
signs of peritoneal irritation. Q: Give a differential diagnosis, order investigations.
Physical exam for abdominal pain: see question #29 and #43.
Differential diagnosis: uterine perforation by IUD, pelvic inflammatory disease (PID more common with IUD), ovarian cyst
with torsion or rupture, tubo-ovarian abscess, ectopic pregnancy (also more common with IUD), gastroenteritis, inflammatory
bowel disease, intestinal obstruction due to volvulus or IBD, appendicitis.
Investigations: ABX 3 views, abdominal-pelvic ultrasound, CBC, lytes, urea, creatinine, INR/PTT, glucose, hCG.
Urinalysis. Stool for occult blood. Cervical swabs for culture and pap smear. If OR is imminent order type and cross for 2
units. CXR.
46. 20 year old female at 36 weeks gestation with hypertension and proteinuria. Counsel patient on etiology and risks to
fetus and mother. Q: Outline a plan for management.
Background: Hypertension may be pregnancy induced or pre-existing hypertension can be worsened by pregnancy. Preeclampsia is pregnancy-induced or worsened hypertension (systolic BP increased by 30 mmHg, and diastolic by 15mmHg over
non-pregnant/T1 pressures) with renal impairment (proteinuria > +1 on dipstick or > 300 mg/24 hour collection) and/or nondependent edema (e.g. face, hands) onset > 20 weeks. Eclampsia is pre-eclampsia with CNS involvement, usually decreased
level of consciousness and seizures. Other end organs may be affected, particularly the liver and placenta. May progress to
death through multi-organ failure. Imbalance of thromboxane (vasoconstrictor) and prostaglandin (vasodilator) causes
arteriolar constriction capillary damage protein extravasation and hemorrhage. Both pre-eclampsia and eclampsia fall
under the new heading of pregnancy-induced hypertension (PIH).
Mild PIH: no CNS involvement or criteria for sever PIH.
Severe PIH: 2 of the following:
1. BP > 160/110
2. pulmonary edema or cyanosis
3. CHF
4. proteinuria/oliguria/ Cr
5. LFTs/RUQ or epigastric pain

28

6.
7.
8.
9.

visual disturbances/hyperreflexia/clonus/headache/convulsions
N&V
thrombocytopenia/microangiopathic hemolysis
IUGR

Eclampsia: grand mal seizures in woman with preeclampsia.


Counseling: You have pre-eclampsia. Define as above. Condition is common: 5% of pregnant population, more common in
first pregnancies. Cause seems to be imbalance of thromboxane (vasoconstrictor) and prostaglandin (vasodilator) which causes
arteriolar constriction capillary damage protein extravasation and hemorrhage. Severity varies. Risks for mother: end
organ dysfunction (kidneys, liver), cerebral hemorrhage (50% of deaths), left ventricular failure/pulmonary edema, loss of
pregnancy due to abruption (often with DIC), seizures, HELLP syndrome, death. Risks for fetus: malnutrition, hypoxia,
incomplete maturation, which can result in low birth weight (IUGR), prematurity, death. Overall treatment strategy is to slow
progression of hypertension until the baby can be delivered. Delivery is curative.
Management plan:
Initial exam and investigations: vitals, body weight, examine for edema, check for RUQ tenderness, reflexes, CBC, lytes,
creatinine, urea, uric acid, urinalysis with microscopy, 24 hour urine protein/creatinine clearance, LFTs, INR/PTT. Fetal HR,
non-stress test, biophysical profile (ultrasound with 5 criteria), Doppler flows. Bed rest in left lateral decubitus position (to
reduce abdominal vessel compression), normal dietary salt/protein intake. No use of diuretics/antihypertensives.
Follow up: daily BP, daily weight, daily reflexes, fetal movement counts at home (if patient lives reasonably close to a hospital
and can get transportation fast), frequent follow-up visits for blood work, urinalysis and fetal monitoring, bed rest (preferably
left side). Instruct patient on worsening signs: rapid weight gain, liver pain, visual disturbance, persistent headache, drowsiness
or seizures.
Delivery: early hospital admission (at 36 weeks) for close monitoring and administration of IV MgSO4 4 g over 20 minutes, if
signs of CNS involvement are present (hyperreflexia, decreased LOC, seizure). Possible induction of early delivery or
Cesarean section. Consider IV hydralazine (first line treatment) 5-10 mg IV over 5 minutes q15-30min, second line: labetolol
20-50 mg IV q10min, third line: nifedipine 10-20 mg PO q20-60min if delivery is not imminent or if delivery is imminent to
decrease BP after conservative measures tried (diuretics and salt or fluid restriction not useful and may be harmful).
47. 30 year old woman with vaginal bleeding at 30 weeks gestation. Take a history. Q: Give a differential diagnosis.
Order investigations.
History: patient ID (name, age, occupation), GTPAL, weeks of gestation. Onset of bleeding, duration, estimate quantity
(number of pads soaked), color and consistency of blood, associated fever, chills, abdominal discomfort (pain), contractions,
fetal movement, lightheadedness, last sexual intercourse (may cause spotting due to friable cervix). Problems with previous
pregnancies, problems in this pregnancy, medical visits to this point, investigations done. Associated abdominal trauma
(accident or abuse), drug use (cocaine), father and mothers blood type, medications, alcohol, smoking, past medical history,
family history, review of systems.
Differential diagnosis: placenta previa (placenta covers internal os of cervix the most common cause of painless bleeding in
the third trimester), bloody show (shedding of cervical mucous plug), abruptio placenta (separation of placenta from uterine
wall usually painful), vasa previa (fetal bleed due to root vessels of umbilical cord overlying the cervical os extremely
dangerous to the fetus). Other causes: uterine rupture, coagulopathy (DIC), molar gestation, vaginal tear, vaginal infection,
cervical polyp, cervicitis, cervical cancer, placenta accreta, bleeding from bladder or bowel. (NOTE: no vaginal exam until
previa has been ruled out by U/S).
Investigations: maternal vitals, CBC, INR/PTT, fibrinogen, type and cross-match if bleeding is severe, Rh status (may need
Rhogam gamma globulin to prevent formation of antibodies against fetal blood if mother is Rh negative and father is Rh
positive), fetal monitor, fetal ultrasound, maternal monitoring, IV access, pelvic speculum and manual exam with digital
cervical exam (do these only after ultrasound to rule out placental previa can cause further bleeding in previa), Apt test for
fetal hemoglobin in vaginal blood, test maternal blood for presence and amount of fetal hemoglobin (determines amount of
Rhogam required to neutralize fetal blood antigenicity).
48. 31 year old female with right hand numbness and weakness. Take a history and perform focused physical exam. Q:
Differential diagnosis, investigations and treatment.

29

History: name, age occupation, amount of work done with hands, description of symptoms, onset, duration, chronology, time
of day, aggravating and relieving factors. Previous episodes, investigations. Ask about pain at night, difficulty turning a key or
opening jars (signs of weakness). Functional limitations. Associated injury, neck pain, numbness or weakness in other areas,
visual problems, headache, nausea. Medications, drug/alcohol use, smoking, allergies, past medical history (especially DM,
hypothyroidism, rheumatoid arthritis, pregnancy), surgical history, family history, review of systems.
Physical exam: see question #30.
Differential diagnosis: carpal tunnel compression of median nerve, cervical radiculopathy, stroke/TIA, diabetic peripheral
neuropathy, brachial plexus injury or tumor.
Investigations: nerve conduction studies.
Treatment: modify manual work, wrist splint (often worn at night), NSAIDs, local corticosteroid injections, control underlying
systemic contributors (e.g. diabetes, hypothyroidism, arthritis), surgical decompression via flexor retinaculum release
(Neurosurgery or Plastic surgery consult).
49. 53 year old female with incidental solitary lung nodule on CXR. Take a history. Findings: non-smoker, textile
worker, no symptoms. Q: CXR shows homogeneous round 2 cm opacification in right upper lobe. Give a differential
diagnosis and identify the most likely diagnosis. Order further investigations.
History: name, age, occupation, living conditions, hobbies, pets, esp. birds, cats, travel history, contact with hazardous
substances (e.g. asbestos). Positive TB skin test. History of pneumonia, TB etc. Malignant symptoms: weight loss, fatigue,
change of bowel habits, anorexia, night pain Smoking. Alcoholism. Lung symptoms: cough, sputum, shortness of
breath/dyspnea, hemoptysis, wheeze, orthopnea, PND, chest wall pain, Medications: drugs/alcohol, allergies, past medical
history, family history, review of systems.
Differential diagnosis: less than 3 cm is more likely to be benign but greater than 3 cm is more likely malignant: neoplasm
(45%) primary = 70%, benign (hamartoma, lipoma) = 15%, metastatic = 10%; infection (53%) TB, histoplasmosis,
coccidiomycosis; other (2%) granuloma (scar tissue from old pneumonia, TB granuloma, histoplasmosis, silicosis, sarcoid),
vascular (A-V malformation, infarct), congenital (cyst), round pneumonia, round atelectasis, loculated effusion. Percentages
are for lesion greater than 3 cm.
Most likely diagnosis: granuloma.
Investigations: old CXR for comparison (if lesion is old and unchanging, interventions are less aggressive, calcification is also
associated with benign lesions such as old granulomas), CT chest with CT guided needle biopsy, sputum for cytology and acidfast staining (TB), TB skin test, bronchoscopy with biopsy and washings if lesion seen, open biopsy or lobectomy.
Algorithm: solitary nodule previous CXR benign or unchanged (repeat in q3-6months for 2 years if unchanged observe,
if changed at any time continue), malignant or changed CT thorax: cancer (stage and treat), calcification (observe), no
diagnosis bronchoscopy or transthoracic needle aspiration still no diagnosis (resect for diagnosis), inflammatory (treat
cause), cancer (stage and treat).
50. Mother with 6 month old child who has diarrhea. Take a history. Finding: recently switched from breast milk to
milk powder formula. Q: Give a differential diagnosis. Recommend treatment.
History: Distinguish between acute diarrhea, chronic diarrhea with or without failure to thrive. Name, age, age of onset and
chronology of diarrhea, consistency, color, quantity and frequency of diarrhea, blood in stool, concurrent illness, vomiting,
fever, anorexia, difficulty breathing, lassitude, dry mouth, eyes, low urine output, illness affecting other children in the family
or adults. Feeds and feeding history (esp. fruit juice). Growth pattern: weight loss? Recent immunization, travel, antibiotics.
Medications, past medical history, allergies, birth history, pregnancy problems, maternal illness during pregnancy, family
history, review of systems.
Differential diagnosis: infection (bacteria: campylobacter, salmonella; antibiotic induced: c. diff colitis; parasitic: giardia; post
infectious: secondary lactase deficiency), intolerant of formula change (component of lactose intolerance), toddlers diarrhea (6
months to 36 months, resolves spontaneously by age 2-4 years lots of juice overwhelms small bowel disaccharide
malabsorption), overfeeding, osmotic diarrhea due to high osmolality liquids such as juice, gastroenteritis. With weight loss

30

consider malabsorption syndrome: celiac disease, lactose intolerance, milk protein allergy, cystic fibrosis (with chest
infections), IBD (unusual at this age), enzyme deficiencies, liver disease, biliary atresia, thyrotoxicosis.
Treatment: try non-milk (soy based) formula. Arrange follow up.
Note: Treatment for Toddlers diarrhea reassurance, self-limiting, 4 fs fiber, normal fluid intake, 35-40% fat, discourage
excess fruit juice.
51. 50 year old male alcoholic. Vomited bright red blood 1 week ago. Perform physical exam. Q: Give differential
diagnosis. What is your treatment plan?
Physical exam: see question #29.
Differential diagnosis: esophagitis, gastritis, duodenal ulcer, peptic ulcer, Mallory-Weiss (partial thickness) tear, esophageal
varices, gastric or esophageal cancer, lung tumor, aorto-enteric fistula (rare can occur after previous aortic surgery).
Treatment plan: place on omeprazole 20 mg PO OD, patient to refrain from alcohol, discontinue NSAIDs, draw blood for
INR/PTT, AST, ALT, ALP, GGT, total bili, CBC, lytes, urea, creatinine. CXR (check for perforation air under diaphragm),
refer to gastroenterology for endoscopy. Consider admission if patient unreliable or transportation is a problem.
52. Telephone rings: hysterical mother says her child swallowed a cleaning agent. Manage over the phone. Q: What do
you do after hanging up the phone? What investigations do you order in the ER?
Over the phone: Establish calm, reassurance, obtain caller name, phone number, address. Childs age, weight, medical
problems, medications, allergies. Identify agent ingested, have caller read hazard label if possible. How much was taken and
when? What is childs present condition? Any other agents taken? Have child drink 2-3 glasses of milk if alert (works for
alkali or acid). Do not induce vomiting (exposes esophagus and pharynx to the corrosive agent again). If amount ingested was
very small, may not need to come in, warn that child may vomit. If amount unknown or significant, child may come to ER
immediately. Mom may drive if calm, otherwise you will send an ambulance. Have child lie on his side in case of vomiting.
After hanging up: send ambulance, call the poison center with the description of the agent and ask for direction as to treatment
etc.
Investigations: CBC, lytes, glucose, urea, creatinine, serum osmolality, serum ketones, ABG, CXR, toxicology screen if
ingested agent unknown.

1994
53. 60 year old male with 20 minutes chest pain. Diaphoretic, ECG shows 2 mm ST elevation in V1, 2, 3. Manage.
History and Physical: see question #13, #14 and #17.
Treatment: Raise head of bed. Give oxygen 6 L/min by mask. Monitor oxygen saturation. Order stat CBC, lytes, glucose,
INR/PTT, serial CK-MB and Troponin, ABG, CXR, ECG. Give chewable ASA 160-325 mg immediately. Secure IV access,
bolus IV lasix 40 mg, push if fluid overload is suspected, and ventolin if wheezes are heard, give sublingual nitro spray or 0.3
mg SL nitro if blood pressure is adequate and 1 mg morphine IV. Repeat nitro SL q5min x 3. May require additional morphine
and nitro. Repeat CK-MB and Troponin q8h x 3.
Diagnosis: severe anterior wall ischemia evolving to infarct. Required thrombolytics or cardiac catheterization. Urgent
Cardiology consult if available.
54. 28 year old male arrives in the ED having fallen 6m from a scaffold. Assess.
Rescusitation: see question #6.
History: if available: Nature of fall, preceding (seizure, etc.) and subsequent events, has patient ever experienced similar
symptoms before, did patient lose consciousness, were there seizure-type phenomena, injuries during fall, duration of
unconsciousness, post-ictal drowsiness, medications and drugs, alcohol/illicit drug involvement, smoking, allergies, past
medical history, family history, review of systems.

31

55. 22 month old female child brought to emergency by her mother with fractured left humerus. Two previous
fractures in past 3 months, bruises seen on forehead. Manage.
Warning signs of child abuse: explanation doesnt match injury, delay in seeking treatment, recent family crisis, injuries of
varied ages/recurrent/multiple injuries, distinctive marks, atypical pattern of injury, unrealistic expectations of child behavior
by caregivers.
Risk factors for child abuse: environmental (social isolation, poverty, domestic violence), caregiver (substance abuse,
jealousy between boyfriend and father, parents abused themselves, personality/character disorder or mental illness, poor social
and vocational skills/below average intelligence), child factors (disability, difficult child i.e. temperament, premature).
History: how did the injury happen? Who was looking after the child when it happened? Who are the childs care givers, and
who lives in the house or comes in contact with the child. How did the child get the bruises? What happened with the other
fractures? Any other injuries in the past? Is the child accident prone or difficult to handle? What is the childs personality:
open vs. withdrawn. Are there other children in the house? Have they had broken bones or other injuries? Was this child a
planned pregnancy, problems with pregnancy, birth history. Developmental milestones as in question #9. Where does the child
live (isolation), income level of parents, problems with the law, alcoholism, drug use, smoking by caregivers or other adults in
the home, what is the typical response of caregivers when the child cries or misbehaves? Were the caregivers abused as
children? Is there spousal abuse, sexual abuse or incest? Has the Childrens Aid Society been involved with this child or other
children? Interview relatives, friends. Childs medical history, medications, allergies, review of systems.
Physical exam: observe childs behavior, constant watchfulness associated with child abuse. Inspect for malnutrition, bruises,
scars, burns, especially on the flexor surfaces. Inspect oral cavity, perineum, anus, genitalia. Ophthalmoscopy for retinal
hemorrhages (shaken baby syndrome). Evaluate for development, neurological exam for possible brain injury.
Investigations: X-rays for old fractures, if records not available, CBC, lytes, urea, creatinine, INR/PTT, albumin
(malnutrition).
Treatment: admit for the childs safety and investigations, consult child psychiatrist and pediatric orthopedic surgeon.
Obligated to report suspicion of child abuse to Childrens Aid Society. Family therapy, frequent follow up to monitor
development.
56. 35 year old male. 1 week hyperactivity, histrionic, spending spree, bizarre behavior. Take a history.
Differential: Causes of one week of bizarre behavior: manic episode (bipolar mood disorder), depression, drug-induced
(steroids, amphetamines, alcohol), organic (hypothyroidism, frontal lobe tumor, MS, dementia), schizophrenia.
History: Description of symptoms: onset, duration, chronology, aggravating and relieving factors (drug use). Is patient a
danger to himself or others (suicidal or homicidal)? Does patient have alternating up and down periods, how long do these last?
How frequent? Are they cycling faster than before? Ask about mood, sleep, interest, guilt, energy, concentration,
appetite/weight, psychomotor, suicide/morbid ideation. Paranoia, ideas of reference (thought broadcasting; special messages,
mind reading), special powers, magical thinking, secret identity, voices, visual or tactile hallucinations. Current life events,
stress, relationship problem, bereavement. Previous psychiatric problems, family history or psychiatric disorders, substance
abuse, relationship problems, problems at work. Work and relationship histories. Ask about hypothyroidism, adrenal
dysfunction, hypercalcemia, mononucleosis. Medications, drugs, alcohol, smoking, allergies, past medical history including
psychiatric history and history of abuse. Family history, review of systems.
Manic episode: Expansive, elevated or irritable mood x 1 week with 3 of following: GSTPAID grandiosity (or inflated self
esteem), sleep (less need for), talkative, pleasurable activities (with painful consequences), activity increased (goal directed or
psychomotor), ideas (flight of), distractibility. Not mixed episode. Severe enough to cause psychotic features/impaired
social/occupational functioning. Not substance abuse or GMC.
Diagnosis of major depression: see question #4.
Mental status: see question #4.
57. 22 year old male. Hears voices. Take a history.

32

History as in question #56 and #4 with special attention to the chronicity of symptoms, and work, school, and relationship
histories.
DSM IV Criteria for the diagnosis of schizophrenia:
A: 2 or more of the following characteristic symptoms occurring for a significant portion of a 1 month period: delusions,
hallucinations, disorganized speech, catatonic or grossly disorganized behavior, negative symptoms (flat affect, alogia
poverty of speech, avolition, anhedonia, apathy, affectional impairment). Only 1 symptom if delusions are bizarre or
hallucinations consist of a voice keeping running commentary or 2 voices conversing.
B: Social or occupational dysfunction.
C: Continuous sign of some disturbance for at least 6 months including the month of more severe disturbance in A.
D: Schizoaffective or mood disorder excluded.
E: Substance abuse or general medical disorder excluded.
F: If there is a history of autistic disorder or pervasive developmental disorder, then the diagnosis of schizophrenia is made
only if delusions or hallucinations are prominent for at least 1 month.
Note that schizophrenic symptoms < 1 month = brief psychotic disorder, < 6 months = schizophreniform disorder, > 6 months
= schizophrenia.
58. 16 year old male with three episodes of sudden loss of awareness lasting < 1 minute, wants information on epilepsy.
Counsel.
History: see question #3.
Counseling: Cause of seizures disturbed electrical activity in the brain, often with a tiny focus of abnormal tissue from
previous infection, trauma (including birth trauma) or inherited. About 2% of the population have epilepsy. Your seizures
appear to be absence or petit-mal, which start in young people. One third of cases resolve spontaneously with age. Seizures do
not damage the brain unless they are prolonged (> 30 minutes) and absence seizures are not associated with decreased
intelligence or learning ability. Most people with this type of epilepsy are well controlled on medication and have no
limitations in their activities, careers or relationships.
Further counseling and treatment: see question #3.
59. Young man in the emergency department with a stab wound to the belly. Manage.
See question #6.
60. 18 year old female wants oral contraceptive. Manage.
See question #2.
61. 35 year old woman feels depressed. Manage.
See question #4.
62. 5 year old girl with pain on urination. Mother concerned about UTI and sexual abuse. Order investigations and
counsel.
Algorithm: Possible UTI Is immediate antimicrobial therapy indicated? (e.g. infant, toxic, dehydrated) if yes (urine
specimen for culture via SPA/catheter initiate Abx therapy [TMP/SMX or cephalosporins i.e. cefixime, cefprozil]: consider
hospital if toxic, vomiting and cannot take PO meds or if < 6 months); if no (U/A on specimen bag U/A positive for LE,
nitrites, WBC? no? unlikely UTI if no symptoms yes? Obtain urine for culture SPA/catheter) is the culture
positive? no? (no UTI) yes? (7-14 days of Abx; prophylaxis [TMP/SMX or nitrofurantoin] until imaging completed)
clinical response in 48 hrs? no? (immediate urinary tract U/S) yes? (U/S as soon as convenient) ultimately VCUG
as soon as convenient.
Investigations: midstream clean catch voided urine specimen (or bag urine). May need to catheterize or aspirate
suprapubically (SPA) to obtain a good specimen. Urine dipstick. Microscopy, culture and sensitivity most common
pathogen is E. coli serotypes from bowel flora others include Klebsiella, Proteus, enterococci, S. saprophyticus. If child

33

appears systemically ill, take blood for cultures, CBC, urea, creatinine, lytes. Renal ultrasound for major malformations and
voiding cystourethrogram (VCUG) should be done in all children 2 months to 2 years old with UTI. Radiological
investigations may be postponed until the second UTI in girls over 2 years old due to higher rate of benign UTI. Postpone the
VCUG 3-6 weeks to allow normalization of flow after UTI.
Counseling: Ask why the mother is concerned about sexual abuse. UTI alone is not a good indicator. Explain that, in girls
UTIs are common because of short urethra and proximity to anal area. Describe front to back wiping after urination, and
general hygiene. Give prescription for TMP/SMX (Septra) 2/10 mg/kg/d PO OD or Nitrofurantoin 2 mg/kg/d PO OD x 7-10
days if urine dip is positive for white cells and patient not allergic. Arrange follow up in > 2 weeks for re-culture of urine.
Explain need for ultrasound and VCUG to rule out flow abnormalities which may threaten kidney function (this may be
postponed until second UTI in girls over the age of 2 due to increased incidence of benign UTI), arrange these. Consider
hospitalization for pyelonephritis, rehydration or child < 6 months old.
63. Elderly lady on Digoxin and Lasix with syncopal attacks. Take a history. Q: Give a differential diagnosis. What
investigations would you order?
History: onset, duration, chronology, description of events. Have the patients episodes been witnessed? Does the patient lose
consciousness, are there warning signs of post-ictal symptoms, can the patient prevent an episode by sitting down or other
means. If dizziness is a feature, is this light-headedness or true vertigo (vertigo means that the patient senses actual movement
of either the room or themselves). What is the patient doing when these episodes occur? Are muscle jerks a feature?
Associated fatigue, weakness, nausea/vomiting, chest pain, shortness of breath, palpitations, focal neurological symptoms.
Past medical history (e.g. diabetes, heart disease), medications, drug use, alcohol, smoking, allergies, family history, review of
systems.
Differential diagnosis: Medication induced bradycardia (Digoxin), hypovolemia (Lasix). Cardiovascular arrhythmia, valvular
disease, subclavian steal. Metabolic: hypoglycemia. Central nervous system: seizures (e.g. narcolepsy, tumor), stroke/TIA,
cervical spondylosis, anxiety with hyperventilation, middle ear (benign positional vertigo, acoustic neuroma, Menieres
disease). Autonomic: vagal, orthostatic hypotension. Infection in elderly can present in many ways: UTI etc.
Digoxin overdose: anorexia, nausea, vomiting, bradycardia, visual effects: yellow, green or white halo around objects,
decreased level of consciousness, abdominal pain and diarrhea. ECG shows junctional tachycardia, PVCs, AV block, and
sometimes PSVT.
Physical exam should include: vitals, orthostatic BP, check for signs of dehydration (thirst, mucous membrane moistness, HR,
urine output, skin turgor, BP), cardiopulmonary exam (see question #13), neurologic exam (see question #5) and mini-mental
status exam (see question #4).
Investigations: Digoxin level, CBC, lytes, urea, creatinine, INR/PTT, glucose, ECG, 24 hour Holter monitor, echocardiogram,
EEG, CT head, carotid Doppler.
64. 30 year old male, married with 2 children. Brought in by police for violent and dangerous behavior. Take a history.
Q: Would you admit this patient? What are the criteria for a Form 1?
History: Attempt to determine whether patient is sad (depressed), bad (antisocial, reaction to stressful or frustrating events,
poor anger management), or mad (mania, schizophrenia). Is the episode related to drugs of abuse or organic (brain tumor,
metabolic disturbance)? Cover history for depression (see question #4), mania (see question #4) and schizophrenia (see
question #57) with mental status exam (see question #4) and mini mental (see question #4).
Criteria for admission: patient requires observation or medication in a controlled, safe setting for diagnosis, patient appears to
be a danger to himself or others, environment at home unsuitable for the patient at this time, patient requires medical work up
for organic causes, patient in need of detoxification.
Criteria for a Form 1: Forcible admission for assessment without right to appeal, maximum 72 hours, can be administered by
any licensed physician who has seen the patient within a week, both criteria must be met:
1. Patient appears to be danger to himself/herself or others.
2. Patient appears to be currently suffering from a mental illness.
65. 65 year old male with dysphagia. Take a history. Q: Differential diagnosis and investigations.
History: see question #10.

34

Investigations: Barium swallow (liquid and with marshmallow for transfer), endoscopy with biopsy, CT chest, esophageal
manometry, 24 hour pH reflux study.
66. 65 year old male outpatient with shortness of breath, cough, sputum. Take a history and perform a physical exam.
Findings: Lobar consolidation with yellow-green sputum. Q: Given a diagnosis of pneumonia, recommend treatment.
History: Name, age, occupation, travel history, pets. Cough (acute, chronic, worse in any position/season/night, anything
relieve), sputum (what color, quantity, frequency, quality), hemoptysis (quantity, frequency, quality e.g. blood tinged/clots),
dyspnea (constant, duration, onset, frequency, severity, exercise tolerance, triggers, alleviating), fever, chills, malaise, fatigue,
increase in asthmatic symptoms (wheeze, cough), preceding viral illness. Onset, chronology of symptoms, positional factors
(orthopnea), chest pain, ankle swelling. History of COPD? Medications, compliance with meds (observe use of puffers), drugs
of abuse (alcohol), smoking, allergies, past medical history, family history, review of systems.
Physical Exam: Cardiopulmonary exam as in question #13 and #24.
Treatment: for diagnosis of community-acquired pneumonia, admit if patient is systemically ill (may have septicemia), if the
patient is debilitated or hypoxia is a feature (send blood cultures and give oxygen). Start IV cefuroxime 750 mg IV q8h.
Switch to a more specific oral antibiotic when culture results become available. For outpatient therapy: Clavulin (amoxicillin +
clavulanate) 500/125 mg PO BID. Follow up in 1 week. Discontinue therapy after 3 afebrile days.
Note: this therapy does not cover atypicals. Practices vary according to the treatment population:
Antibiotic therapy for pneumonia
Presentation
Community acquired, no COPD
Community acquired with COPD
Alcoholics & debilitated patients
Diabetics or hospital acquired

Likely Organisms
Streptococcus pneumonia (typical) or Mycoplasma pneumoniae & chlamydia (atypical)
add Haemophilus influenza
add Gram negatives, Legionella and anaerobes (in aspiration)
add Staphylococcus aureus and in very ill patients: Pseudomonas aeruginosa

Therapies:
1. IV cefuroxime 750mg IV q8h covers: Strep, Staph (unless it is MRSA), gram negatives, anaerobes. Change to more
specific oral agent when organism known.
2. Clavulin (amoxicillin + clavulanate) 500/125mg PO BID. Covers Strep, Staph, gram negatives and anaerobes.
3. Septra (trimethoprim + sulphamethoxizole) 2 tabs PO BID (or 1 DS tab BID) covers Strep, Staph and gram negatives.
4. Erythromycin 500mg PO/IV QID covers Strep and atypicals.
5. Penicillin G 1-2 million units IV q4h effectively covers Streptococcus only. Oral version is penicillin V 250-500mg PO
q6h.
6. Piperacillin 3g IV q6h + Tobramycin 2mg/kg IV q8h is a standard therapy for Pseudomonas aeruginosa. Piperacillin also
covers Strep, gram negatives and anaerobes, while Tobramycin adds further gram negative coverage with synergy.
7. Clindamycin 300mg IV q8h or PO q6h covers Strep, Staph and anaerobes.
8. Vancomycin 1g IV q12h or 125mg PO q6h for MRSA also covers Strep and anaerobes.
Sanford Guide: Rx influenced by local prevalences.
Presumed viral pneumonia in
adults: cought, no sputum,
dyspnea/hypoxia, interstitial
infiltrates
Adults over age 18: Community
acquired; non-hospitalized.

Influenza, parainfluenza, adenovirus, RSV,


hantavirsu
Smokers: S. pneumo, H. influenzae, Moraxella
catarhalis. Post-viral bronchitis: S. pneumo,
rarely S. aureus. No co-morbidity: Mycoplasma,
Chlamydia pneumoniae, viral, rarely S. pneumo.
Alcoholic stupor: S. pneumo, anaerobes,
coliforms. Epidemic: Legionaires. Birds:
Psittacosis. Rabbits: Tularemia. Parturient
livestock or cats: Coxiella burnetii. Airway
obstruction: Anaerobes.

For influenza A or B: zanamivir 10mg


inhaled BID x 5d or oseltamivir 75mg PO
BID x 5d. Start within 48 hours of
symptoms onset.
Azithro 0.5gm PO x1 then 0.25 gm/d or
clarithro 500mg PO BID or clarithro ER
1gm PO OD.

For influenza A: rimantadine


100mg PO 2x/d or amantadine
100mg PO 2x/d.

FQ with enhanced activity vs. S.


pneumo or O Ceph 2 or AM/CL
875/125mg PO BID or doxy 100mg
PO BID or telithro 800mg PO OD
Rx duration varies: rx until afebrile 3-5d.
x7-10d.
Adults over age 18: Community
Non-ICU bed: [P Ceph 3 IV (erythro 15In ICU: (P Ceph 3 IV + azithro
acquired; hospitalized. (Note:
20 mg/kg/d q6h or azithro 500mg IV OD)]
500mg IV OD) or (FQ with
severe S. pneumo in postor (cefuroxime + erythro) or (FQ with
enchanced activity vs. S. pneumo);
splenectomy, myeloma,
enhanced activity vs. S. pneumo)
add P Ceph 3 for dual coverage if
lymphoma pts.)
suspect Gm-neg enteric.
O Ceph 2: cefdinir 300mg PO q12h, cefpodoxime proxetil 200mg PO q12h, cefprozil 500mg PO q12h, cefuroxime axetil 250-500mg PO q12h.
P Ceph 3: cefotaxime 2gm q4-8h IV, ceftriaxone 1-2gm IV OD, ceftrizoxime not as active in vitro vs. S. pneumo.

67. Young female with malaise, tender lymph nodes in the neck, left upper quadrant abdominal pain. Perform a
physical exam. Q: Give a differential diagnosis.

35

Physical exam: see question #29 and #43. Also: Palpate for lymph nodes in the neck, supra and infra-clavicular, axillae, groin.
Examine the oral cavity and pharynx. Check for rashes.
Differential diagnosis: neoplastic: lymphoma, leukemia. Viral infection: mononucleosis, HIV, EBV. Bacterial: syphilis.
Inflammatory autoimmune disease: sarcoidosis, lupus. Liver disease with portal hypertension. Serum sickness, allergic
reaction.
68. 65 year old man with ataxia, dizziness, macrocytic anemia. Take a history. Finding: poor diet. Q: Give a
differential diagnosis. What is the most likely diagnosis? What investigations would you order?
History: onset, chronology, description of symptoms. Setting in which symptoms occur. Functional limitations (driving,
walking, stairs, reaching upward). Differentiate light-headedness from true vertigo (room or self spinning). Peripheral
numbness, psychiatric features: mild depression, irritability, paranoia (seen in B12 deficiency). Weakness, eye symptoms,
tremor. TIA or stoke phenomena: sudden neurologic deficit (loss of vision, speech, motor or sensory changes). Check for
heart problems, hypertension, diabetes. CAGE alcoholism screen as below, history of syphilis, MS, hypothyroidism (decreases
secretion of intrinsic factor) use of chemotherapeutic agents (interfere with DNA synthesis). Diet, weight loss or gain, chronic
diarrhea (malabsorption), abdominal pain. Symptoms of hypothyroidism. Previous gastric surgery. Signs of intracranial
hypertension (hydrocephalus): morning nausea, vomiting, headache. General signs of malignancy: anorexia, fatigue, night
sweats. Past medical history, medications, drugs/alcohol, smoking, allergies, family history, review of systems.
CAGE questionnaire: Control have you tried to cut down on your alcohol? Anger have you ever felt angry when someone
suggested you decrease your alcohol intake? Guilty have you ever felt guilty about your drinking? Eye opener do you
sometimes have a drink to get started in the morning?
Differential diagnosis: Anemia due to vitamin B12 deficiency: usually due to malabsorption (Crohns, celiac disease etc.),
lack of intrinsic factor pernicious anemia (auto-antibodies against gastric parietal cells associated with thyroid and
adrenal insufficiency), post gastrectomy, long term use of antacids, pancreatic insufficiency or malnutrition (vegan diet).
Severe hypothyroidism. Due to Folate (B6) deficiency: Poor nutrition (alcoholism, poverty, infancy found in green leafy
vegetables), malabsorption, medication or drug-induced (alcohol, anticonvulsants, antifolates e.g. MTX, BCP), increased need
(pregnancy, prematurity, hemolysis, hemodialysis, psoriasis, exfoliative dermatitis). Ataxia, dizziness: Wernickes
encephalopathy, hepatic encephalopathy, inner ear problem (benign positional vertigo, acoustic neuroma, Menieres), postural
hypotension, brainstem stoke or TIA, intracranial tumor.
Most likely diagnosis: Vitamin B12/folate deficiency secondary to poor diet.
Investigations: CBC and differential with blood smear, lytes, urea, creatinine, INR/PTT, GGT, AST, ALT, ALP, serum folate,
screen for serum B12, RBC folate, serum ferritin. Schillings test: measures absorption of B12. Bone marrow Barium enema if
suspect pernicious anemia (pernicious anemia is associated with bowel cancers).
69. 42 year old man found unconscious in the street. Appears to have been struck in the head. Perform physical exam.
Findings: GCS 11, unilateral body weakness. Q: What is your differential? Evaluate C-spine film. Describe your
initial treatment and investigations.
Initial management: rescusitate as required using the ATLS/ACLS format (see question #6).
History: if available: Nature of collapse, preceding and subsequent events e.g. trauma, has patient ever experienced similar
symptoms before, did patient lose consciousness, were there seizure-type phenomena, injuries during fall, duration of
unconsciousness, post-ictal drowsiness, medications and drugs, smoking, allergies, past medical history, family history, review
of systems.
Differential diagnosis: concussion, subdural bleed, epidural bleed, brain contusion, seizure or post-ictal weakness, brainstem
or spinal cord injury.
Clearing C-spines: The principle of clearing C-spines is to rule out both bony fractures and ligamentous injury, either of which
can make the spine dangerously unstable. Most emergency physicians will clear the cervical spine in the case of an alert
patient who has no pain on palpation of the dorsal spinous processes and a normal cross-table lateral C-spine X-ray. If the
patient has neck pain, flexion/extension plane films are done. These involve gently flexing, then extending the neck and taking
views at each extreme. The patient must be alert enough to warn the examiner of paresthesias in the hands or increased neck

36

pain on movement during this procedure, which may indicate compromise of the neural elements. Flexion/extension views
may be done under fluoroscopy if the patient is not alert.
Treatment: normalize vitals, oxygen saturation, ABGs, hydrate to maintain BP, give blood if necessary, correct coagulopathy.
Immobilize cervical spine. Consider intubation (careful of the neck). Control ICP, load with dilantin 1g IV to prevent seizures,
give 20% mannitol 50g IV, rapid sequence intubate with sux, fentanyl, etomidate/propofol/ketamine, spray cords with lidocaine
and hyperventilate to pCO2 35mmHg. Consult Neurosurgery. CT head and neck. May need MRI for delineation of brainstem
or spinal cord injuries.
70. 4 year old boy with cough for 6 weeks. No improvement on antibiotics 3 weeks ago. Take a history. Q: What is
your differential diagnosis? Give the most likely diagnosis and describe a treatment plan.
History: Name, age. Prodromal illness, fever, malaise, rhinorrhea, sore throat, shortness of breath, wheeze. Is cough
productive? Color of sputum, quantity. Any chest pain? Aggravating and relieving factors. Onset of cough, chronology, time
of day (night), worse with exposure to polluted air, cold air, dust, smoke, exercise, in grassy areas, weedy areas, forests, certain
rooms, in bed. Did symptoms improve with Amoxicillin? Allergic symptoms: red eyes, itching, itch in back of throat. Family
pets. Air conditioning. Type of bedclothes and pillows (feather, synthetic, foam). Recent change in the childs environment,
different bedclothes, new room, change of season. Are there smokers in the house? Past medical history, medications,
allergies, family history (asthma, allergy, CF), review of systems.
Differential diagnosis: asthma, bronchitis, bronchiolitis (upper respiratory tract infection), chronic sinusitis, rhinitis, TB,
recurrent pneumonia, collapsed lung, cystic fibrosis.
Approach to cough:
1. Productive: bronchiectasis, bronchitis, abscess, bacterial pneumonia, TB
2. Nonproductive: viral infections, interstitial lung disease, anxiety, allergy
3. Wheezy: suggests bronchospasm, asthma, allergy
4. Nocturnal: asthma, CHF, postnasal drip, GERD, or aspiration
5. Barking: epiglottal disease (croup)
6. Positional: abscess, tumor
7. Differential diagnosis:
a. Airway irritants: inhaled smoke, dusts, fumes, aspiration (gastric contents, oral secretions, foreign body),
postnasal drip
b. Airway disease: URTI including postnasal drip and sinusitis, acute or chronic bronchitis, bronchiectasis,
neoplasm, external compression by node or mass lesion, asthma, COPD
c. Parenchymal disease: pneumonia, lung abscess, interstitial lung disease
d. CHF
e. Drug-induced
Investigations: CXR, CBC, lytes, INR/PTT, urea, creatinine, pulmonary function tests (> 4 years of age).
Most likely diagnosis: asthma.
Treatment: Acute: O2 to keep SpO2 > 92%. Fluids. 2-agonists: Salbutamol (Ventolin) 0.03 cc/kg in 3 cc NS q20min via mask
until improvement then q1hour as necessary. Ipratropium bromide (Atrovent) if severe: 1 cc added to each of first 3 Ventolin
masks. Steroids: prednisone 2 mg/kg in ER, then 1 mg/kg PO OD x 4 days. Chronic: Sodium cromoglycate (Intal) 2 puffs
QID. Add ventolin 2 puffs PRN if acute breathlessness with wheeze is a feature. Modify the home environment to decrease
contact with common allergens: dust mites, pollen, pet hair (especially cats). Control dust with thorough and regular cleaning.
Boil bedclothes, plastic undercovers on mattress and pillows, remove rugs, install air conditioner. Remove pets. No smoking
in the house (second hand smoke is a cause of childhood asthma). Warn parents of the symptoms of a severe asthma attack,
status asthmaticus and when to come to the ER. Discuss treatment strategy, a regular anti-immune medication (sodium
cromoglycate or inhaled steroid) with PRN bronchodilator.
71. A 60 year old woman with a history of atrial fibrillation and congestive heart failure returns to the office for the
results of her Digoxin level, which is subtherapeutic. Take a history and counsel. Q: What are the effects of a Digoxin
overdose?
History: Name, age, occupation. Ask about symptoms of atrial fibrillation: sudden fatigue, palpitations, general weakness,
light-headedness, CNS embolic phenomena (TIA, stroke). Symptoms of Digoxin overdose which the patient may have had in
the past: anorexia, nausea, vomiting, bradycardia, visual effects: yellow, green or white halo around objects, decreased level of

37

consciousness. Symptoms of congestive heart failure: ankle edema, shortness of breath, orthopnea, paroxysmal nocturnal
dyspnea. Cough, wheeze, hemoptysis. Is patient taking her Digoxin regularly, if not, why? Medications and whether patient is
taking these, drugs and alcohol, smoking, allergies, past medical history, family history, review of systems.
Counseling: explain action of Digoxin: prevents fibrillation by slowing the heart rate while increasing its force of contraction.
Helps both atrial fibrillation and congestive heart failure. Must be taken regularly. Discuss reasons why patient may not have
been taking medication. Memory problem? Concern about the side effects? Feels she no longer needs the medication?
Emphasize the need to discuss these concerns with you before altering medications. Discuss symptoms of overdose which the
patient might watch for. Consider alternative medication if side effects are a problem. Develop a follow-up plan to monitor
compliance and cardiovascular status. Arrange follow-up.
Effects of Digoxin overdose: anorexia, nausea, vomiting, bradycardia, visual effects: yellow, green or white halo around
objects, decreased level of consciousness.
72. 60 year old woman with hypercalcemia on an insurance physical. Take a history and perform a physical exam.
Finding: sister had a parathyroid adenoma. Q: Give a differential diagnosis for hypercalcemia. What investigations
would you order?
History: Symptoms of hypercalcemia: Bones, Stones, psychosis-based Moans, and abdominal Groans fatigue, muscle
weakness, arthralgias, renal colic due to nephrolithiasis, emotional lability (can progress to psychosis and coma), bone pain,
abdominal pain, nausea, vomiting, constipation, ileus, polyuria, polydipsia, nocturia. Onset, duration of these. General
malignancy symptoms: weight loss, night sweats, fatigue. Orthostatic hypotension (Addisons). Heat intolerance,
hyperactivity (hyperthyroid). Diet, especially amount of milk and use of calcium supplements and antacids. Medications,
drugs and alcohol, smoking, allergies, past medical history (especially heart burn, reflux, gastritis, peptic ulcer), family history
(especially of multiple endocrine neoplasia MEN), review of systems.
Physical exam: Trousseaus sign (inflate BP cuff, leave on 1-2 minutes, distal arm goes into titanic flexion, indicates
hypercalcemia). Inspect for signs of Addisons: bronze skin tone, orthostatic hypotension, or of Cushings: moon facies, striae,
buffalo hump. Chest: palpate sternum and ribs for bone pain, examine breasts for signs of malignancy: dimpling, mass.
General cardiopulmonary exam. Abdominal exam, palpate liver carefully for masses, percuss kidneys for pain. Rectal: test
stool for occult blood. Examine long bones for straightness and tenderness (Pagets disease of bone).
Differential diagnosis: parathyroid adenoma (hyperparathyroidism) due to solitary adenoma, hyperplasia, carcinoma, inherited
MEN (multiple endocrine neoplasia may also have pituitary adenoma causing Addisons or Cushings), malignancy
(myeloma, lung, breast, squamous in any site), high bone turnover (Pagets disease of bone, hyperthyroidism, Vit A excess),
vitamin D in pharmacologic doses, milk-alkali syndrome (large ingestion of milk and alkali, usually for gastric hyperacidity),
aluminum intoxication, drugs (thiazides, lithium, CaCO3), familial hypocalciuric hypercalcemia.
Investigations: CBC, lytes (including Ca & Ca++, Mg, PO4), urea, creatinine, albumin, AST, ALT, ALP, GGT, INR/PTT, serum
cortisol, serum PTH, TSH, serum protein electrophoresis (for monoclonal gammopathy of myeloma). Plane X-rays of tender
or malformed bones, including skull (see salt and pepper lesions). CT head, thyroid, adrenals.

1993
73. 50 year old woman states that the Russians are leaking radiation into her house. Take a history with mental status
exam.
History should focus on depression, mania and schizophrenia. Mental status exam. See questions #4 and #57.
74. Obese patient wants help with weight loss. Counsel.
History: Name, age, occupation, weight history, note ups and downs if present, past attempts to lose weight, successes,
obstacles, goals. Dietary habits: frequency of meals, snacking, eating at night, foods eaten, binge eating, guilt about food,
hoarding, concealing eating from others. Estimate daily caloric intake. Alcohol intake. Smoking, drug use. Exercise history.
Overweight relatives? Diseases associated with weight gain: hypothyroidism, DM type II, Cushings, major depression, anxiety
disorder, some medications (TCA, steroids, OCP). Problems associated with overweight: gout, sleep apnea, cholecystitis, back
pain, cardiovascular disease, hemorrhoids, lower limb joint pain and osteoarthritis. Why is patient seeking medical help for
this now? Crisis in patients life, stress, anxiety? Assess patients self-image: does patient feel underweight, overweight or
normal? Does patient feel that weight interferes with health? With activities? Screen for eating disorders. Medications, drugs

38

and alcohol, allergies, past medical history, family history, review of systems (include sleep habits, apneic spells, OTC
medications).
Physical exam: vitals, calculate BMI (body mass index) = weight (kg)/height 2 (m2). Ideal is 20-25 for a male, but varies
depending on frame. 25-30: overweight; 30-35: obese; > 40: morbidly obese. Use standard height-weight tables for males and
females to gauge percent overweight (>20% = obese). Direct physical to pertinent positives from history. Inspect for
Cushingoid features: moon facies, buffalo hump, striae, visual field defects. Fat distribution: centripetal fat associated with
greater heart disease and diabetes risk. Fundoscopic exam for retinopathy. Cardiopulmonary exam, abdominal exam (not liver
size). Check for signs of hypothyroidism: hypothermia, goiter, dull facial expression, lid lag absent, lids droop, periorbital
swelling, hair is sparse, coarse and dry, skin is also coarse, dry scaly and thick. Patients are forgetful, show change in
personality which may progress to psychosis, deep tendon reflexes show slow return phase (brisk contraction and slow
relaxation), leukonychia (whitened nails), orange palms and soles due to carotene deposition, bradycardia, pericardial effusion,
pleural effusions, myxedema, non-pitting edema, carpal and tarsal tunnel nerve compression due to myxedema, causing
paresthesias and numbness in the hands and feet.
Counseling: Motivation: how would being at ideal body weight improve the patients life? Emphasize health, lifestyle, self
esteem, relationship benefits. Discuss nutrition-related problems: heart disease, obesity, hypertension, osteoporosis, anemia,
dental decay, cancer, gastrointestinal disorders, respiratory compromise, high lipids, diabetes, sleep apnea, osteoarthritis.
Discuss diets tried and why these failed. Fad diets involve unusual or extreme eating patterns and are not designed to be
maintained for a lifetime therefore these should be discouraged. Weight loss agent Ponderal no longer available. SSRIs such
as Paxil may assist with weight loss, unfortunately, when the drug is discontinued, most people regain weight. Explain that the
brain has a satiety set point which can be reset over time with reduction in caloric intake. Warn that the bodys ability to
determine caloric content is very good, and will not be fooled by so-called diet products. Recommend a balanced diet
consisting of ordinary foods, with three distinct meals per day of small size. No eating at night and be careful of snacks.
Inform patient that he will be hungry for at least the first two weeks of reduced intake. Suggest visualization techniques,
redirection of interests, and to think of hunger as a sign of positive progress on weight loss. Group support can be beneficial
too: Weight watchers, overeaters anonymous etc. behavior modification.
Dietary recommendations: reduce fat to 20% of caloric intake. Ideal caloric intake can be estimated at 8-10 Cal/lbs (ideal
weight) for females and 10-12 Cal/lbs (ideal weight) in males. Emphasize that caloric intake is more important for weight loss
than food composition (i.e. excessive calories lead to weight gain even if they are non-fat).
Exercise recommendations: sudden intense exercise in sedentary patient unwise. Exercise tends to provide a good excuse for
overeating and may be dangerous in an obese patient. Recommend mild daily exercise such as 1 hour walking per day. More
vigorous exercise can be initiated when weight is lost.
Arrange regular follow-up for body mass monitoring and counseling.
75. Alcoholic smoker with cough, sputum, shortness of breath. History and physical exam. Order investigations.
See question #66.
76. 70 year old man with dysphagia. Manage.
See question #65.
77. 64 year old woman with resting tremor. Perform focused physical exam.
Hallmarks of Parkinsons disease: TRAP: Tremor (rest, pill-rolling, 4-7 Hz, can be suppressed by voluntary movement),
Rigidity (lead pipe and cog-wheeling), Akinesia/bradykinesia, Postural instability (festinating gait, retropulsion, falls).
Remember to explain to the examiner what signs of Parkinsons you are looking for at each step in the exam. Other features
include: mask-like face, lack of blinking, blepharoclonus (fluttering of closed eyelids), dysphagia, drooling, hypophonia,
micrographia, gait: start hesitation, small shuffling steps, loss of arm swing, subcortical dementia (apathy, forgetful, poor
ability to use knowledge).
Physical exam: vitals, observe patient at rest: look for pill-rolling tremor in the upper limb which is worst at rest, may also
have head tremor (titubation), stooped posture, open-mouthed, mask-like face, generally hypokinetic with decreased blinking
and drooling. Mini mental status: dementia associated with Parkinsons (50% of patients), may find poor short term memory,
poor concentration, abstraction, micrographia. Depression may also be a feature, screen for depression: MSIGECAPS. Cranial

39

nerves, body power, pronator drift, deep tendon reflexes and Babinski are normal in Parkinsons. Tone: bilateral lead pipe
(constant) rigidity with possible cog-wheeling due to tremor superimposed on passive motion. Test elbow, forearm rotation
and knee by applying rapid passive motion while feeling the muscle tendon. Cerebellar testing: finger-nose and heel-shin tests
show improvement of tremor with intention (i.e. a resting tremor rather than an intention tremor), rapid alternating movements
are poor bilaterally in Parkinsons and Rhomberg is positive due to postural instability. The Parkinsonian gait is unsteady and
shuffling with small steps, decreased arm swinging, and a tendency to fall forward or backward. Patients may try to increase
forward speed to keep from falling (festination).
Mini-Mental Status: orientation to time/place (5 pts, year, season, month, day, day of week; 5 pts, country, province, city,
hospital, floor), memory (3 pts, honesty, tulip, black; 3 pts, delayed recall), attention/concentration (5 pts, serial 7s, WORLD
backwards), language tests: comprehension (3 pts, three point command), reading (1 pt, close your eyes), writing (1 pt,
complete sentence), repetition (1 pt, no ifs, ands or buts), naming (2 pts, watch, pen), spatial ability (1 pt, intersecting
pentagons)
78. 12 year old female with fever, photophobia, neck stiffness. Manage.
History: name, age, onset of symptoms, duration, increasing or decreasing in severity, fever, nausea, vomiting, photophobia,
phonophobia, neck stiffness, headache, rash. Immunocompromised? (HIV, asplenia, prematurity), parameningeal infection?
(sinusitis, mastoiditis), environmental risk factors? (day-care centers, household contact, travel to endemic regions).
Medications or other interventions tried? History, severity, chronology of migraines. Premonitory visual disturbance? Recent
neurosurgery, head trauma, other illness? Contacts with meningitis at school or work. Medications, drugs/alcohol, allergies,
past medical history, family history, review of systems.
Physical exam: vitals, GCS, note general appearance of patient if patient is very ill, a resuscitation approach (ATLS/ACLS)
may be appropriate, orientation. Inspect for meningococcemial rash. Cranial nerves: pupillary reflexes, note photophobia if
present, extraocular muscle movement, check for double vision, visual fields, facial sensation and movement, gross hearing,
sternocleidomastoid and trapezius power. Tone: passive rapid movement at elbows, rotation of forearms and flexion/extension
of knees. Pronator drift. Cerebellar testing: finger-nose, heel-shin, rapid alternating movements of forearms. Power of
deltoids, triceps, biceps, wrist extension and flexion, finger abduction and adduction, psoas, quadriceps, hamstrings, ankle
dorsiflexion and plantar flexion. Deep tendon reflexes at triceps, biceps, brachioradialis, knee, ankle, Babinski. Light touch,
pin prick over limbs and body, vibration sense at joints. Signs of meningismus: Kernigs: pain in the neck on extension of the
knee with the hip in 90 degrees of flexion. Brudzinskis: pain on passive flexion of the neck. Nuchal rigidity. Opisthotonos:
spasm in which head and heels are bent backward and body bowed forward.
Investigations: CBC, lytes, INR/PTT, urea, creatinine, glucose, blood cultures, ABG, CT head followed by lumbar puncture if
history and physical are suspicious for raised intracranial pressure (lumbar puncture may, rarely, precipitate brain herniation in
the presence of raised ICP).
Treatment: isolation, initiate IV antibiotics immediately (before CT and LP) if the clinical picture is suspicious for meningitis.
Cefotaxime 2 g IV q4h + ampicillin 50 mg/kg IV q6h. Consult ICU. Consider intubation and intensive management of ICP.
Monitor glucose, acid-base & volume status and manage as needed. Steroids for Hib meningitis (give early). Anticonvulsants
for seizures. Report to public health.
79. 17 year old female with chronic diarrhea. Take a history.
Background: Causes of chronic diarrhea in a 17 year old: Crohns, ulcerative colitis, irritable bowel, malabsorption (Celiac
disease, tropical sprue), lactose intolerance, intestinal infection (C. difficile, giardia, amoebiasis), pancreatic dysfunction,
unusual but may be laxative abuse.
History: onset of diarrhea, duration, consistency and color of stools, do they float? Is there blood or mucous? Frequency of
BMs/day, weight loss, appetite. Dietary history. Is diarrhea worse with milk (lactose intolerance tends to produce explosive
diarrhea after milk ingestion hereditary)? Laxative use. Use of antibiotics in the past 6 weeks. Travel history. Fever,
nausea, vomiting, infectious illnesses. Associated abdominal pain, fatigue, uveitis, mouth or anal ulcers, ankylosing
spondylitis, sacroiliitis, renal problems (due to malabsorption), arthritis (these are associated with Crohns). Malnutrition
signs/symptoms: lassitude, weakness, hair falling out, skin rash, easy bruising, weight loss, anemia, neurologic findings (carpal
and tarsal nerve compression, confusion, emotional lability, loss of corticospinal vibration and position sense), glossitis.
Medications, drugs and alcohol, smoking (decreases risk and symptoms of inflammatory bowel disease), past medical history
(IBD, abdominal surgery), family history, review of systems.

40

Classification of Chronic Diarrhea


Type
Inflammatory
Ulcerative colitis
Crohns disease
Malignancy: lymphoma, adenocarcinoma
Osmotic
Ingestion
Lactose intolerance
Medications, laxatives
Maldigestion and Malabsorption
Pancreatic insufficiency
Bile salt deficiency
Celiac Sprue
Whipples disease
Bowel resection
Secretory
Bacterial enterotoxins
Secretagogues VIP, gastrin, carcinoid
Functional
IBS

Characteristics
Fever, hematochezia, abdominal pain; usually weight loss with carcinoma

Stool volume decreases with fasting


Increased stool osmotic gap:
fecal [Na+] + [K+] < serum osmolality 25 mmol/L
Weight loss, fecal fat > 7-10 g/24h stool collection
Anemia, hypoalbuminemia

Large volume (> 1 L/d); little change with fasting


Normal stool osmotic gap:
Secretory: fecal [Na+] + [K+] = serum osmolality

80. Young mother with black eye, hit by her boyfriend. Manage.
Warning signs of domestic violence: obsessive need to control victim by controlling money, restrictions on going out, not
allowed to see certain people. The Spanish Inquisition where were you, who were you with, what did you do? Social
isolation. Threats. Verbal abuse aimed at decreasing self-esteem of victim. Cycle of violence followed by remorse, then
increased violence.
Risk factors for domestic violence: social isolation, poverty, substance abuse, partners parents had abuse relationship,
personality/character disorder or mental illness.
History: Describe violent episode, what triggered it? Were objects used as weapons? Injuries? Was the boyfriend remorseful
afterward? History of previous episodes of violence or loss of temper by boyfriend. What is patients response? Has patient
been in an abuse relationship before? Were the patients parents in an abusive relationship? Is boyfriend controlling, does he
restrict her activities, question her excessively after she has been out, engage in verbal abuse or threats? Is the violence
increasing in severity? Are there children in the house? Who are the biological parents? Ask about violence to the children,
sexual abuse. Does the patient or her partner abuse alcohol or other drugs? Is money a problem? Is the boyfriend willing to
seek help?
Counsel: Explain that the boyfriend hitting the patient is a criminal assault and an example of domestic violence. Domestic
violence tends to increase over time unless the victim leaves, or the abuser and couple seek therapy. Very often, women dont
leave their abusive partner until they are seriously hurt or before they are killed. Domestic violence between adult partners
tends to be reflected in future behavior of children who are exposed to it and there is a risk of violence to the children. Child
abuse is a criminal act and if suspected, is reportable to police by law. Spousal abuse is also a criminal act but is not reportable
by law. Recommend that the patient not return to the abuser if there is risk to her safety (e.g. not the first assault, abuser not
remorseful). If the patient does return, an exit plan should be developed to ensure patient safety. Document all evidence of
abuse (pictures, sketches) and related visits; quote patient directly in chart.
Alternatively, the patient can contact the police to obtain a restraining order on the abuser. Develop a plan with the patient to
seek alternate living arrangements (womens abuse shelter), enlist the help of patients support structure (friends, other family
members), contact the police (patient should be informed that, if contacted, the police will lay charges whether the patient
wants to or not). Counsel patient on how to enter into controlled, safe, contact with the abuser to discuss possible therapy for
anger management and controlling behaviors, with therapy as a couple for relationship problems. Social worker referral.
Arrange follow up.
81. Patient kicked by horse, now hypotensive in emergency department. Manage.
Rescusitation: The ACLS and ATLS format is useful: see question #6.
History: if available: Where was the patient kicked, when, was there mechanism of action for other injury, i.e. collapse/fall,
preceding and subsequent events, did patient lose consciousness, duration of unconsciousness, did the patient fall, injuries
during fall, medications and drugs, smoking, allergies, past medical history, family history, review of systems.

41

Management: For unstable blood pressure, blood loss is the most likely cause (CBC may be normal with a large acute blood
loss), type and cross for 4-6 units depending on estimated severity and hang blood as soon as possible. Is the patients
abdominal wound the first priority? Examine for head injury, other injuries. Check for abnormalities on abdominal exam
suggestive of splenic rupture. With clear surgical abdomen (rigidity, rebound, absence of bowel sounds), consult general
surgery and prepare patient for immediate OR. If less severe abdominal bleeding is suspected, consider CT abdomen or
diagnostic peritoneal lavage.
82. Suturing station. Suture laceration on a rubber forearm. Choose suture type. Is a tetanus booster required?
Suturing station: past years have included a point for introducing yourself to the rubber forearm.
History: Name, age, occupation, mechanism of injury, environment in which injury occurred. How long since the injury. Any
distal loss of sensation, motor power? Other injuries. Past medical history, medications, drug/alcohol use, smoking, allergies,
review of systems.
Choice of suture: use non-absorbable monofilament such as 3-0 Prolene or Ethylon. Braided sutures can harbor bacteria and
absorbables cause more inflammatory reaction in the skin. Given a choice between 3-0 silk (a braided non-absorbable) and
chromic gut (a braided absorbable), choose silk.
Technique: for small wound use interrupted sutures starting at the middle of the wound. Anesthetize with lidocaine without
epinephrine, cleanse and irrigate wound beforehand and drape, glove and observe sterile technique.
Tetanus immunization status: Dose Td 0.5 mL IM. Usual schedule of immunizations for tetanus (prepared as diphtheriatetanus toxoid plus pertussis vaccine, i.e. DTP) is 2, 4, 6, 18 mo., 4-6 years, Td (diphtheria-tetanus toxoid) at 14-16 years and
repeat q10y.
Last tetanus immunization treatment
0-5 years ago
none
5-10 years ago
boost (Td)
>10 years ago
boost and give immunoglobulin (passive)
uncertain
boost and give immunoglobulin (passive)
Follow up: Warn of signs of wound infection. Remove sutures in 7 days (5 days on the face to minimize scarring, the face
heals faster and is less likely to become infected due to better blood supply). Recommend Tylenol plain if pain is a problem.
83. Female, 7 weeks pregnant with lower abdominal pain and vaginal bleeding. Take a history. Finding: previous
spontaneous abortion at 6 weeks gestation. Q: Give a differential diagnosis, which is most likely? What three findings
on vaginal examination would confirm this diagnosis?
Definitions:
1. Threatened abortion = any uterine bleeding or cramping in the first 20 weeks of gestation.
2. Inevitable abortion = intolerable pain or bleeding x 1 week, cervix open. Life-threatening to the mother.
3. Incomplete abortion = membranes ruptured, part of products of conception passed, cervix open.
4. Complete abortion = uterus empty, bleeding and complete sac and placenta passed, cervix open.
5. Missed abortion = fetal death and retention of products; presents as pregnancy not progressing, cervix closed.
6. Habitual = 3 or more consecutive spontaneous abortions.
20-30% of pregnancies have uterine bleeding or cramping in the first 20 weeks. Half of these abort. Most spontaneous
abortions are associated with abnormal fetus.
History: patient ID (name, age, occupation), GTPAL, weeks of gestation. Onset of bleeding, duration, estimate quantity
(number of pads soaked), color and consistency of blood, associated fever, chills, abdominal discomfort, light headedness.
Problems with previous pregnancies, problems in this pregnancy, medical visits to this point, investigations done. Associated
abdominal trauma (accident or abuse), drug use (cocaine), father and mothers blood type, medications, alcohol, smoking, past
medical history (diabetes, lupus), family history, review of systems.

42

Differential diagnosis: threatened abortion, incomplete abortion, non-uterine bleeding source trauma: post-coital vs. lesion:
cervical polyp, neoplasm etc. (vaginal, cervical, vulvar), abnormal pregnancy (ectopic, molar), physiologic bleeding (due to
placental development).
Most likely diagnosis: incomplete abortion. Three findings which would confirm this: ruptured membranes, products of
conception passed, cervix dilated (os open).
84. 60 year old male with hemoptysis and shortness of breath. Take a history. Findings: history of CAD, HTN, hasnt
been taking antihypertensives for six weeks. Q: describe CXR, (shows enlarged heart, upper lobe vascular
redistribution, Kerley B lines, bilateral interstitial infiltrates and bilateral small effusions). Read ECG: shows Q waves
and inverted T waves in V1-4. What is the diagnosis?
History: name, age, occupation. Onset of symptoms, duration, time of day. Has patient had these before? Smoking history.
Prelude of fatigue, ankle edema, orthopnea, paroxysmal nocturnal dyspnea, palpitations, chest pain/heaviness/tightness, pain in
left arm or jaw/teeth. History of angina, other cardiac problems. History of GI bleeds, reflux, varices, gastritis, peptic ulcer,
COPD history, cough, sputum, wheeze. History of immobilization, leg pain or swelling, previous DVT, PE. Medications has
patient been taking them? Drugs, alcohol, allergies, smoking, surgical history, family history, review of systems.
CXR: consistent with pulmonary edema and CHF.
Diagnosis: ECG: anterior wall myocardial infarct.
85. 2 year old child with 9 week history of cough, on Amoxicillin for 2 weeks. Take a history. Q: Give a differential
diagnosis. What investigations would you order?
See question #70.
86. Young female with secondary amenorrhea for 6 months. Take a history. Q: What investigations would you order.
Give a differential diagnosis. What is the most likely diagnosis, what results would confirm this diagnosis?
History: age of menarche, regularity of previous menses, flow, duration, accompanying cramps, bloating, psychic disturbance.
Headache, visual field disturbance (for sellar tumor). Signs of virilization: increased quantity and coarseness of body hair and
facial hair, acne, increased sexual drive, increased muscle bulk. Galactorrhea? Diet history: has patient lost/gained weight
lately? Thyroid symptoms: energy levels, emotional lability/depression, cold intolerance, or feels hot, jumpiness. Exercise
history: is patient engaging in vigorous exercise such as running? Sexual history: contraception, is pregnancy a possibility?
Medications, drugs and alcohol use, smoking, allergies, past medical history with surgical history, family history, review of
systems.
Approach: History and physical pregnancy test TSH and Prolactin (high/low hypo/hyperthyroid; high, > 100, or
symptoms of hyperprolactinemia CT to rule out tumor) progesterone challenge (+ve withdrawal bleed anovulation;
no withdrawal bleed end-organ failure or outlet obstruction) FSH, LH (high ovarian failure; low hypothalamic
dysfunction)
Investigations: hCG, TSH, serum LH, FSH, serum prolactin, serum testosterone, sex-hormone binding globulin (SHBG),
DHEA-S, progesterone trial for uterine bleeding (indicates functional endometrium).
Differential diagnosis: pregnancy, polycystic ovary syndrome (Stein-Leventhal syndrome), hypothalamic dysfunction,
excessive exercise, stress, weight loss, adrenal dysfunction (e.g. Cushings), thyroid dysfunction, prolactinoma,
hypopituitarism.
Most likely diagnosis: PCOS
Confirmed by: elevated LH, low or normal FSH with well estrogenized vaginal mucosa, increased serum androgens, ovarian
cysts seen on ultrasound.
87. Mother with low birth weight baby, just delivered. Take a history. Q: On physical exam of the baby you find
emaciation with wrinkled yellow skin and yellow tears. What is the problem? Give three underlying causes for this.
Give two potential problems which may arise in the next 48 hours.

43

Mothers obstetrics and newborn history: see question #25.


Causes of intra-uterine growth restriction (IUGR)
1. symmetric IUGR (normal head to body size), familial, maternal gestational infections (mnemonic ToRCH),
toxoplasmosis (carried in cat feces), rubella, cytomegalovirus (CMV), herpes
2. Asymmetric IUGR (small body): placental insufficiency due to maternal malnutrition, smoking, drugs and alcohol,
illness during pregnancy (e.g. Crohns), hypertension.
3. Jaundiced, emaciated baby: hyperbilirubinemia. Three underlying causes: ABO or Rh incompatibility, neonatal liver
insufficiency (CMV) and sepsis (TORCH).
All causes of neonatal jaundice: Unconjugated = physiologic neonatal jaundice OR pathologic: hemolytic ABORh
incompatibility, neonatal sepsis, splenomegaly, hereditary spherocytosis, G6PD etc.; non-hemolytic breast milk jaundice,
breakdown of cephalohematoma, polycythemia, sepsis, Gilberts, Crigler-Najjar, hypothyroidism. Conjugated: GI obstruction
in fetus (increases enterohepatic circulation), bile duct obstruction, drug-induced and multiple other less common causes.
Two potential problems arising in the next 48 hours: kernicterus (hyperbilirubinemic seizures and brain damage: deposition
of bilirubin in the brainstem and basal ganglia leading to mental retardation, cerebral palsy, hearing loss and paralysis of
upward gaze), hydrops fetalis (generalized edema, including pulmonary, with high output heart failure).
88. A young man fell while inebriated and sustained a laceration to the right wrist. Perform a focused physical exam.
Findings: numbness on ulnar side of hand, Allens test shows no ulnar artery refill, FDS impaired in little and ring
finger. Q: What four structures were lacerated? Management?
Physical Exam:
Vascular: capillary refill, is hand pink and warm? Allens test: compress ulnar and radial arteries at the wrist, have patient
open and close hand to remove blood, then release one side the hand should flush pink due to blood supply from the side
released. (Usually used to demonstrate function of ulnar artery). Doppler probe.
Sensory: digital nerves: check two point discrimination on either side of each digit. Pin-prick sensation: Median nerve
territory is the palmar surface of the thumb, and the palmar surface and dorsal tips of the index, middle and thenar side of the
ring fingers. The ulnar side of the hand is ulnar nerve, and the dorsal surface of the thenar side of the hand is radial nerve
innervated. The median nerve also innervates most muscles of the thenar eminence, and the 1st and 2nd lumbricals. The thumb
is weak in abduction at 90 degrees to the plane of the hand in median nerve dysfunction. The ulnar portion of the palm is
supplied by the cutaneous branch of the ulnar nerve, while the thenar portion is supplied by the cutaneous branch of the median
nerve.
Neuromotor examination of the Hand
Median
Sensory
Redial aspect of index finger pad
Motor Extrinsic Flex DIP of index finger (FDP)
Motor Intrinsic

Thumb to ceiling with palm up


(abductor pollicis brevis)

Ulnar
Ulnar aspect of little finger pad
Flex DIP of little finger (FDP,
extensor carpi radialis)
Abduct index finger (first dorsal
interosseous)

Radial
Dorsal webspace of thumb
Extend wrist and thumb
(extensor pollicis longus)

Motor: (dont ask patient to apply force against resistance as this may rupture a partially severed tendon test active ROM
only). Median nerve: thumb abduction. Ulnar nerve: finger spread against resistance. Radial nerve: wrist extension. FDS
flexes the MCP and PIP joints of the fingers, while FDP (flexor digitorum profundus) flexes the DIP. To test FDP function hold
the MCP/PIP joints in extension and ask patient to flex DIPs. Both the FDS and FDP can flex the entire finger, but the FDP
tends to flex them all at once, while the FDS can flex fingers in isolation. To test FDS: hold all fingers except the one you are
testing in extension and ask patient to flex the remaining finger.
Tendon Examination of the Hand
MCP
Extensor tendons
Communis
Flexor tendons
Intrinsics (lumbricals)

PIP
Extensor digitalis (lateral bands)
Flexor digitorum superficialis

DIP
Intrinsics (lateral bands)
Flexor digitorum profundus

Structures superficial to the flexor retinaculum (in order from ulnar to thenar): ulnar nerve, ulnar artery, cutaneous branch of
ulnar nerve, palmaris longus tendon, cutaneous branch of median nerve.

44

Structures immediately deep to flexor retinaculum (ulnar to thenar): 4 flexor digitorum superficialis tendons (FDS), median
nerve, palmaris longus tendon, cutaneous branch of median nerve.
Structures lacerated: given diminished ulnar territory sensation, Allens test shows no refill from ulnar circulation, and FDS
weakness in little finger and ring finger, the following structures were included in the laceration: ulnar nerve, ulnar artery,
flexor retinaculum, ulnar two divisions of FDS.
Treatment: Clean and explore wound under local anesthetic and sterile conditions. Consult plastic surgery for microvascular
repair. If at night, may suture skin closed and arrange for patient to be seen by plastic surgeon the next day.
89. A young man sustains a head injury on falling from his bicycle. Patient has been hemodynamically stabilized.
Perform a focused neurological exam. Q: Lateral skull and lateral C-spine X-rays provided. Are they adequate? Are
they normal? The patient has continuing sanguinous discharge from his nose. What is the likely cause of this.
Treatment?
Begin by assessing the level of consousness using the Glasgow Coma Scale:
Glasgow Coma Scale (GCS):
Glasgow Coma Scale
Eye Opening
(E)
Spontaneous
4
To speech
3
To Pain
2
never
1

Verbal Response
Oriented and converses
Confused conversation
Inappropriate words
Incomprehensible sounds
None

(V)
5
4
3
2
1

Best Motor Response


Obeys commands
Localizes pain
Withdrawal to pain
Abnormal flexion (decorticate)
Abnormal extension (decerebrate)
Nil

(M)
6
5
4
3
2
1

NB: Standard painful stimulus is rubbing the knuckle on the sternum. For withdrawal, apply pressure on the base of the nail
bed with a pen. Decorticate posture is arm flexion with leg extension on the same side of the body, may be unilateral or
bilateral. Indicates a lesion above the brainstem. Decerebrate posture is arm and ipsilateral leg extension, may be unilateral or
bilateral, indicates brainstem involvement. A GCS of 8 or less is considered an indication for intubation because of the risk of
poor protection of the airway from aspiration.
Head and Neck: inspect for lacerations and contusions, cranial nerves: extra-ocular movements, visual fields by confrontation,
pupillary reactivity, dolls eyes (careful of neck, may not be able to turn head enough to do this), and accommodation, corneal
reflex and facial sensation, palpate facial bones for stability, look in nose and ears for blood or CSF leaks, hemotympanum,
facial muscle power, gross hearing, sternocleidomastoid power and trapezius power. Check oral cavity, gag reflex, palpate
dorsal cervical spines for pain and alignment, is the trachea midline? Pronator drift, cerebellar tests: finger-nose, heel shin,
rapid alternating movements (dysdiadocokinesis). Body power, tone, sensation. Deep tendon reflexes, Babinski. Log roll
patient onto back, inspect, palpate spine.
If GCS is significantly less than 15: Head and Neck: inspect for lacerations and contusions, cranial nerves: pupillary
reactivity, dolls eyes (careful of neck, may not be able to turn head enough to do this), corneal reflex, palpate facial bones for
stability, look in nose and ears for blood or CSF leaks, hemotympanum. Check oral cavity, gag reflex, palpate dorsal cervical
spines for alignment, is the trachea midline? Brainstem (breathing pattern). DRE for sphincter tone. Deep tendon reflexes,
Babinski. Log roll patient onto back, inspect, palpate spine.
Differential diagnosis: concussion, subdural bleed, epidural bleed, brain contusion, seizure or post-ictal weakness, brainstem
or spinal cord injury.
Clearing C-spines: The principle of clearing C-spines is to rule out both bony fractures and ligamentous injury, either of which
can make the spine dangerously unstable. Most emergency physicians will clear the cervical spine in the case of an alert
patient who has no pain on palpation of the dorsal spinous processes and a normal cross-table lateral C-spine X-ray. If the
patient has neck pain, flexion/extension plane films are done. These involve gently flexing, then extending the neck and taking
views at each extreme. The patient must be alert enough to warn the examiner of paresthesias in the hands or increased neck
pain on movement during this procedure, which may indicate compromise of the neural elements. Flexion/extension views
may be done under fluoroscopy if the patient is not alert.

45

Treatment: Continuous sanguinous nasal discharge after head injury is likely a leak of CSF mixed with blood. This represents
a break of the meninges and can, rarely, lead to meningitis (< 5% of cases). Prophylactic antibiotics not indicated as this
selects for resistant organisms. If meningitis results, it is usually due to less virulent organisms than in other settings. > 90%
of leaks resolve spontaneously within 4 weeks. If leak does not subside spontaneously, a surgical repair may be necessary.
Consult neurosurgery. CT head and neck. May need MRI for delineation of brainstem or spinal cord injuries.
Key Points:
1. Never do lumbar puncture in head injury.
2. All patients with head injury have C-spine injury until proven otherwise.
3. Dont blame coma on alcohol: there may also be a hematoma.
4. Low BP after head injury means injury elsewhere.
5. Must clear c-spine both radiologically AND clinically.
90. Older man with 50 RBC/HPF on routine urinalysis. Take a focused history. Q: What is the likely diagnosis? Give
three other possible diagnoses. What two investigations would you order?
History: see question #28.
Likely diagnosis: benign prostatic hyperplasia.
Differential diagnosis: transitional cell carcinoma of bladder, UTI, nephrolithiasis, hydronephrosis, prostatitis, prostate cancer,
renal cell carcinoma, essential hematuria (tends to occur in children).
Two investigations: prostate specific antigen (PSA), cystoscopy, renal, bladder and prostate ultrasound, intravenous pyelogram
(IVP).
91. 20 year old female with hypertension. Perform a physical exam. Q: Give a differential diagnosis. What
investigations would you order?
See question #11.
92. 67 year old male complains of bladder distension, inability to urinate and dribbling of urine from the urethra. Take
a history. Q: What investigations would you order?
History: patient ID. Onset of symptoms, chronology, previous episodes. Associated constipation, perineal numbness, leg
weakness, diabetic neuropathy. Is patient on a new medication? Suprapubic pain, pain on urination, frank blood in the urine,
color of urine, difficulty initiating or maintaining urinary stream, renal pain, groin pain. Previous renal colic or diagnosis of
nephrolithiasis? Known prostatic hypertrophy or cancer? Diabetes? B12 deficiency? Recent surgery? History of
hypercalcemia, hypertension. Malignant symptoms: night sweats, weight loss, fatigue. Medications, drugs/alcohol, smoking,
past medical history, past surgical history, family history, review of systems.
Investigations: urinalysis, urine microscopy and culture with sensitivities, cystoscopy, PSA, renal and pelvic ultrasound.

1992
93. 60 year old female feeling depressed. Complains of stomach pain. Perform focused mental status exam.
Mental status: appearance, behavior (dress, grooming, posture, gait, apparent age, physical health, body habitus, expressions,
attitude - cooperative?, psychomotor activity, attention, eye contact), speech (rate, rhythm/fluency, volume, tone, quantity,
spontaneity, articulation), mood (subjective emotional state in patients own words), affect (Quality euthymic, depressed,
elevated, anxious; Range full, restricted; Stability fixed, labile; Appropriateness; Intensity - flat, blunted), suicidal ideation
(low, intermediate, high poor correlation between clinical impression of suicide risk and probability of attempt), thought
process (coherent, flight of ideas, tangentiality, circumstantiality, thought blocking, neologisms, clanging, perseveration, word
salad, echolalia), thought content (delusions bizarre vs. non-bizarre, obsessions, preoccupations, phobias, recurrent themes),
perceptual disturbances (illusions, hallucinations, depersonalization, derealization), insight, cognition, judgment.
94. 16 year old girl brought to the office by a classmate for weight loss over the past six months. The classmate is
worried about anorexia nervosa. Take a history and counsel.

46

History: amount of weight lost, time frame. How did the patient lose the weight? What is the patients diet now? Still losing
weight? How often does the patient weigh herself? Are you proud of this weight loss? Do you think you need to lose more?
Are you afraid of becoming fat? Do you admire women who are smaller than you? Binge eating, post-prandial vomiting,
laxative or diuretic abuse, excessive exercise, diet pills. Wearing baggy clothes to conceal fatness, unable to look at self in a
mirror or to be touched by others. Ask about the home environment, is there a problem with expressing conflict openly? Signs
of malnutrition, amenorrhea (> 3 consecutive menstrual cycles missed), sallow skin, rash, easy bruising, dry and sparse hair,
lassitude, weakness, anemia, neurologic findings (carpal and tarsal nerve compression, confusion, emotional lability, loss of
corticospinal vibration and position sense), glossitis, heart burn, teeth erosion, GI bleeding.
Counsel: Determine ideal body weight using standard height/weight charts (BMI = weight (kg)/height 2 (m2), ideal is about 2025 for females). Show patient her position on the chart. Explain that anorexia nervosa is a modern disease of highly motivated
young women. These women exercise extreme control over their bodies, often as a means of sublimating their inability to
express conflict at home. Warn patient that excessive weight loss has led to the deaths of many young women who were
unable to correct their anorexia. Explain that proper body weight is essential for health and mental function, including learning
and performing well at school/career. You understand that the patient may be proud of her weight loss. Being underweight
may show a great deal of self control and will power, but being at ideal weight shows more. Invite patient to develop a healthy
body image by not equating soft or fatty body areas with overweight. Emphasize that attractiveness and good health depend on
a good balance of fatty tissues as well as lean. Contract with the patient to gain a certain number of pounds per week. Discuss
how she will do this.
Contract for specific weight gain goals (2 lbs/week). Involve dietician. Halt diuretics, laxatives, diet pills. Close monitoring
of weight, vitals, heart rhythm, potassium. Arrange follow up with patient and her family to discuss family dynamics,
expression of conflict in the home.
95. 2 year old child with history of fever and 1 seizure. Counsel parents.
See also question #9.
Most likely diagnosis: benign febrile seizure (febrile seizures usually occur 6 months to 6 years, associated with initial rapid
rise in temperature, no neurologic abnormalities/evidence of CNS infection/inflammation before or after, no history of nonfebrile seizures, most commonly generalized tonic-clonic, < 15 minutes duration, no recurrence in 24 hours, atypical may show
focal origin/> 15 minutes/> 1/24 hours/transient neurologic defect).
Counsel parents regarding febrile seizure: A typical febrile seizure is a brief generalized tonic-clonic seizure related to high
fever (at least 39 degrees Celsius) and occurring between the ages of 3 months and 7 years. The post-ictal stage is
characterized by improvement in confusion, lethargy, limpness. The greatest risk factor for febrile seizures is a history of
febrile seizures in the parents. This is the most common seizure in children (3-5% of children, M > F). Occur between the ages
of 6 months and 6 years. Thought to be due to initial rapid rise in temperature. These seizures may come about as a result of
fever from any cause, including post immunization. In the absence of an abnormal developmental history (CP, developmental
delay), and an otherwise well child, they are usually benign. Seizures do not cause mental impairment unless they are
prolonged (> 30 min) but can be a symptom of brain damage.
Prognosis after a single febrile seizure: 65% will never have another seizure, 30% will have further febrile seizures, 3% will
go on to have seizures without fever, and 2% will develop lifelong epilepsy (risk factors for this are: developmental and/or
neurological abnormalities of child prior to seizures, family history of non-febrile seizures and an atypical initial seizure).
Treatment of recurrence: control fever with antipyretics (Tylenol), tepid bath, fluids for comfort only and use Ativan
(lorazepam) 1 mg SL/PO (or diazepam 5-10 mg PR) if a seizure occurs at home. Turn patient onto his/her side, do not force
objects or fingers into mouth. Bring to ER if seizure does not stop within 10 minutes. Seizures do not cause mental
impairment unless they are prolonged (> 30 min), although seizures can be a symptom of brain damage. Patient should be
investigated with CT head and EEG. Prophylactic anticonvulsant therapy is a consideration with repeated seizures.
96. Operating room nurse sustained needle stick injury. Worried about hepatitis and AIDS. Counsel.
History: Name, age, occupation. Determine severity of exposure: hollow bore needle? Needle gauge? Depth of penetration?
Did needle contain blood from a patient? Was any blood injected? Is the HIV and hepatitis status of the patient known? Is the
nurse immunized against Hep B?

47

Patient known to have:


HIV
Hep B
Antigen Hep B
Antibody Hep C

Odds of Transmission
0.3%
40%
10%
5%

Counsel: HIV has a high mortality rate within 5 years of testing positive, and is eventually fatal in most cases, however
patients with HIV have a much longer life expectancy than in the past due to improved anti-retroviral therapy. Hepatitis B
causes fulminant hepatic necrosis in 1% of those infected, which is fatal in 60% of cases. 5% of those infected with Hep B
remain in a chronic carrier state, which is associated with a 25-40% risk of cirrhosis and 2-5% per year risk of hepatocellular
cancer. There is a > 50% risk of chronic liver disease once infected with Hep C, the risks of cirrhosis and hepatocellular cancer
are similar to those for Hep B. The overall risk of transmission is as described above. Recommend baseline testing for HIV,
Hep B, and C in nurse and patient (require consent for HIV testing). If the patient was recently infected, he/she may not
become positive on antigenic testing for up to 3 months. Therefore nurse and patient should be retested. Recommend HIV
prophylaxis (AZT + 3TC x 4 weeks and consult Infectious Disease specialist) for significant needle stick from a patient with
known HIV or who is at high risk (multiple partners, IV drug use, anal intercourse, recent immigrant from endemic area). If
the nurse is not effectively immunized (i.e. antibody titers tested) against Hepatitis B, recommend immunization. If patient is
found to be Hep B or C positive, give the nurse passive immunity gamma globulin, Hep B gamma globulin (HBIG) within 7
days of exposure, has been proven to be effective in preventing transmission.
97. Demonstrate and counsel a patient on breast self-examination and mammography screening.
Breast exam: Sitting: Inspect in four separate positions: 1) sitting with arms at her side, 2) sitting with both arms raised above
her head, 3) sitting with hands pushing on hips and elbows out, 4) patient leaning forward: inspect for size and symmetry,
visible masses/contour changes, skin retraction, erythema, dimpling, nipple retraction/inversion/ulceration/size & shape, peau
dorange around nipple and elsewhere. Palpation of axillary, infraclavicular, supraclavicular nodes. Supine (with pillow under
shoulder), each breast examined separately, drape other breast, small circular motions covering an area of approximately 1
square inch divide into light, medium and deep palpation and perform in all four quadrants. Can denote position of lumps by
clock position with cm distance from nipple. Nipple squeeze to try to exude any fluid from the nipple (ask patient to squeeze
nipple herself). Watch for dimpling, bloody nipple discharge and inflammation.
Mammography: Yearly mammography screening of proven benefit from age 50. Benefit as a screening test equivocal from
age 40 in the general population but is recommended if there is a positive family history of breast cancer. Breast cancer in two
first degree relatives (parents, siblings, children) is an indicator for yearly mammography starting at 5-10 years before youngest
family members presentation.
98. 60 year old male, difficulty walking. Perform a neurological exam.
See neurological exam in question #5.
99. 37 year old well G1P0 female, 9 weeks pregnant. Manage.
History: Patient ID: Planned pregnancy? Status of any relationships at present including relationship with the childs father.
Social supports (family, friends, boyfriend), do they know? Are they helping? Employment/financial/educational status of the
patient, does the patient feel prepared to raise a child? Provisions for care of child when born? GTPAL (number of gestations,
term pregnancies, premature births, abortions, live children), history of problems, if any, with previous pregnancies. Current
pregnancy, establish gestational age (GA) by last menstrual period (LMP) if regular periods and sure dates (if unsure a dating
ultrasound would be needed). The GA is the number of weeks from the first day of the LMP. The EDC is first day of LMP + 7
days 3 months. Smoking (prepared to quit?), alcohol (no alcohol during pregnancy), illicit drugs, diet, exercise, medications
(avoid during pregnancy: including over the counter). Diabetes, family history of inherited disorders, heart disease, circulatory
problems, renal disease, hypertension. Menstrual history, regularity of cycles, how long has patient not used contraception.
Any morning sickness, vaginal bleeding? Past medical and surgical history, medications, drugs/alcohol, smoking, allergies,
family history, review of systems.
Psychiatric: Cover mnemonic for major depression. MSIGECAPS: mood (depressed), sleep (increased or decreasedif
decreased, often early morning awakening), interest (decreased), guilt/worthlessness, energy (decreased or fatigued),
concentration/difficulty making decisions, appetite and/or weight increase or decrease, psychomotor activity (increased or
decreased), suicidal ideation positive diagnosis of major depression requires five of these over a 2 week period, one of the

48

five must be loss of interest or depressed mood. Symptoms do not meet criteria for mixed episode, significant
social/occupational impairment, exclude substance or GMC, not bereavement.
Physical: vitals, weight, height, palpation of neck and thyroid gland, fundoscopic exam, check lid lag, reflexes,
cardiopulmonary exam, breast exam, abdominal exam. Palpate uterus, measure symphysis-umbilicus distance. Doppler for
fetal heart (may not detect until 10 weeks). Vaginal bimanual and speculum exam (cervix should be closed). Pap smear (if
none in last 6 months, use speculum, not brush in os), swab cervix for cultures (GC, chlamydia).
Investigations: CBC, lytes, INR/PTT, urea, creatinine, urinalysis, ECG. Blood group and type, Rh antibodies, VDRL and
HbsAg routine, rubella titer, HIV serology offered, serum folate, urine dip, microscopy and culture, TB skin test in patients
from an endemic area, genetic testing as indicated on history or for sickle cell in blacks. Triple screen (MSS) MSAFP,
hCG, uE3 Trisomy 18, Trisomy 21, NTD (at 16 weeks). Amniocentesis at 15-16 weeks for alpha-fetoprotein and acetyl
cholinesterase. Chorionic villus sampling (10-12 weeks) should be offered given the patients age. Fetal ultrasound.
Counseling: Discuss risk of Downs syndrome due to maternal age, value of fetal genetic testing. Recommend daily
pregnancy vitamin preparation, milk and healthy diet. Do not increase food intake dramatically excessive weight gain not
recommended, 2-3 lbs per month for a total of 25-30 lbs gain in weight ideal. Do not diet during pregnancy. Continue normal
activities and customary exercise. No alcohol, no smoking, no medications of any kind unless discussed with MD. Control
morning sickness with small meals and bland foods. Lying on side decreases swelling and discomfort. Hemorrhoids,
backache, heartburn, increased vaginal discharge are common. Follow-up every 4 weeks until 32 weeks, then increase to every
2 weeks. Call if any concerns or troubling symptoms, especially abdominal pain, vaginal bleeding, persistent headache, illness
or infection.
100. 60 year old female with bloody vaginal discharge. Take a history.
History: Name, age, occupation. Think about: blood dyscrasias, thyroid dysfunction, malignancy, PCOS, endometriosis, PID,
fibroids, unopposed estrogen, or polyps. Onset of bleeding, frequency, estimate quantity (number of pads), color, consistency
of discharge, associated pain, vaginal discomfort, cramping. Previous episodes, history of fibroids, polyps, PID, PCOS. Post
coital and rectal bleeding. Weight loss, night sweats, fatigue. History of easy bruising/bleeding, inherited blood coagulation
disorders. Age of menarche, age of menopause, age of first sexual activity. Use of hormonal replacement therapy, which
preparation? History of fibroids, reproductive tract cancers, last Pap smear. Pregnancy history. Medications, drugs/alcohol,
smoking, past medical history, surgical history, family history, review of systems.
101. 30 year old man with hematemesis and abdominal pain in the emergency department. BP 80/50, tachycardia.
Manage.
Resuscitate as in question #6. Consult gastroenterology for immediate endoscopy.
102. Pregnant woman, 36 weeks gestation, has proteinuria and BP 150/85 (pre-gestational BP 110/65). Manage.
See question #46.
103. A mother is worried that her 1 year old looks pale. Take a history. Finding: breast fed for the first 2 months, then
2% milk. Q: What is the most likely diagnosis? What investigations would you order?
History: Name, age. Feeds and feeding history (esp. fruit juice, excess milk). Growth pattern: weight loss? Diarrhea?
(consistency, color, quantity and frequency), blood in stool, melena stools, concurrent illness, vomiting, fever, anorexia,
difficulty breathing, lassitude, dry mouth/eyes, low urine output, illness affecting other children in the family or adults. Recent
immunization, travel, antibiotics. Medications, past medical history, allergies, birth history, pregnancy problems, maternal
illness during pregnancy, family history, review of systems.
Most likely diagnosis: Iron deficiency anemia (most common cause of childhood anemia). Typically in bottle-fed infants (6-24
months) receiving large volumes of cows milk should add iron-fortified cereal and iron rich foods starting at 6 months.
Investigations: CBC with peripheral smear, lytes, urea, creatinine, INR/PTT, serum ferritin, albumin.
104. Elderly man with creatinine 1000. Take a history. Q: Give a differential diagnosis. What investigations would you
order?

49

History: Patient ID. suprapubic pain, pain on urination, frequency, urgency, frank blood in the urine (globular clots from
bladder or string shaped clots from ureters), color of urine, difficulty initiating or maintaining urinary stream, renal pain, groin
pain. Provoking factors. Associated symptoms including saddle anesthesia, loss of bowel control. History of recent UTI, STDs,
TB exposure, pelvic irradiation, bleeding diathesis, smoking. Fever, chills, nausea, fatigue. Previous renal colic/diagnosed
nephrolithiasis? History of hypercalcemia, hypertension. Malignant symptoms: night sweats, weight loss, fatigue.
Medications, drugs (NSAIDs, anticoagulants)/alcohol, smoking, past medical history, past surgical history, family history
(polycystic kidney disease?), review of systems.
Differential diagnosis:
1. Pre-renal: Hypovolemia, poor cardiac output, renovascular disease, NSAID/ACEi use, liver failure.
2. Renal: Vascular malignant HTN, cholesterol emboli, HUS/TTP; Tubulo-interstial ATN (ischemic/toxin
endogenous/exogenous), AIN; Glomerular (< 5%). Causes: X-ray contrast, myoglobinuria, acute glomerulonephritis, DIC,
pyelonephritis, intrarenal precipitation in hypercalcemia, myeloma.
3. Post-renal: Obstruction: upper (clot, tumor, stone, external compression), lower (BPH, clot, stone, stricture, autonomic
dysfunction).
Investigations: CBC, lytes, urea, creatinine, phosphate, ionized Ca++, magnesium, INR/PTT, AST, ALT, ALP, GGT, prostate
specific antigen, CK-MB, troponin, ABG. Urinalysis: microscopy, dip, culture and sensitivity. Abdominal x-ray, abdominal
pelvic ultrasound. Post-void catheterization. (Avoid IVP due to dye).
105. 1 year old boy with 6 months diarrhea. Take a history. Q: Give a differential diagnosis.
See question #50.
106. 58 year old lady in hospital 4 days post-op hysterectomy for fibroids. Agitated, had tactile hallucinations the
previous night. Take a history. Finding: history of alcoholism. Q: What is the most likely diagnosis?
History: onset of hallucinations, duration, description. Tactile hallucinations or bugs crawling on skin or on ceiling suggest
alcohol withdrawal. Associated fever, agitation, sweating, tremor, decreased level of consciousness, seizure? Any problems
with surgical recovery, wound healing, mobilization? Amount of alcohol consumed at home. History of alcoholism, leg
swelling, SOB, chest pain. Current state. Post-op medications (morphine, Demerol) previous bad reactions to these or to
antibiotics? Previous episode like this one? Past medical history, medications, drug and alcohol use, smoking, allergy, family
history, review of systems.
Most likely diagnosis: alcohol withdrawal.
107. Young man with a swollen cervical lymph node. Perform a focused physical exam. Q: CXR shows mediastinal
widening with perihilar nodes. Describe. Give five features on history which would be helpful for diagnosis.
Physical exam: vitals, jaundice, nutritional status, buccal mucosa, teeth, breath (hepatic fetor), parotid hypertrophy, glossitis,
inspect chest for telangectasia, gynecomastia, loss of axillary hair. Hands: palmar erythema, clubbing, Dupuytrens
contracture, wasting of hand intrinsics. Palpate for lymph nodes in the neck, supra and infra-clavicular, axillae, groin. Examine
the oral cavity and pharynx. Check for rashes.
Abdominal exam (supine): see question #29.
Differential diagnosis: lymphoma, leukemia, viral infection (mononucleosis, HIV, EBV), inflammatory autoimmune disease
(sarcoidosis, lupus), serum sickness (severe allergic reaction short of anaphylaxis), TB, liver disease with portal hypertension.
Five features on history helpful for diagnosis: viral prodrome, family history of sarcoid, lymph nodes painful, bone pain,
pruritis, weight loss.
108. Female patient found to have a nodule on routine CXR. Perform a focused physical exam. Q: Give a differential
diagnosis. What investigations would you order?
Cardiopulmonary exam as in question #13 and #24. See also question #49.

50

Investigations: old CXR for comparison (if lesion is old and unchanging, interventions are less aggressive, calcification is also
associated with benign lesions such as old granulomas), CT chest with CT guided needle biopsy, sputum for cytology and acidfast staining (TB), TB skin test, bronchoscopy with biopsy and washings if lesion seen, open biopsy or lobectomy.
Algorithm: solitary nodule previous CXR benign or unchanged (repeat in q3-6months for 2 years if unchanged observe,
if changed at any time continue), malignant or changed CT thorax: cancer (stage and treat), calcification (observe), no
diagnosis bronchoscopy or transthoracic needle aspiration still no diagnosis (resect for diagnosis), inflammatory (treat
cause), cancer (stage and treat).
109. 60 year old male slipped and fell 6 days ago. Comes to you because of hemoptysis. Perform a focused physical
exam. Finding: positive Homans sign. Q: What is the most likely diagnosis? Give a plan form management.
Cardiopulmonary exam as in question #13, plus additional attention to calf size, tenderness, redness and pleuritic chest pain.
Homans sign: pain in the calf on dorsiflexion of the foot indicates thrombophlebitis. Check that trachea is midline. Is the
patient on DVT prophylaxis or anti-coagulation?
Most likely diagnosis: pulmonary embolus.
Specific investigations for PE: CT chest (only shows clinically significant PE), V/Q scan (conclusive when is shows high or
low probability), pulmonary angiogram (gold standard but invasive), ECHO, and serial (q2d) leg Dopplers for presence of
DVT above the knee. Others: CXR (often normal, Hamptons hump, Westermarks sign, rarely dilatation of proximal PA),
ECG (sinus tachycardia, S1Q3T3), ABG (PaO2 usually decreased, PaCO2 decreased due to increase in overall minute
ventilation, increased A-a gradient), D-dimer. See also question #17.
Treatment: if suspicion of PE is high, anticoagulate before waiting for these tests with heparin 7500 U IV bolus (80 U/kg),
then infuse at 1200 U/hr (18 U/kg/h). Measure PTT q6h, adjust dose for PTT 70-90s (2.5-3 times normal baseline). Start
coumadin, to INR 2-3, continue coumadin for 3 months.
110. Telephone in room. Mother calls because her child has just ingested a caustic cleaner. Manage over the phone. Q:
What do you do after hanging up the telephone? Give a plan for management.
See question #52.
111. Telephone in the room. Physician in a peripheral center calls wishing to transfer an unstable patient who has been
in a motor vehicle accident. Manage over the phone. Q: CXR shows opacification of the right lung. What is your
diagnosis? Give the immediate management of this problem.
Over the telephone: Physicians name, name of center, patients name. Injuries, investigations done, vitals, lab values. GCS,
is patient intubated? Peripheral physician must not transfer patient until he is stabilized, i.e. good BP, good oxygen sats,
bleeding controlled, blood products given as needed. Estimated time of arrival? Physician accompanied?
CXR: likely hemothorax.
Treatment: Chest tube in ER, drain hematoma and connect to suction through a bubble chamber. Consult thoracic surgery,
prepare patient in case of immediate OR.
112. Young woman with bilateral migratory arthritis of recent onset. Take a history. Q: Give a differential diagnosis.
What investigations would you order?
Note: Migratory arthritis suggests gonococcal infection.
History: Patient ID. Onset of arthritic symptoms, durations, joints affected, chronology. Associated fever, malaise, fatigue,
rash, abdominal pain and cramps, vaginal discharge, pain with urination, dyspareunia (painful intercourse). History of arthritis
(rheumatoid, osteoarthritis), psoriasis, Lyme disease (camping trips), Reiters syndrome, ankylosing spondylitis, sexually
transmitted diseases including PID. Sexual history: present partners, number of partners, fidelity of partner(s), use of condoms.
Medications, drugs/alcohol, smoking, allergies, past medical history, family history, review of systems.

51

Differential diagnosis: gonococcal arthritis, psoriatic arthritis, Lyme disease, Reiters syndrome, ankylosing spondylitis,
rheumatoid arthritis, osteoarthritis, gout.
Investigations: CBC, ESR, lytes, urea, creatinine, INR/PTT, blood cultures. Cervical swab for culture and sensitivity. Joint
aspirate for microscopy and culture.

1999
113. 48 year old woman with multiple complaints. Has had negative investigations by several other doctors. Take a
history and perform a mental status examination. Q: Without looking at the patient again, describe her appearance.
What is your diagnosis?
114. 32 year old mother presents to your office because her 4 year old son has had an allergic reaction to peanuts. He
was brought to the emergency departement and treated yesterday. He is now well. Counsel.
115. 30 year old female with lower abdominal pain for 1 week. Perform a physical exam. Findings include distended
abdomen and tenderness at McBurneys. Q: The patient has a history of Crohns and presents with the following
abdominal x-ray (shows small bowel obstruction), what is the diagnosis?
116. A 2 day old infant with jaundice and a serum bilirubin of 220 mol/L (ref. Max 200 mol/L). Take a history from
the mother. Q: What are the possible causes for this abnormality? Give investigations and counsel.
See Question #25.
117. 35 year old male outpatient with shortness of breath, cough, sputum. Take a history and perform a physical exam.
X-ray findings: PCP. Q: Recommend further investigations and treatment.
118. 66 year old male with symptoms of claudication. Do a focused physical exam. Q: What 2 investigations would be
most appropriate. What 5 risk factors on history would point to the diagnosis.
119. Mother wants to go on a camping trip with her son who suffers from enuresis. Counsel.
120. Young female with secondary amenorrhea for 6 months. Take a history. Q: What investigations would you order.
Give a differential diagnosis. What is the most likely diagnosis, what results would confirm this diagnosis? Counsel
with regards to OCP.
See question #86.
121. Telephone in the room. Nurse in a peripheral center calls wishing to transfer a patient who has had a febrile
seizure. Manage over the phone. Counsel to get patient ready for transfer.
122. 25 year old male has had a scaffold fall on him. He presents with chest pain and would like something for the pain.
Normal respiratory exam and normal cardiac exam. Manage.
123. 30 year old G1P0 with pre-eclampsia. Counsel.
See question #46.
124. 40 year old male just passing through wants Tylenol #3 to hold him until next week. Manage.
See question #16.

2004 - October
125. 39 year old woman who would like to quit smoking. Counsel.
History: Name, age, occupation. Smoking habits: amount, duration, frequency, time of day. Gain from smoking (e.g. weight
loss, decreased anxiety, social relationships). Personal concerns about smoking and quitting, foreseen benefits from quitting,
interest in quitting (a person will only quit if they are willing). Previous attempts and results, medical situation: SOB, cough,

52

asthma, COPD, HTN. Social situation: other smokers in family/social network. Nicotine dependence: preoccupation or
compulsion to use, impairment or loss of control over use, continued use despite negative consequences, minimization or
denial of problems associated with use. Past medical history, allergies, medications, alcohol/drug use, family history, review of
systems.
Counseling: 2 important components that need to be addressed: 1) physical/chemical addiction: symptoms of withdrawal
(tremors/irritability) and 2) habitual/environmental factors: psychological, social, and spiritual components. Advise patient of
health risks: Smoking is the single most preventable cause of death responsible for 80% of lung cancers, COPD, cardiovascular
disease, also a factor in PUD, low birth weight babies, premature aging, upper GI/respiratory cancers, respiratory infections,
SIDS. Ages 25-34 have highest prevalence of smoking, 15% of smokers smoke > 25 cigarettes/day. After assessing smoking
habits advise every smoker to quit at every visit assess stage of change (see table below). Motivate smoker to attempt to
quit: benefits include decreased respiratory infections, increased exercise tolerance/energy, increased taste/smell, ask for a
commitment to quit (set a date), assist the smoker to quit (physician counseling).
Begin with self-help materials: remove ashtrays/lighters, increase high fiber snacks/gum, increase aerobic exercise, selfreward, may also use nicotine patch/gum or attend smoking withdrawal programs. Reward goals that are met: plan for new
social relationships and activities to make it easier to make a serious attempt to change behavior. Follow-up: set firm dates.
Anticipate problems: weight gain, withdrawal symptoms. Continue to monitor/support. Do not give up if failed. Most
relapses occur in first year; most people try > 5 times before quitting.
Stages of Change Model
Barriers
Motivational drift
Low social support
Lack of perceived self-efficacy
Attitudes and emotions
Lack of knowledge
Denial/trivialization
Perceived invulnerability
Faulty conceptions

Maintenance

Action

Contemplation

Precontemplation

Processes
Reinforcement management
Helping relationship
Reinforcement
Self evaluation
Consciousness raising

Treatment:
Nicotine patch continuous self-regulated amount of nicotine, decreases craving and/or withdrawal, will not replace
immediate effects of smoking, habit or pleasure. Indications: nicotine dependent, high motivation to quit smoking.
Contraindications: smoking while on patch, allergy, MI, CVA. Relative contraindication: pregnancy. Duration of treatment: 412 weeks usually adequate.
Zyban (bupropion) approved in Canada in August 1998 acts on dopaminergic (reward) and noradrenergic (withdrawal)
pathways. Contraindications: seizure disorder, alcoholism, eating disorder, recent MAOI use, current pregnancy; caution if
using SSRI (reduction of seizure threshold). Dose varies with amount the patient smokes. Patient continues to smoke for first
week of treatment and then completely stops (therapeutic levels reached in one week). Recommend abstinence from alcohol
due to risk of toxic levels with liver dysfunction. Side effects: headache, insomnia, dry mouth, weight gain.
126. 65 year old woman post hysterectomy having hallucinations at night for the last two nights. Take a history. Q:
Most likely diagnosis? If this woman wants to go home from hosptial and is medically cleared is she competent to make
that decision? Would you allow her to go home?
History: onset of hallucinations, duration, description. Associated fever, agitation, sweating, tremor, decreased level of
consciousness, seizure? Misperceptions and illusions, impaired attention span, disorientation, impaired level of consciousness,
delusional thinking, affective symptoms (mood sad). Fluctuating course? Any problems with surgical recovery, wound
healing, mobilization? Amount of alcohol consumed at home. History of alcoholism, leg swelling, SOB, chest pain. Current
state. Post-op medications (morphine, Demerol) previous bad reactions to these or to antibiotics? Previous episode like this
one? Past medical history, medications, drug and alcohol use, smoking, allergy, family history, review of systems.
Most likely diagnosis: delirium.

53

Differential Diagnosis: I WATCH DEATH: I = infectious (encephalitis, meningitis, UTI, pneumonia), W withdrawal
(alcohol, barbiturates, benzodiazepines), A acute metabolic disorder (lytes, hepatic/renal failure), T trauma (head injury,
postop), C CNS pathology (stroke, hemorrhage, tumor, seizure, Parkinsons), H hypoxia (anemia, cardiac failure,
pulmonary embolus), D deficiencies (vit. B12, folic acid, thiamine), E endocrinopathies (thyroid, glucose, parathyroid,
adrenal), A acute vascular (shock, vasculitis, hypertensive encephalopathy), T - toxins, substance abuse, medication (alcohol,
anesthetics, anticholinergics, narcotics), H Heavy metals (arsenic, lead, mercury).
Investigations: CBC, lytes, calcium, phosphate, magnesium, glucose, ESR, liver/renal tests, urinalysis, ECG. As indicated by
history: TSH, CT head, toxicology screen, VRDL, LP, LE preparation, B12 and folic acid levels, EEG.
Management: Treat underlying cause (GMC etc.). Stop all non-essential medications. Maintain nutrition, hydration,
electrolyte balance and monitor vitals. Psychosocial quiet/well lit environment, room close to observation, family member
for reassurance. Pharmacological haloperidol 2-5 mg IM, lorazepam 1 mg SL, physical restraints if patient violent.
127. 30 year old man with abdominal pain in the emergency department. Findings: Diaphoretic. BP 80/50, tachycardia.
Manage.
128. 32 year old mother presents to your office because her 4 year old son has had an allergic reaction to peanuts. He
was brought to the emergency departement and treated yesterday. He is now well. Counsel.
129. 35 year old woman feels depressed. Findings: Recently lost her father. Take a focused history including a mental
status exam.
130. 35 year old male with back pain and stiffness. Take history and perform a focused physical exam. Findings: 10 cm
separation between lumbar spines while erect increases by less than 5 cm when back is flexed forward (positive WrightSchober test), lateral flexion impaired. Q: Give the diagnosis and two associated conditions.
131. 79 year old female collapses in the mall. Patient is drowsy, unresponsive to verbal stimuli. She is there with her
grand-daughter. Findings: HR 40, BP 80/40, ECG complete heart block. Manage.
132. A young man sustains a head injury on falling from his bicycle. Patient has been hemodynamically stabilized.
Perform a focused neurological exam. Q: Lateral skull and lateral C-spine X-rays provided. Are they adequate? Are
they normal? The patient has continuing sanguinous discharge from his nose. What is the likely cause of this.
Treatment?
133. 30 year old woman with vaginal bleeding at 30 weeks gestation. Take a history. Q: Give a differential diagnosis.
Order investigations.
134. 65 year old male outpatient with shortness of breath, cough, sputum. Take a history and perform a physical exam.
Findings: Lobar consolidation with yellow-green sputum. Q: Given a diagnosis of pneumonia, recommend treatment.
135. Mother with 6 month old child with failure to thrive. Take a history. Q: Give a differential diagnosis. Recommend
treatment.
136. Abcess station. Perform an irrigation and drainage.

2004 - May
137. 77 year old female admitted for resection of colon cancer which presented with painless bleeding per rectum has
decided to withdraw her consent for surgery. She has been seen by psychiatry and deemed competent, you are covering
for her family doctor, counsel.
138. 37 year old female 9 weeks pregnant is worried about genetic problems more frequent in women becoming
pregnant later in life and is asking about genetic testing. Counsel.
139. 30 year old male has had 1 week of worsening hip pain which has progressed to failure to weight bear. Vitals: BP
130/80, P 85, T 39.5. Conduct a focused history and physical. Q: What is at the top of your differential. What single
investigation would help confirm your suspicion.

54

140. 45 year old female presents to your office with difficulty sleeping. Take a focused history. Q: What is the likely
diagnosis and what is the single most appropriate outpatient treatment.
141. Mother of 3 year old presents because her child seems to have delayed speech development compared to an older
brother. She is worried that her child may be retarded. Take a history and counsel.
142. 56 year old male patient on Warfarin for atrial fibrillation presents to your office with 3 day history of coughing up
blood. Take a history and perform a focused physical examination. Q: This patient is unhappy with his care and would
like an original copy of his chart so he may go to another physician. What do you do.
143. 25 year old male has fallen 40 feet (12 meters) off his parents roof while helping with repairs. He presents with
shortness of breath, tachypnea, and vitals as follows: BP 80/40 P 130 RR 20. Manage.
144. 56 year old male presents to the ER with chest pain which seems to have resolved partially with Nitro spray
administered en route by EMS. He has a normal ECG (which may show some mild changes in the inferior leads).
Manage. Second ECG taken after another bout of chest pain shows clear ST elevation in 3 consecutive inferior leads
(II, III, aVF). Q: After receiving treatment but before Cardiolgy has seen this patient he wants to discharge himself
despite the risks of doing so. What do you do?
145. Mother of 2 year old has brought her son back to see you when her sons cough has failed to resolve after 4 weeks.
He has received antibiotic treatment (Amoxicillin) and has been using a cough suppressant without improvement.
Findings: Cough is non productive but loose. History of eczema-like rash in child and mother. Father smokes. Q: Single
most appropriate diagnosis. What information on history leads you to this conclusion.
146. 23 year old female with 24 hours of progressive abdominal pain. Do a focused physical exam. Findings: Patient in
fetal position. LLQ pain. Guarding, rebound. Q: Differential diagnosis. Long term consequences of the most likely
diagnosis.
147. 66 year old male with symptoms of claudication. Do a focused physical exam. Q: What 2 investigations would be
most appropriate. What 5 risk factors on history would point to the diagnosis.
148. 21 year old male brought into the emergency department by a friend due to neck stiffness. Take a focused history.
Findings: Patient is on haldol. Q: What is the most likely cause of this patients neck stiffness. Outline a
management plan for this patient. The patient is treated and discharged but 2 hours later his mother calls because he
was out on the balcony trying to fly, what if any are your professional responsibilities to this patient.
149. 33 year old male with elevated LFTs on routine blood work while applying for life insurance. The patient is upset
about these results and would like you to fix it. Labs: AST 110 ALT 170 ALP 100 Bili 26. Findings: patient is
consuming approximately 20 alcoholic beverages per week. Father died at 56 years old of cirrhosis. Q: Differential
diagnosis (top three). Given the above labs what are the top three likley causes of these elevated enzymes.
150. 71 year old female who has not had a family doctor for three years. Her daughter who is your patient and has
asked that you see her because her mother is worried about problems with her memory and is worried about developing
dementia. Findings: Previously got a shot once a month and used to take thyroid pills but has not kept up with
these since her family doctor moved. Q: Most likely diagnosis. What are two likely causes of this diagnosis. What two
tests would you order to confirm your suspicions.

Other Cases:
151. 42 year old male post surgical receiving blood products, has a reaction. Manage.
History: Name, age, occupation. What is the reaction? Chills, fever, urticarial, rash, anaphylaxis, muscle pain, back pain, N/V,
chest pain, wheezing, dyspnea, tachypnea (ARDS), feeling of impending doom, hemoglobinuria, renal failure DIC. When
was the transfusion started and how long afterwards did the reaction occur? Why is the patient receiving blood? Recent
bleeding or surgery? Focused history of current medical problems and treatments. How much has been received and was the
blood checked. Stop transfusion and re-check blood type in bag for obvious clerical errors (most common cause of transfusion
reactions is still clerical error).

55

There are multiple types of reactions and treatment varies according to the type of reaction. You must determine the type of
reaction by the time course and symptoms:
Acute complications of blood transfusions
Cause
Febrile Non-hemolytic
Antibodies stimulated by previous exposure
transfusion reactions
against antigens on donor lymphocytes,
granulocytes, platelets or to lymphokines

Signs, symptoms
Chills, fever

Management
Stop transfusion
Acetaminophen
Steroids
Filtered blood
Washed blood
Antihistamines
Slow infusion
Steroids
Washed blood
IV epinephrine
IgA deficient blood components in
future
Stop infusion
Hydrate aggressively

Allergic (urticarial)
reactions

Interaction between donor plasma proteins


and recipient IgE antibodies

Itching, rash

Anaphylaxis

Rare, usually IgA deficient patients reacting


against IgA in donor plasma

Bronchospasm
Shock

Acute hemolytic
transfusion reactions

Usually due to incorrect patient/blood


identification hemolysis due to
complement activation

Citrate toxicity

Toxicity secondary to hypocalcemia

Muscle pain, back pain, chest pain


Fever, N/V, wheezing
Dyspnea, tachypnea
Feeling of impending doom
Hemoglobinemia, renal failure
Hypocalcemia signs

Hyperkalemia

High levels of potassium in stored blood


due to hemolysis

Hyperkalemia signs

Circulatory overload

With prior CHF and in elderly

Signs of CHF

Dilutional
coagulopathy
Bacterial infections

Seen with massive transfusion, packed cells


contain no factor VIII or V or platelets
Never give blood > 4 hours after a bag has
been entered!

Coagulopathy

Prevented by giving 10 mL of 10%


calcium gluconate for every 2 units
of blood
Kayexalate
Calcium gluconate
Insulin + glucose
Salbutamol
Minimize amount of saline given
with the blood
Correct with FFP and platelets

Chills, rigors, fever, hypotension,


shock, DIC (profound symptoms
with gram negatives)

Broad spectrum Abx


Blood culture and sensitivity
Resuscitation for shock

Delayed complications from transfusions include (days to weeks):


1. Viral infection: HIV < 1:500,000; HBV < 1:250,000; HCV: < 1:10,000.
2. Delayed hemolytic transfusion reaction 5-10 days weak alloantibodies anemia, fever, history of recent
transfusion, jaundice, positive direct Coombs test.
3. Iron overload repeated transfusions over long periods of time secondary hemochromatosis (dilated
cardiomyopathy, cirrhosis, DM, hypothyroidism, delayed growth and puberty) use of iron chelators after
transfusion can reduce chance of overload.
4. Graft versus host disease transfused T-lymphocytes attack host 4-30 days later usually in
immunocompromised fever, diarrhea, liver function abnormalities, pancytopenia 90% mortality prevention
with gamma irradiation of blood components

56

Vous aimerez peut-être aussi